135

Fisica_I

Embed Size (px)

Citation preview

Page 1: Fisica_I
Page 2: Fisica_I

Múltiplos y prefijos de unidades métricas* Unidades base del SI

Múltiplo Prefijo (y abreviatura) Cantidad física Nombre de la unidad Símbolo

Longitud .\1asa Tiempo Corriente eléctrica Temperatura Cantidad de sustancia Intensidad luminosa

metro kilogramo segundo ampere kelvin mol candela

m kg s A K mol cd

Algunas unidades derivadas del SI

Nombre de Cantidad física la unidad Símbolo Unidad SI

Frecuencia hertz Hz s-1

Energía joule J kg-m2/s2

Fuerza newton N kg-m/s2

Presión pascal Pa kg/(m's2 )

Potencia watt W kg-m2/s3

Carga eléctrica coulomb C A-s Potencial eléctrico volt V kg-m2

/ (A-S3)

Resistencia eléctrica ohm D kg'm2/(A2_ S3)

Capacitancia farad F A2's4/(kg-m2)

Inductancia henIY H kg'm2/(A2's2 )

Campo magnético tesla T kg/(A-s2)

10 2';

10 21

1016

1013

1012

109

10 6

lO" 10 2

10

10-1

10-2

10-3

10-6

10-9

10-12

10-15

10-18

10-21

10-24

yota- (Y) zeta- (Z) exa- (E)

peta- (P) tera- (T) giga- (G) mega- (M)

kilo- (k) hecto- (h) deca- (da)

deci- (d) centi- (c) mili- (m) micro- (J.l) nano- (n) pico- (p) femto- (f) ato- (a) zepto- (z) yocto- (y)

'Por ejemplo, 1 gramo (g) multiplicado pOl~ 1000 (l03) es 1 kilogramo (kg); 1 gramo multiplicado por 1/1000 (l0-") es 1 miligramo (mg).

Fórmula cuadrática . -.-,' _.', -, __ ~: ._.'. ,- -- ',' .

Si ax2 + bx -'- e = O, entonces

- b ± \ [,2 - .fIJe. x==

21J

Definiciones de funciones trigonométricas

j( x sen 8 Ysen e = :... cos 8 = ­ tan8=--=­

l' r cos 8 x

8° (rad) sen 8 cos 8 tan 8 segundo cuadrante primer cuadrante

0° (O) O 1 O

30° Crr-j6) 0.500 \,13/2 = 0.866 V313 = 0.577 1"

Y

45° (17/4) V2/2 = 0.707 V2/2 = 0.707 1.00 e

x x

60° (17/3) 'vI3/2 = 0.866 0.500 V3=1.73

90° (77/2) 1 o YO tercer cuadrante cuarto cuadrante

Page 3: Fisica_I

Factores de conversión

Masa

Longitud

Área

Volumen

Tiempo

Ángulo

Rapidez

Fuerza

1 kg = 103 g 1 u = 1.66 X 10-24 g = 1.66 X 10-27 kg 1 tonelada métrica = 1000 kg

1 g = 10-3 kg

Presión

1 cm = 10-2 m = 0.394 pulg 1 m = 10'3 km = 3.28 ft = 39.4 pulg 1 km = 103 m = 0.621 mi 1 pulg = 2.54 cm = 2.54 X 10-2 m 1 ft = 0.305 m = 30.5 cm 1 mi = 5280 ft = 1609 m = 1.609 km

1 nm = 10-9 m

1 cm2 = 10-4 m2 = 0.1550 pulg2

= 1.08 X 10-3 ft2 1 m2 = 10 4 cm2 = 10.76 ft2 = 1550 pulg2 Energía 1 pulg2 = 6.94 X 10-3 ft 2 = 6.45 cm2

2= 6.45 X 10-4 m1 ft2 = 144 pulg2 = 9.29 X 10-2m2 = 929 cm2

1 cm3 = 10-6 m3 = 3.35 x 10-5 ft 3

= 6.10 X 10-2pulg3

31 m3 = 10 6 cm = 10 3 L = 35.3 ft 3

= 6.10 X 10 4 pulg3 = 264 gal 1 litro = 10 3 cm3 = 10 3 m3 = 1.056 ct

= 0.264 gal = 0.0353 ft 3 Potencia

1 pulg3 = 5.79 X 10-4 ft 3 = 16.4 cm3 3= 1.64 X 10-5 m

1 ft3 = 1728 pulg3 = 7.48 gal = 0.0283 m3

= 28.3 L 31 ct = 2 pt = 946 cm = 0.946 L

Equivalentes1 gal = 4 ct = 231 pulg3 = 0.134 ft3 = 3.785 L masa-energía

1 h = 60 min = 3600 s 1 día = 24 h = 1440 min = 8.64 X 104 s 1 año = 365 días = 8.76 x 10 3 h

= 5.26 X 10 5 min = 3.16 X 10 7 s

1 rad = 57.30

10 = 0.0175 rad 60" = 17/3 rad Temperatura

150 = 17/12 rad 90° = 17/2 rad 30° = 17/6 rad 1800 = 17 rad 45° = 17/4 rad 3600 = 217 rad

1 rev/min = (17/30) rad/s = 0.1047 rad/s Fuerza egs

1 m/s = 3.60 km/h = 3.28 ftis Energía cgs = 2.24 mi/h

1 km/h = 0.278 m/s = 0.621 mi/h = 0.911 ft/s

1 ft/s = 0.682 mi/h = 0.305 m/s = 1.10 kmih

1 mi/h = 1.467 ft/s = 1.609 km/h = 0.447 m/s

60 mi/h = 88 ft/s

1 N = 0.225 lb 1 lb = 4.45 I\' Peso equivalente de una masa de 1 kg en

la superficie de la Tierra = 2.2 lb = 9.8 N

1 Pa (N/m2) = 1.45 X 10-4 lb/pulg2

= 7.5 X 10-3 ton (mm Hg) 1 ton (mm Hg) = 133 Pa (N/m2)

= 0.02 lb/pulg2

1 atm = 14.71b/pulg2 = 1.013 X 105 N/m2

= 30 pulg Hg = 76 cm Hg 11b/pulg2 = 6.90 X 10 5 Pa (N/m2) 1 bar = 10; Pa 1 milibar = 10 2 Pa

1 J = 0.738 ft·lb = 0.239 cal = 9.48 X 10-4 Btu = 6.24 X 1018 eV

1 kcal = 4186 J = 3.968 Btu 1 Btu = 1055 J = 778 ft·lb = 0.252 kcal 1 cal = 4.186 J = 3.97 X 10-3 Btu

= 3.09 ft'lb 1 ft·lb = 1.36 J = 1.29 X 10-3 Btu 1 eV = 1.60 X 10~9 J 1 kVVh = 3.6 X 10 6 J

1 'vV = 0.738 ft'lbis = 1.34 X 10-3 hp = 3.41 Btu/h

1 ft·lb/s = 1.36 W = 1.82 X 10-3 hp 1 hp = 550 ft'lb/s = 745.7 W

= 2545 Btu/h

1 u = 1.66 X 10 27 kg ~ 931.5 MeV 1 masa de electrón = 9.11 X 10-31 kg

= 5.49 X 10.4 u ~ 0.511 MeV 1 masa de protón = 1.67262 X 10-27 kg

= 1.007276 u ~ 938.27 MeV 1 masa de neutrón = 1.67493 X 10-27 kg

= 1.008665 u ~ 939.57 MeV

TF=~Te+32

Te = ~ (TF - 32)

h = Te + 273

1 dina = 10-5 ~ = 2.25 X 10-6 lb

1 erg = 10-7J = 7.38 X 10 6 ft'lb

Page 4: Fisica_I

Física 1

Page 5: Fisica_I
Page 6: Fisica_I

Física 1

Jerry D. Wilson Lander University

Greenwood, SC

Anthony J. Buffa California Polytechnic State University

San Luis Obispo, CA

con Bo Lou Ferris State University

Big Rapids, YrI

TRADUCCIÓN Roberto Luis Escalona García Traductor profesional en ciencias Virgilio González Pozo Universidad Nacional Autónoma de México José de la Cera Alonso Cniversidad Nacional Autónoma de México

REVISIÓ:\J TÉC:\JICA M.E.S. Ernesto Araujo Castillo M.E.e. Luciano Escamilla Reyes M.e. César González Simián M.e. Ludovico González Simián M.e. Carlos Mata Martínez M.E.e. Moisés Navarro Guzmán

Comité de Física Universidad Autónoma de Nuevo León

®

México' Argentina' Brasil' Colombia' Costa Rica' Chile' Ecuador España' Guatemala' Panamá· Perú' Puerto Rico' Uruguay' Venezuela

Page 7: Fisica_I

/ Datos de catalogación bibliográfica

WILSON, JERRY D., ANTHOI'¡Y J. BliFFA Y BOLOU Física 1.

PEARSON EDUCACIÓN, México, 2008

ISBN: 978-970-26-1172-1 Área: Bachillerato

Formato: 21 x 27 cm Páginas: 144

Authorized adaptation from the English language eclition entitled College Physics, Fifth Edition, /n) Jem) D. Wilson and Anthol1Y J. Buffa, published by Pearson Education, Inc., publishing as Prentice Hall Inc., Copyright © 2002. ISBN 013-067644-6

Ésta es ~a adaptación de la obra Física, quinta edición, de Jem) D. Wilson y Allthony J. BlIf!a, traducida y publicada por Pearson Educación de México, 2003 © como Prentice Hall © 2003 ISB:'-i 970-26-0425-7.

Todos los derechos reservados. Esta eclición en español es la única autorizada.

Edición en español Editor: Hugo Rivera Oli,'er

e-mail: [email protected] Editor de desarrollo: Bernardino Gutiérrez Hernández Supervisor de producción: Enrique Treja Hernández

PRIMERA EDICIÓN, 2008

D.R. © 2008 por Pearson Educación de México, S.A. de c.v. Atlacomulco 500-50. Piso Col. Industrial Atoto c.P. 53519, Naucalpan de Juárez, Edo. de México

Cámara Nacional de la Industria Editorial Mexicana. Reg. l\;únl. 1031.

Prentice Hall es una marca registrada de Pearson Educación de México, S.A. de c.v.

Reservados todos los derechos. Ni la totalidad ni parte de esta publicación pueden reproducirse, registrarse o transmitirse, por un sistema de recuperación de información, en ninguna forma ni por ningún medio, sea electrónico, mecánico, fotoquímico, magnético o electroóptico, por fotocopia, grabación o cualquier otro, sin permiso pre'-io por escrito del editor.

El préstamo, alquiler o cualquier otra forma de cesión de uso de este ejemplar requerirá también la autorización del eclitor o de sus representantes.

ISBN 10: 970-26-1172-5 ISBN 13: 978-970-26-1172-1

Impreso en México. Pl'inted in Mexico.

1 2 3 4 5 6 789 O- 10 09 08

®

o AGO

EDITORIAL IMPRESORAAPOLO, SA DE C.v. CENTENO No. 150·6 COL. GRANJAS ESMERALDA 09810 MÉXICO, D.F.

200a

e

Page 8: Fisica_I

Acerca de los autores Jerry D. Wilson, nativo de Omo, es Profesor Emérito de Fí­sica y ex director de la División de Ciencias Biológicas y Físicas de la Lander University en Greenwood, Carolina del Sur. Ob­tuvo su licenciatura en la Omo University, la maestría en la Union College y, en 1970, el doctorado en la Omo Universit)'­Su maestría la obtuvo mientras se desempeñaba como físico del comportamiento de materiales en la General Electric Co.

Como estudiante de doctorado, el profesor Wilson de­sempeñó el puesto de instructor y comenzó a impartir cursos de física. En esta época, fue coautor de un texto de física que ya ha alcanzado su décima edición. Junto con su carrera do­cente, el profesor Wilson siguió escribiendo y ha sido autor o

coautor de seis títulos. Habiéndose retirado de la docencia de tiempo completo, sigue escribiendo y produciendo, entre otras obras, The Curiosity Comer, una columna sema­nal para diarios locales que también se publica en Internet.

Habiendo varios libros de texto competitivos, seguramente habrá quien pregunte por qué opté por ser coautor de otro texto de física basado en álgebra. He impartido varios cursos de in­troducción a la física y conozco bien las necesidades de los estudiantes y las dificultades que ei~fren­tan para dominar la materia. Decidí escribir un libro de texto que presenta los principios básicos de la física de forma clara y concisa, con ejemplos ilustrativos que ayudan a superar la principal dificultad en el aprendizaje de la física. la resolución de problemas. También quería escribir un libro de texto pertinente para las situaciones del mundo real, afin de mostrar a los estudiantes la importancia de la física en su mundo cotidiano, cómo funcionan las cosas y por qué suceden las cosas. Una vez aprendidos los fundamentos, es fácil entender tales aplicaciones.

-Jerry Wilson

Anthony J. Buffa recibió su grado de licenciatura en física del Rensselaer Polytechnic Institute y tanto su maestría como el doctorado en física de la University of illinois, Crbana­Champaign. En 1970, el profesor Buffa se unió al profesorado de la California Polytechnic State University, San Luis Obispo, donde es actualmente profesor de física, y ha sido investigador adjunto en el laboratorio de radioanalítica del departamento de física desde 1980.

El principal interés del profesor Buffa sigue siendo la do­cencia. En Cal Poly, ha impartido cursos que van desde intro­ducción a la física hasta mecánica cuántica, ha desarrollado y

.1 modificado muchos experimentos de laboratorio y ha impar­tido física elemental a maestros locales en un taller patrocinado por la NSF. El doctor Buffa ha combinado la física con su afición al arte y la arquitectura, creando sus pro­pias ilustraciones y dibujos, que utiliza con gran eficacia en sus cursos.

Procuro enseñar a mis estudiantes el crucial papel que la fí.sica desempeña en el entendi­miento de todos los aspectos del mundo que los rodea, trátese de tecnología, biología, astronomía o cualquier otro campo. En ese sentido, hago hincapié en el entendimiento conceptual antes que en la realización de cálculos. Para ello, me apoyo mucho en métodos visuales. Confío en que los dibujos y otras características pedagógicas de este libro ayuden a alcanzar las metas de los pro­fesores que lo usen.

-Tony Buffa

vii

Page 9: Fisica_I

Contenido breve� Prefacio Xl

1 Introducción a la física 1�

2 Cinemática: descripción del movimiento 26�

Apéndices

1� Relaciones matemáticas A-1�

11� Teoría cinética de los gases A-3�

Contenido Prefacio Xl

1� Introducción a la física 1�

Introducción 1�

Parte 1 2�

1.1� Física: clasificación y aplicaciones 2� 1.2 Antecedentes históricos 4� A FONDO: Galileo Galilei y la Torre de Pisa 7� 1.3 El método científico 10�

Parte 2 Sistemas de unidades 12�

A FONDO: Nanotecnología 73� 1.4� Cantidad física 14� 1.5 Clasificación de las cantidades físicas 15� 1.6 Sistema Internacional de Unidades (SI) 14� 1.7 Equivalencias 17� A FONDO: ¿Es importante la conversión de unidades?�

Júzguelo usted 73� 1.8 Factor de conversión 19� 1.9� Conversión de unidades 19� 1.10 Resolución de problemas 19�

111� Datos planetarios A-4�

IV� Lista alfabética de elementos

químicos A-5�

V� Propiedades de isótopos selectos A-S

Respuestas a Ejercicios de refuerzo A-S

Respuestas a Ejercicios impares A-S

2 Cinemática: descripción� del movimiento 26�

2.1� Distancia y rapidez: cantidades escalares 27� 2.2� Desplazamiento unidimensional y velocidad:�

cantidades vectoriales 29� Aprender dibujando: Coordenadas cartesianas�

y desplazamiento unidimensional 29� 2.3� Aceleración 34� Aprender dibujando: Signos de la velocidad y la�

aceleración 36� 2.4 Ecuaciones de cinemática (aceleración constante) 39� 2.5 Caída libre 43� A FONDO: La Torre inclinada de Pisa: un ejercicio en�

estabilidad 45� Repaso del capítulo 50� Ejercicios 51�

Apéndice 1 Relaciones matemáticas A-1� Apéndice TI Teoría cinética de los gases A-3� Apéndice ID Datos planetarios A-4� Apéndice IV Lista alfabética de elementos�

químicos A-S Apéndice V Propiedades de isótopos selectos A-S Respuestas a Ejercicios de refuerzo A-S Respuestas a Ejercicios impares A-ll .

viii

Page 10: Fisica_I

/

Aprender dibujándo Coordenadas cartesianas y desplazamiento Signos de la velocidad y la aceleración 36

unidimensional 29

Aplicaciones [Los ensayos Afondo aparecen en negritas, y "(bio)" indica una aplicación biomédica]

Capítulo 1 Capítulo 2 GaWeo GaWei y la Torre de Pisa 7 GaWeo GaWei y la Torre de Pisa 45 Nanotecnología 13 ¿Es importante la conversión de unidades?

Júzguelo usted 18

Ilustraciones Physlet® Capítulo 1 1-1 Microscopio de barrido por túnel 13

Capítulo 2 2-1 Cálculo de rapidez media 28 2-2 Velocidad y rapidez 30 2-3 Determinación gráfica de velocidad 33 2-4 Determinación gráfica de aceleración 37 2-5 Caída libre 46

Contenido ix

Page 11: Fisica_I

Prefacio Creemos que todo curso de introducción a la física debe tener dos metas básicas: (1) ex­plicar los conceptos fundamentales de la física y (2) capacitar a los estudiantes para usar esos conceptos en la resolución de muy diversos problemas.

Estas metas están relacionadas. Queremos que los estudiantes apliquen conceptos a los problemas que están tratando de resolver. No obstante, es común que inicien el proceso de resolución de problemas buscando una ecuación. Es tentador tratar de sus­tituir valores en ecuaciones antes de visualizar la situación o de considerar los concep­tos de física que podrían usarse para resolver el problema.

Las investigaciones en enseñanza de la física han revelado que un número sor­prendentemente elevado de estudiantes que aprenden a resolver problemas típicos bastante bien como para aprobar los exámenes, no entienden realmente los conceptos más elementales de la física. Simplemente resuelven problemas cuantitativos y obtienen la respuesta correcta, pero no saben por qué es correcta. Además, muchos estudiantes no comprueban sus respuestas numéricas para ver si concuerdan con su entendimiento del concepto físico en cuestión.

Nuestras metas-características de esta edición Nuestras metas para esta edición de este libro son sencillas, pero difíciles de lograr. Pensando en los objetivos del curso, identificamos las áreas que requerían mejoras y nos esforzamos por fortalecer las ventajas del texto.

Primero, pedimos a un colega de confianza que participara en el proyecto. Bo Lou, de la Ferris State University, ha sido parte importante de Física desde la tercera edición. Él escribió el Manual de soluciones para el profesor y la Guía de estudio para el es­tudiante y desempeñó un papel importante como miembro de AZTEC (club de abso­lutamente cero tolerancia de errores). Además, Lou se encargó de actualizar los ejercicios de fin de capítulo. Obtuvo el doctorado en física de materia condensada en la Emory Gniversity.

Creemos, y muchos usuarios comparten nuestra opinión, que las ventajas de este libro de texto son:

Bases conceptuales. Creemos que si los estudiantes entienden a fondo los principios de la física podrán resolver problemas con facilidad. Esta creencia se basa en un enfo­que para desarrollar las destrezas de resolución de problemas que hace hincapié en en­tender los conceptos básicos y no en aprender de memoria las ecuaciones y usarlas de manera mecánica. Continuamente, durante la preparación de este libro, hemos organi­zado discusiones e incorporado herramientas pedagógicas para garantizar que la com­prensión conceptual sea el cimiento para el desarrollo de destrezas prácticas.

Cobertura concisa. Para no perder de vista los conceptos fundamentales, un libro de texto debe hacer hincapié en lo fundamental y reducir al mínimo el material superfluo. En este texto se han evitado temas de interés marginal, así como los que presentan difi­cultades formales o de matemáticas para los estudiantes. Asimismo, no hemos desper­diciado espacio en deducir relaciones cuando ello no arroja más luz sobre el principio en cuestión. Por lo regular, para los estudiantes de un curso como al que \'a dirigido es­te libro, es más importante entender lo que significa una relación y cómo puede usarse, que saber cuáles técnicas matemáticas o analíticas se usaron para deducirla.

xi

Page 12: Fisica_I

A FONDO

Aplicaciones Doppler: células de sangre y gotas de lluvIa

Células de sangre Además de su bien conocido papel de producir la imagen de un feto (Fig. 7.3), el ultrasonido propor­dona una '-ariedad de otros USOS en el campo de la medicina. ~

mo el efecto Doppler, puede detectar y proporcionar infamación sobre objetos en mo\'imienlo, también puede usazse para exami­nar el flujo de la sangre en las mayores arterias y '-enas de los brazos y piernas (Fig. 1). Los. reflectores aquí son células rojas sanguineas. Las pruebas proporcionan a los doctores informa­ción que los ayuda a diagnosticar coágulos. oclusior.es arteriales

HGURA 1 Apftcoción médco del efecto!>oppkn El efecto Doppler se usa para examinar el flujo de la ~ en !.as mayores arterias y \'lonas, aquí la arteria carótida en el Olello. La turbulenda ene! flujo de la sangre puede ser detectado, laque podrfa revelar un estrechamiento de 105 vasos sanguíneos, roágulos o aneurismas (dilatación anormal de Jos \·asos sanguíneos).

Aprender dibujando

Oo.=tO

Rayo paralelo

1 ~ ,I I

ro--- j:-­

Rayo central

,--(

e insulidenda V€flO5a Los procedimientos con ultrasoTúdo ofre­cen una altematin menos inYash"a a otros procedimientos de diagnóstico. como la arteriograHa (imágenes de rayos X de una arteria después de Ja inyección de una tintura).

Otro uso médico del ultI"asonido es el clectrocard.iograma, que es un ex--4IIleIl. del corazÓo'" Sobre un monitor, este pro:edi­miento ultrasónico puede- exhibir los IIlO\·im.ientos de pulsación del corazón, y eJ médico puede ver las <:áIllaI'as del corazón, \rál· \'U.!as y flujo de!a. sangre al entrar}' salir de este órgano.

~1ientras esta¡'Tl06 \'iendo el cuerpo humano y el sonido, he aquí algo para que u;;ted lo ensaye: en una habitadón tranquila, ponga sus pulgares firmemente en sus oídos y escuche. ¿Oye un bajo sonido pulsan!e? ¿Por qué p.a;;a esto? ,Creerla que está es­cuc.hando el5<lnWo, aproximadamen:e a 25 Hz. generado por la rontracri6.... y relajaci6n de las fibras rnus<:ulares en sus manos y brazos? A~'q'.Je en el rango audihle, esos sonidos no se oyen. normalmente, porque el oído humano es relatiyamente Lrtsensi­avo a ~nidos de baja frecuencia. Gotas de lluvia El radar ha sido usado desde los años 40 para s-wninisttar i ..úormación sobre tormentas y otras formas de precipitK:ión. Esta información se obtiene a partir de la intensi· dad de la sefla! reflejada. Tale. radares <:onvendonales pueden también detectar la "'signatura'" rotativa de un tomado, pero s6­lo después de que la tormenta e:>tá bien desarrollada.

Un adelanto <:onsiderab!e en la predicción del tiempo se lo­gró ron el desarrollo de U-"l ilille'ma de radar <¡\le pudo medir el corTimiento de la &e<:uencia Doppler al adición a la magnitud de la señal de eco reflejada por la plffipltaci6n (usualmente go­t:as de llu\ia). El <:orrimiento Doppler está relacionado coo la ve­locic!ad de la precipitad6n soplada por el viento,

Un sistema Dcppler de radar ffig. 2a}puede penetra! una tar­menta)' moni!orearsus\'~Jocidlldesóel\·iento. La di-ección de una l!U\ia soplada por el tiento de una tormenta da un mapa de"'<:am­po" de viento de la regiful afectaÓa. Tales mapas proporcionan

\

Aplicaciones. Tradicionalmente, este libro ha incluido numerosas aplicaciones relacionadas con medicina, ciencias, tecnología, arquitectura y la vida cotidiana, tanto incorporadas al texto principal como en ensayos dentro de recuadros. Aunque esta edición sigue incorporando una ga­ma más amplia de aplicaciones que la mayoría de los textos, también hemos aumentado el nú­mero de aplicaciones biológicas, en reconoci­miento al alto porcentaje de carreras en medicina y similares que incluyen el curso en el que se usa el libro. Como ejemplos de temas que se tratan en los ensayos con orientación biológica pode­mos citar la nanotecnología, la ingravidez y sus efectos sobre el cuerpo humano, la física del ta­ponamiento de los oídos, las resonancias desea­bles e indeseables, el análisis de grasa corporat la cirugía de córneas y la bioingeniería. En la pági­na ix se da una lista completa de las aplicaciones que se tratan.

En esta edición se han mejorado las características

pedagógico, 'iguient"

Recuadros de Aprender dibujando. La vi­sualización es una de las herramientas de resolu­ción de problemas más importantes en física. En muchos casos, si los estudiantes pueden hacer un diagrama de un problema, podrán resolverlo. Las secciones de 11 Aprender dibujando" ofrecen al es­tudiante ayuda específica para elaborar ciertos ti­pos de diagramas y gráficas que ilustran aspectos clave de diversas situaciones físicas.

Objetivos de aprendizaje integrados. Los ob­jetivos de aprendizaje específicos, señalados al prin­cipio de cada sección de cada capítulo, ayudan a los estudiantes a estructurar su lectura y facilitan el re­paso del material.

xii Prefacio

Page 13: Fisica_I

Estrategias y sugerencias para resolver problemas. El tratamiento inicial de la resolu­ La "Ag 1.10 resume l:'Stos paSOS en un diagrama de flujo. Los ejemplos queslguen

ilus-::ran el procedimiento. Los pasos se han numerado para relacionarlos con el prare­dim..icnto.ción de problemas va seguido en todo el libro por

numerosas sugerencias, consejos, advertencias, Ejemplo 1.9 • Colcular elÓTeo de un rectóngufo: practicar

el procedjmJenfo pora resolver problemas atajos y técnicas útiles para resolver tipos especí­Dos estudiMtl$ mi<kn 1" \C\ngltud de]~ady.wcnte;<; de su dormltnrio h..'d:aogular. Uno 00­ficos de problemas. Estas estrategias y sugeren­ ti~ lS ft. 8 pulg,,)' el otro, 4.25 m. ¿Qué árell tiene el dormitorio en ro~ ("".lldrados!

Razonamiento. 1.':;5 Ioogitudcs 5l:' dan en dift'rentes unidad!.'s, ilS qu~ par,) oo.''l!.ner metroscias ayudan a los estudiantes a aplicar principios cuadrados (m x m} es r.ec"t"Sarlo convertir las unidades i.ng~ pies r pulgad¿s,) met:r05.

Sofuclór..generales a contextos específicos y a evitar errores l. Dos l~dos ady~tes de ~ habtuc:-6n dan su longitud y $.U anchur.a, ~ que podemc>s ~bir.y escollos comunes. Dado: Llrgo = 1 = 13 ft. 8 pulg Hl1fftJr. Áre.J {en metros C\wJriXiosJ

.-\..'lmo - =:' '" .¡25 m

Z. H.lg.1 un díagr,1ma qlX" ayu<i(' il 'risualiz.J.r 13 Situ.'ldón (..p¡g. 1.11 l.Ejemplos conceptuales. Esta obra fue ) y 4. En esta ~tuaciÓ11 t.m sencilla. la KUadón ~ es muy conodda. El área.1\ de ununo

rcd.ingu!o esA ... /X6. y se nos-d;l:¡\ tOl.nl'O / romo". J. Har que cambiar unidades:. ConYirtamos primero la medíciÓl'l. de.llargo ro puJg~dns. yde los primeros libros de texto de física en incluir llU'gOl..... pulgad<3S.:lmrlros:

ejemplos de naturaleza conceptuat además de 15ft - Spulg'" (15ft x :: '~. ~) + 8pulg. 18Spulg

los cuantitativos. Nuestros ejemplos conceptua­~<:..: ~les piden a los estudiantes pensar en una situa­ 18Spttlg x·· ~ - ­.¡¡gcm '¡.78 m

... FIGURA 1.10 C»ogmma de Au,'oción física y escoger la predicción correcta de d&l ~nlo $UgO-'ido po!O 0bse1'\'(' lo fkil que- es convertir unidades en el sistema métrko decimal {cent metros a meo­

resc>.'Ver ¡>roblemos Iros}. Realice 13 conyer<;,ión expl dtamente si e<: ~rio, utiliundo e-l factor d.to ('(lfwersi6n

1 m/lOOcm. &. AÍllrare¡¡lice('!..:Jkulo:

entre varias, con base en una comprensión de los principios pertinentes. El análisis que sigue ("Ra­ A =}.J':. U''''' ..1.7$ m X 4.25 m

- 2O.315m~'" :O.3m: (¡o.úG!'{,1/(:ttw,,'rrJ;mJ...r:Jo/lfrNCS;'P"TlllÚ?)zonamiento y respuesta") explica claramente cómo 7. La resp'Jt'Std p.l~ r.u:onO!ble. ?U~ un dormi!OOo de 5 lO. ~ lT'IW"OS!los fJ.ctores de 200 {'S

normal (.mnq~ a lodCl mundo S<!' !e hdCl!'chirosu dom-Jto:io). Supongamos que-, por des.:ui­do,. digi~ 47.S en lug.'lr de .US m 1.;1 cakul<ldO!".il. El rnultM:!o hab!' a sido A = 47.8 m x

identificar la respuesta correcta y por qué las de­4.25 m '" 103 ...:. t:"na habhacioo ron un área dp unos 200 m: tendr .. dime:'l:S.~es de, diga-­más son erróneas. mos. lQm por 20 m (l fuctORS lk 2(0). D3doque no ha~ m\l~¿.omtitoriostan gr"nd~.

la flliIgrUlud deJ t\"!'ouJt.:ldo ct~'r.i ~mos ~i'l.l11a posibilid.:ld de un elTOl'.

Ejercido de re-MrIo. Ltidim~deunlibrodetextosonO..22mX G.2bmX4.0an. ;.Q.JI¿ ..-oI.ur.:cn d~ UN mochila OCUp;1l';¡ el libro? Dé}¡¡ respuesw en mctrosalbkús yen cenoEjemplos resueltos. Hemos tratado de hacer I metros cibicos. (U::> mpue..-tasdr I~ /.., EkrrtCID:' di rrfuer.:.o ~ d:m ¡rf .fillal dd lib1l.l./

las soluciones a los Ejemplos dentro del texto lo En muchos problemas inte-n'iencn funcion~ trigonométriols b..isicas-·. Las más co­

más claras y detalladas posible. El objetivo no só­ munes se dan en la no!.,) a! rr.argen de la pagina 23;.si \1E'CCSita otntS, C-.;lllsuite el a~n.

dice I o las tablas al n?"('f'SO ce la rontr.lporta<ia.

lo es mostrar a los estudiantes qué ecuaciones usar, sino explicar la estrategia que se está si­ Ejemplo 1.10 • Oeterrr-Jnor lo longitud de un lodo de un triónguk):

... AGORA \.11 Un paso tml paa aplicoción de tflgonometric resot.'er probfemas l"n diagramaguiendo y el papel de cada paso dentro del plan .avuda ,) visualiz.ll' la s!t'J,)dó"., \' Se dls..·f'K1 un}.trd n tri.ulgular romo Sol! mue;lr.1 ~ 1:1 '-F"¡g. l.1~ ¿Qui! longild tene diado ~! ...oderlamt'lC'l'. \';''''<et'ltYmPm 1.9. 'f.Je e<otá junto al amino t'mpeodr-<!<i()?general. Se anima a los estudiantes a buscar el

"por qué" de cada paso, junto con el "cómo". Es­ta técnica facilita la aplicación de las técnicas ilus­tradas a otros problemas cuya estructura no es idéntica. Cada Ejemplo resuelto incluye también lo siguiente:

• Paso de razonamiento. Esta sección, que sigue al planteamiento del problema y precede a la solución, sugiere el pensamiento crítico y el análisis que los estudian­tes deben efectuar antes de comenzar a usar ecuaciones.

• Ejercicio de refuerzo. El Ejercicio de refuerzo al final de cada Ejemplo concep­tual y cada Ejemplo resuelto normal hace aún más hincapié en la importancia de entender los conceptos y permite practicar los problemas. (Al final del libro se dan las respuestas a estos Ejercicios de refuerzo.)

Integración de ejercicios conceptuales y cuantitativos. Con el fin de desva­necer la barrera artificial entre las preguntas conceptuales y los problemas cuantitati­vos, no hemos separado estas categorías en los ejercicios de fin de capítulo. Más bien, cada sección inicia con una serie de preguntas de opcióJ;l múltiple y de respuesta corta para repasar el contenido del capítulo, comprobar que el estudiante ha comprendido los conceptos y pedirle que razone con base en los principios. El objetivo es demostrar a los estudiantes que se requiere el mismo tipo de entendimiento conceptuaL indepen­dientemente de si la respuesta deseada implica palabras, ecuaciones o números. Las preguntas conceptuales se marcan con pe para facilitar la asignación de preguntas. Es­te libro incluye al final respuestas cortas a todas las preguntas conceptuales de número impar (así como a todos los problemas cuantitativos impares), para que los estudiantes puedan verificar que han entendido los problemas.

Prefacio xiii

Page 14: Fisica_I

Ejercicios apareados. Casi todas las secciones numeradas incluyen al menos un par de ejercicios

If:¡ ft(Olt~ ~ OUT';'I ~ mI (7.9)¡'-l'-I­ ob=€rm.ior ~!r.(IQIUJrl;.lJ que tratan situaciones similares. El primer proble­1 -.::.

v ma del par se resuelve en la Guía de estudio y manualilcm.d.. ¡.'~=,rnpide::d:,laJualtJ.·

." v'" Topidl": Jtl ~l1lido de soluciones pata el estudiante; el segundo problema, Como 1 - {i'.Id es menor ~lue l,Jo ~ mayor que f~ (.'n esta situación. Por ejemplo, su­p0"ga que la rafidl'Z dé la fu(!ntc ~ un ¿l'Cimo de b rapidez del sonido; esto l'S, ,'~" que explora una situación similar a la presentada en ¡./1O Q ¡:l'., " = ¡;;. Entonces, por 1.\ l"<1.Ja..iÓn 7.9. /" = 't.r.

Similarmente, cuando la fuentl? se aleja del observador (Á' - d - dJ. la fl'l?CUencia el primero, sólo tiene una respuesta corta al final observada esta dada por

del libro, para animar al estudiante a que lo resuel­{i.IOl va por su cuenta.

Aqu ¡" es menor que f.- (¿por qué?) Combitl<ll\do las t:"CUacione. 7.9 f j.10~obti,ml:" una ,,'CUaó6n gl'ner<ll pard kJ trc· Ejercicios adicionales. Cada capítulo incluye

cu('ncia observAda con una ful'ntc m.óvil y un observddor estacionario: una sección suplementaria de Ejercicios adicionales -pala !JII:! fU21!tf aC(1(dlld{~

JUici.l 1111 (lbsem1Jor f'it¡ldQ1l;Jri,,;--- (7JI),-- ­( >1(1Y. tomados de todas las secciones del capítulo, con el ... paro !l1l.'1 fllL71f~ Ql~'iJllck:e rúJro- ~':::V. ,- 1.~ • { m: óbx-m:;!&.- ,"'$1.1<'itmllri¡l- " fin de wrificar que los estudiantes puedan sinteti­

Como podr él usted l'Sp.nar, el efecto Dopple-r también o...-urre con un obst..--r.'.3dOT zar los conceptos. mo\·i&tdOSl' y una tucnk l'SGlciOnariól, .:lunquc L~ta situ..,ción es un poco dif,<'renlc Conforme el obserndor se muen~ na(Í<'l Ja ruel'!te, l.' distancia l;'ntre crest,]s ...ucesivas de ondas es la longitud de onda normal (o A. - ¡'I/J; pero la rapidez de onda medida es Entre las nueyas características de esta edición están diierente. Resp.!'Cto al obse-(\'adOT que se acer'l:a, el sonido de::.de la fuente estacion.\ric. til-'nc una rapidez de ond.l dé " '- t· .....Oo' donde:> ~ L~ la mpid~z ddobscnador ~ t'l'S la rapidez de! sonido.m aire qUieto. (El observador 3ccrcánd~~a la iU(>¡1!(' ~ est.i rno­ las siguientes: \'¡endo en dirección opuesta .\ la de 1..15 onda.. que ~ pTOp3gan ;r por e]l(. e¡H.'uentr,j m¡j ... creslasdc ondas en un tiempo dado.) ~ ­

Ilustraciones Physlet B• Las ilustraciones Physlet]:

son applets Java cortas que ilustran claramente,

Ejemplo Integrado ".4 • Un ctrcuifo simple: uso de kls regios mediante animación, un concepto del texto. Se les de Kirchhcff encuentra en el sitio acompail.ante de este libro y

Dos l\'5~!o~ R¡:- K; ......t.in ~'Or\\'(I.ldO$ m pdrak.-Io. 8ta (omc-inJóÓfl ~ ~'ntooCh :ooec1.ld.. ;m serie J. I.m Imel ~i5tOT R1 Que litml:!!J. tiU\'OT ~c..!l?fl.:i" d" le:. 11\"!5. trnol hatl:'fT.' ~l.) lamo "an seguidas de una serie de preguntas que pidenbit?fl unida P:HiI completar el O;':UIIO. con un el«trodo al principio~' el otTO~1 n..-u.1 de ~t.! red, (a) ¿Qu.... n.'5i.;;J:Of J!(I'\3rJ rn.is.:orrientt:', (ll Rv i2) R; (l /3l .~.? E'<pliquelo. (bl En ("!ó\t' cir· al estudiante razonar críticalllente acerca del con­-e::uit<) ~upon&l que' R .. 6.0 P.. R~ '" 3..0 n. R, .. 10,0 !! Y ~l ',oIlait' lerminal do.> La m:h~"" ~ dto 11.0 \~ Aplique. la" ~lil5d(' Kir(.Í\hof~ pradctt'rminar la ....Cm('!l:l-'C1I .:,:JJ: nsislo;>r ~ ('1 vcll.}· cepto en cuestión. Las ilustraciones Physlet]: se in­~.3 lra",h J .. c.,da re."is~úr

{Q} Razonamiento (ontepti)QI. SE> \í' rn.h. :JcU lo qm.> (~ :-uc('.:henJc,J -.l;' h.l,~ un di'!, lel dican con un icono al margen del texto. grama ut.'m;jtic(') deldn:uiIO(<lFig.4.9) Sío po,jr'iJ FI('nsa,r ini~iaJr:'lCn~ qut.:'d n.··*tMN;,,¡ la menor rr<islcnci,] U1.'\',] la lllo'yor comente:>. PC10 ti'nga cui,iJjo; ~lo es ÓC110 solo;i 1000" Iai rtSl$lon:stst.itll.'ll par.al..lo. EsroJWescl casoaqui I...c:. dos!l'Ststtl~en par.llclo !It.>\·,m cada ~ uno <;óltlllna fr.lcciCin ok 1.1 '{lrr~ntl.' totat c;.in '>fT'~M"::(I j(,¡K) .....d en r;H;'i\eokl~dlar.y. ,",'. ¡>¡¡ la corril"l'li' f("l(dl. Por ('onsi¡;utcn~, Id ff'Sp-":P-"ta Cl"l1n'\':td ~"'IJt. R, Ut."'·i1lóll'"l'U!(\,: mTnell(>. Ejemplos integrados. Para destacar aún más el tb) Rnzonamiento. "inculo entre el entendimiento conceptual y la re­Dnda; XI"" 0.0 n ffallar. la COffll'Tl~C en cula f\SislOr y el \'o!t.3~

R: ... 3.0 n .:! tTa,'t'$ d.-. c-,}d~ l~¡",t(): solución de problemas cuantitativos, hemos desa­R .. lO.OP. ti = 120\' rrollado ejemplos integrados para cada capítulo. Estos

I !a.,,:ce;; (l.:tT)(!f'llt>5 in:t~!l¡t"'~ l.l corrier.l~tot.11 (I¡ \' la", ('(>rrl..>ntt':i t'n (ad,) ww ~""lo,;: r(-;:is;t'C>­

l.) >! 1;1. CmTh~ ..¿,ln h.1)" un3 Nl••n.i.. 1.:t wmer.te dere ""1' "'" "t'fltdo horario ejemplos resuel\-en una situación física de formafi'S ·,m p;¡raiek, ti

(como- S(' 1l1Lh.~l!i1 (>f\ la f¡t:urtl). Aplicando d h'<ll'('m,l el' l.\ umún do: ¡"':rro;-hh<>fi d la ~'nml'r,l

unión cJ en 1.1 fi¡< . .¡ al.l~tnI."... tanto cualitativa como cuantitativa. Los Ejemplos ":.1 = O ! - i - 1:.: Ú

Csando ..1 teoT'l"lT\.t di.' la !fIol.ll.3 C'l'l ~ti(.~ Me.1.nO en la Ftg .¡ <1b. CTU2<1rr05!i1 bat,,·n.; de la integrados ilustran cómo el entendimiento de los con­l('f'11linal nt-stilti\-;¡ a la fX""iti\d \ IUl"gOnx(y,tt'ffiC!$R. \ R; p;H'!""m~'k"tar d uJ..~,l.\l.'\.""\lolci"'n

v= ceptos y los cálculos numéricos van de la mano en~u!t:Il'_h:' {~ttaru:io k~ ~~...". di' lo::; \ úlla,..... c'\:plicit;¡mcntol'l "'" '2V

... ~'.;.. ¡-j.f< , - '-IR,)"'" O la tarea de entender y resolver problemas. Cna ten::erJ iXu.'óór. pued(' ol'>!Cll('~ ...~--!i.:;¡n..il>d !OO!t"fr'.a de \.a ffiJfu rnm<.'~ hill>pR." j;¡. 1rn.'nh:,I!\(t'p:l)quc~\el~end"oJ tr.;,~dl! R:erl \c¿d.: R: rR~ . .J,9,1 brod.:

~T. = II .;.. \'';'' -l:R.¡..,.. ¡-/R, '" O

Ejercicios illtegrados. Al igual que los Ejem­ronit.-ndo ef'I 1<1 NtC'fi3 el \ (',Ila¡e cn \'olts ~ la.., ~1",tt·l'Ki.h <.:':1 clt.'71> ~ tI.'>liree!.t...,do l..........., t"CU;)·lo' <ioocs, len('f.'1,C"o ('1\tOllC~

... F~GURA 4..9 E.sbol.o de 1-1 - ,; plos integrados del capítulo, los Ejercicios integra­ó¡og;omas ée circuitos U$O"flOo !Os tegios de Kirchhoft la) Eo<bmo 12 - 61 - IOi '" O dos piden a los estudiantes resolver un problemadd dldgram¡¡ del cin:uiw d~' 1.1 12 - -!: - HlI = 1) ¡:i.'ljde;;cripdim ¡,>,,(elt.l (O"n el E",e-mplo cuantitati,'amente y también contestar una pregun­

ta conceptual relacionada con el ejercicio. Al con­testar ambas partes, los estudiantes pueden "er si su respuesta numérica coincide con su entendimiento conceptual.

Icono de referencia a figuras. En esta edición, hemos incluido un triángulo jun­to a cada referencia a una figura dentro del texto y junto a cada pie de figura. Estas "flechas" indican al estudiante dónde está la figura correspondiente y se localizan fá­cilmente cuando el estudiante ,'uelye al enunciado del texto.

Repaso del capítulo. La sección de Conceptos y ecuaciones importantes se ha in­tegrado a la nuen sección de Repaso de cada capítulo. Los Conceptos clave están en negritas y se definen con palabras, además de símbolos. Este nuevo formato sirve co­mo referencia rápida al estudiar.

xiv Prefacio

Page 15: Fisica_I

Hemos seguido cuidando la exactitud a través del Club de absolutamente cero tolerancia de errores (AZTEC). Este enfoque de equipo para verificar la exactitud fun­cionó muy bien en ediciones anteriores, así que lo hemos aplicado otra vez. Bo Lou de la Ferris State University, autor de nuestro Manual de soluciones para el profesor, en­cabezó el equipo AZTEC y contó con el apoyo de los autores del texto y de otros dos correctores, Bill McCorkle de la West Liberty State University y Dave Curott de la Uni­versity of North Alabama. Cada miembro del equipo resolvió, de manera individual e independiente, todos los ejercicios de fin de capítulo. Luego se reunieron los resul­tados y cualesquier discrepancias se resolvieron mediante una discusión en equipo. Todos los datos de los capítulos, así como las respuestas al final del libro, se verificaron tanto en la primera prueba de impresión como en la segunda. Además, otros profeso­res de física: Xiaochun He de la Georgia State University, Jerry Shi de Pasadena City College, John Walkup de la California Polytechnic State University en San Luis Obis­po, William Dabby del Edison Cornmunity College y Donald Elliott de Carroll College, leyeron minuciosamente las páginas, en busca de errores en el texto del capítulo, en los ejemplos resueltos y en las ilustraciones. Aunque con toda seguridad no es humana­mente posible producir un libro de texto de física carente totalmente de errores, ésa fue nuestra meta; nos esforzamos mucho por eliminar todos los errores en la medida de lo posible.

Sitio Web Companion. Nuestro sitio Web (http://wvvvv.pearsoneducacion.net/ wilson), que contiene aportaciones de destacados investigadores sobre docencia en fí­sica, proporciona a los estudiantes diversas exploraciones interactivas de los temas de cada capítulo que se ajustan fácilmente a diferencias en los estilos de aprendizaje. En­tre las herramientas que el sitio ofrece a los estudiantes están las Ilustraciones Physlet]'; de Steve Mellema y Chuck Niederriter (Gustavus Adolphus College); "calentamien­tos", acertijos y aplicaciones"¿Para qué sirve la física?" de Gregor Novak y Andy Gavrin (Indiana University-Purdue University, Indianapolis); galardonados problemas Phys­let~ basados en Java creados por Wolfgang Christian (Davidson College). Problemas de Práctica numéricos generados algorítmicamente, Preguntas de práctica de opción múltiple destinos en línea creados por Carl Adler (East Carolina University); Ejercicios de clasificación (Ranking Tasks) editados por Tom O'Kuma (Lee College), David Malo­ney (Indiana University-Purdue Universit)', Fort Wayne) y Curtis Hieggelke (Joliet Ju­nior College). Objetivos del capítulo y Soluciones a ejercicios selectos por Bo Lou (Ferris State University); y Preguntas MCAT por Glen Terrell (University of Texas at Arlington) y del MCAT Supercourse de ARCO. Utilizando el módulo Preferencias en la página inicial del sitio o la herramienta de la parte "Results reporter" de cada módu­lo, los estudiantes pueden, si su profesor se los pide, pedir que los resultados de su tra­bajo en el sitio Web acompañante se envíen por correo electrónico al profesor o a su asistente. Entre las herramientas para el profesor están una función de calificación en línea y el Syllabus Manager. En las páginas xix-xx se da más información acerca de los módulos de este sitio.

Prefacio XV

Page 16: Fisica_I

Agradecimientos Queremos reconocer la generosa ayuda que recibimos de muchas personas durante la preparación de esta edición. En primer lugar, agradecemos sinceramente a Bo Lou de la Ferris State University sus imprescindibles aportaciones a los capítulos sobre óptica. Apreciamos enormemente su meticulosa ayuda en la verificación de las solu­ciones y las respuestas a los problemas, así como en la preparación del Manual de solucio­nes para el profesor, la clave de respuestas al final del libro y la Guía de estudio y manual de soluciones para el estudiante. También agradecemos a Dave Curott de la University of North Alabama la preparación del Archivo de reactivos de examen y su participación como verificador de todas las soluciones a los ejercicios de fin de capítulo.

En realidad, todos los miembros de AZTEe -Bo Lou, Dave Curott, Bill McCorkle (West Liberty State University)- así como los revisores de la primera y la segunda pruebas de impresión -William Dabby (Edison Community College), Donald Elliott (Carroll College), Xiaochun He (Georgia State University), Jerry Shi (Pasadena City College), John Walkup (California Polytechnic State University en San Luis Obispo)­merecen más que una expresión especial de agradecimiento por su incansable, oportu­na y en extremo minuciosa revisión de todos los materiales del libro para asegurar su corrección científica.

Docenas de colegas más, mencionados en la sección que sigue, nos ayudaron a re­visar la edición anterior y a planear ésta, y también revisaron el manuscrito durante su desarrollo. Estamos en deuda con ellos, pues sus consideradas y constructivas su­gerencias beneficiaron enormemente al libro.

El personal editorial de Prentice Hall nos ha apoyado mucho. En primer lugar, queremos extender nuestro sentido agradecimiento a nuestra antigua editora, Alison Reeves, por iniciar esta revisión y dirigirla con gran atino. Gracias también a Mary Catherine Hager, editora de desarrollo, Patrick Burt, gerente de proyecto del libro y Beth Sturla Sweeten, editora ejecutiva adjunta, quienes mantuvieron el ímpetu de este complejo proyecto, mientras que el diseñador Jonathan Boylan se aseguró de que la presentación física final fuera visualmente atractiva y fácil de usar. También damos gracias a Mark Pfaltzgraff, gerente ejecutivo de marketing; a Christian Botting, editor adjunto, por su extensa labor con los suplementos, el programa de medios y el progra­ma de revisión; a Erik Fahlgren, editor de adquisiciones y Eileen ~ee, asistente edito­rial, por su ayuda para coordinar todas estas facetas; y a John Chalice, editor en jefe, por su apoyo y empuje.

Además, yo (Tony Buffa), quiero expresar una vez más mi agradecimiento al pro­fesor Jerry Wilson, mi coautor, por su animosa disposición a ayudar y su actitud profe­sional frente al pesado trabajo de esta edición. También estoy en deuda con el profesor Bo Lou, quien contribuyó con muchas ideas y comentarios valiosos que nos fueron de gran ayuda. Como siempre, varios colegas míos de Cal Poly cedieron su tiempo para participar en fructíferas discusiones. Entre ellos están los profesores Joseph Boone, Ronald Brown, Theodore Foster, Richard Frankel y John Walkup. rvli familia -Connie, mi esposa, y mis hijas Jeanne y Julie- fue, como siempre, una fuente continua e in­dispensable de apoyo. También quiero reconocer el apoyo de mi padre, Anthony Buffa, Sr., y de mi tía, Dorothy Abbott. Por último, agradezco a los estudiantes de mis clases que han aportado excelentes ideas durante años recientes.

Por último, los dos pedimos encarecidamente a quienquiera que use este libro -estudiante o profesor- que nos comunique cualquier sugerencia que tenga para mejorarlo. Esperamos con ansia sus comunicaciones.

-Jerry D. Wilson [email protected]

-Anthony J. Buffa [email protected]

xvi Prefacio

Page 17: Fisica_I

Revisores de esta edición Alice Hawthorne Allen Virginia Tech

Anand Batra Howard llniversity

Michael Berger Indiana llniversity

James Borgardt Juniata College

Jeffrey Braun Universih) of Evmlsville

Michael Browne llniversity of Idaho

Mike Broyles Collin County Comnzunih) College

James Canoll Eastern Michigan State llniversih)

Robert Coakley llniversih) of Southern Maine

Sergio Conetti llniversity of Virginia, Charlottesville

Revisores de las ediciones anteriores William Achor Western Maryland College

Arthur Alt College of Great Falis

Zaven Altounian McGillllniversity

Frederick Anderson llniversity of Vermont

Charles Bacon Ferris State College

Ali Badakhshan llniversity of Northem Io¡m

William Benes Wayne State llniversity

Hugo Borja Macomb COillm¡ll1ity College

Bennet Brabson Indiana llllÍversity

Michael Browne llniversity of IdallO

David Bushnell Northem Illinois ll¡¡h'ersity

Lyle Campbell Oklahoma Christian llnh'"rsity

Aaron Chesir Lucent Teclllloiogies

William Dabbv Edison COIllIll¡¡llity College

Puma Das Pllrdlle lllúversity

Donald Elliott Canoll College

Lewis Ford Texas A&M llniuersity

Gary Hastings Georgia State Uniz'ersih)

Xiaochun He Georgia State lllúu"rsity

Andy Hollerman llnil'ersity of LOllisimza, Layfayette

Randall Jones L0/01a lllli¡;ersi~/

KeYÍn Lee llllil'ersity of Nebraskn

Paul Lee California State llnil'ersity, J. -ortl!ridge

William McCorkle West Liberty State ll¡úz'ersity

Lowell Christensen American River College

Philip A. Chute ll¡¡iversity of WiSCOl1sin-Eau Claire

Lawrence Coleman llnii'ersity of Cal! omia-Dm'is

Lattie F. Collins East Te¡¡¡¡essee State ll¡¡ipersit:!

James Cook Middle Te¡¡¡¡essee State llni¡'ersih)

Daúd :,,1. Cardes Belleuille Area COlllllnlllity College

James R. Crawford. Southwest Texas State llnipersity

J. P. Daúdson Unipersity of Km~sas

Donald Dav Montgollle¡y College

Richard Delane\' College of Aeronauti[5

James Ellingson College of DllPage

Arnold Felclman llniZ'ersity of Hn¡paii

John Flaherty l'uba Colleg"

Rober J. Fole\' llniZ'ersity of Wisconsin-Stollt

Paul Monis Abilene Christian llniversih)

K. W. ichalson Central Alabmnq Community College

Erín O'Connor AlIan Hancock College

Anthony Pitucco Glf?lldale Commanity College

William Pollard Valdosta State llni¡'ersity

David Rafaelle G/endale Cam1l111ll ity College

Robert Ross lllliuersity of Detroit-MercJ)

Craig Rottrnan North Dakota State llniZ'ersity

Om Rustgi Bulfalo State College

Cínd\' Schwarz Vassa'r College

Bartlett Sheínberg HouStOll Commllnity College

Jenv Shi Pasadena City College

Donald Foster Wiclzita State UniZ'ersity

Donald R. Franceschetti Memphis State llni¡'ersih)

Frank Gae\' !TT Tec1mical Instítute-Ft. [allderda/e

Rex Gand\' Allblll'll UniZ'ersih)

Simon George Califomia Sta te-Long Beach

Barry Gilbert RJlOde Islmld College

Richard Grahm Ricks College

TomJ. Gray llnipersity of 1 ebraska

Douglas Al Harríngton I\ortheastenz State llniversi~1

J. Erik Hendrickson Ulliuersity of WiSCOilsiií-Eall Claire

Al Hilgendorf llnÍl'ersity of Wisconsin-Stollt

Joseph :VI. Hoffman Frostbllrg State lllliZ'ersity

Jacob VI'. Huang Towsoll lllliZ'ersih)

Ornar A.hmad Karim llllil'ersity of'Vorth Ca1'OIÍll(l-I'\'ilmingtoll

Christopher Sirola Tri-COllnh) Technical College

Gene Skluzacek St Petersbtag College

Ross Spencer Brigham YOll11g llniuersity

yIark Sprague East CarolÍlza Ul1iuersity

GabriellJmeral1 Florida Conllillll1ity College-Jacksolll'il/e

Lorín Vant-Hull lllliversity ofHOllStOil

Karl Vogler Northem Kelltucky llniversity

John Walkup Cal!fomia Polytechl1ic State llniZ'ersity

Lanv Weinsteín Old Dominion llniuersity

Jefferv Wraaa . 00

College of Charles ton

Rob ylie Cad Albert State lllliversity

S. D. Ka\'ianí El Camino College

Victor Keh !TT Technical Institute-Nonua/!c, California

JohnKennv Bradley llniversity

James Kettler Ohio UíliZ'ersity, Eastern Campus

Dana Klínck Hillsborough Commlmih) College

Chantana Lane Un ivers ity of Tenllessee-Chattallooga

Phillip Laroe Canoll College

Rubín Laudan Ol'egoil State ll¡zil'ersity

Bruce A. Lavton Mississippi Glllf Coast COl1lm¡mity College

R. Gary Layton :Vorthel'1l Arizona lllliversity

Federíc Liebrand Walla Walla College

Y1ark Líndsa\' llniueTsi~/ of LOllisville

Bryan Long Columbia State Commzmity College

Prefacio xvii

Page 18: Fisica_I

Michael LoPresto Henry Ford Community College

Dan MacIsaac Northern Arizona University

Robert March University of Wisconsin

Trecia Markes Uniuersity of Nebraska-Kearney

Aaron McAlexander Central Piedmont Conl17111nity College

Jolm D. McCullen Uni¡'ersity ofArizona

Michael y1cGie California Stnte University-Clzico

Gary Motta Lassen College

J. Ronald Mowrey Hnrrisburg Aren Commlmity College

Gerhard Muller Unic'eTsity ofRhode lsland

R. Daryl Pedigo Austin Com111unity College

1. A K. Pillai Universihj ofWisconsin-Ln Crosse

Darden Powers Baylor Universihj

Donald S. Presel University of Mnssnclzllsetts-DartlllOutlz

E. W. Prohofsky Purdue University

Dan R. Quisenberry MerceT University

VI. Ste\'e Quon Ventura Col!ege

George Rainey Califonzia Sta!e Polytecll1lic University

MichaelRam SUNY-Bll/falo

William Riley Ohio Stnte Unitwsity

William Rolnick Wayne Stnte Ullit'ersity

Gerald Royce Mary Washington College

Ro)' Rubins Ulliversity o.f Texas, Arlington

SidRudolph University of Utalz

Anne Schmiedekamp Penllsylvmzia State University-Ogoiltz

Ray Sears University of~ortlz Texas

Y1ark Semon Bntes College

Peter Shull Okllllzollla State Uniuersihj

Thomas Sills Wilbllr Wriglzt Co/lege

Larrv Silva Ap'?alaclziml State Ulliversity

MichaelSimon HOllsntoilic COllllllliizi!y Tecllllicnl Coilege

Soren P. Sorensen University of Tenllessee-K,lOxville

Dennis W. Suchecki San Diego Mesa College

Frederick J. Thomas Sinclair C01l1111wzity College

Jacqueline Thomton St. Petersburg Junior College

Anthony Trippe !TT Teclmical lllstitllte-Sml Diego

Pieter B. Visscher Ulliversity ofAlaballla

Arthur J. Ward Xaslz¡·ille State Tec}¡nical lnstitute

Jolm C. Wells Tennessee Teclmical University

Arthur Wiggins Onklmld Comnnmitj College

Ke\'in Williams !TT Technicai lnstitute-Earth City

Linda Winkler Appalaclzian State Uniuersity

Jolm Zelinsky SOllthem Illinois University

Dean Zollman Kansas State Uni:·ersihj

xviii Prefacio

Page 19: Fisica_I

Exploraciones multimedia de la física

Sitio Web acompañante www.pearsonedlatino.com/wilson Este sitio Web de introducción a la física, específico para este libro, proporciona a los estudiantes y profesores abundantes e innovadores materiales en linea para complementar sus cursos.

,. TO ltnlo t:ht l1'luimum ran¡t IIHoj4;ct!lo hu lo bt Iwn~h.o al 4$ cl.gr~u ifth' landing ~t and lhoe la.uncn spct vt et the nma lulgM (ntillecling alf retl$tancettfKts.)

Explalo in af.w sent'nctl how lhe "lltion b~twH" tl'\tl 'iotrt!cal,nd th. h'OrtlontaJ com~on.ntt ofttl. ínlth,! vefoo:Jty al'fem ttle proJectlle rango

.J

2. r Ontta MQl;In tM acul'I'1t!Or'Idut ~Oar1vityl~ tboi,f.:one '1~tt1ltO" elrth.lf' go!'!'tronthe Moon lmpa..1:ed the llame lnitial velocity\1) tht baU as she does 0f'I tht ~rth, now much futhtrwou1d tl":e ball oo?

Ilustraciones Physlef® por Steve Mellema y Chuck Niederritter (Gustavus Adolphus College)

"Calentamientos" y acertijos por Gregor Novak y Andrew Gavrin (Indiana University­Purdue University, Indianapolis) Las preguntas de calentamiento y de acertijo son preguntas de respuesta corta, relacionadas con el mundo real y basadas en importantes conceptos de los capítulos del libro. Ambos ti­pos de preguntas llaman la atención de los estudiantes, a me­nudo se refieren a sucesos de actualidad v son buenos temas para iniciar discusiones. Los "Calentamientos" están diseña­dos para introducir un tema, mientras que los Acertijos son más complejos y muchas veces requieren integrar más de un concepto. Los profesores pueden asignar preguntas de calen­tamiento después de que los estudiantes lean el capítulo pero antes de la clase en que se trata el tema, y asignar acertijos co­mo tarea para después de la clase.

Y,¡u"Y~"..q7

Las Ilustraciones Physlet®, que se presentan por primera vez en esta edición, son pequeñas applets Ja\'a interactivas que ilustran --~•.?'.''''--­

;''''!fl~ f(~i~_H ~

claramente, mediante animación, un concepto del texto. Se les en­,~IT'"'

cuentra en el sitio acompañante de este libro y por lo regular van C,-,cfC<ltt"1tl:b" I

seguidas de una serie de preguntas que piden al estudiante ra­zonar críticamente acerca del concepto en cuestión. Las ilustra­ciones Physlet'& se indican con este icono al margen del texto:

Ble fdit :0ew ~vorítes Tonls tielp

Force and Motiotl

4pOlic3tlonS

~--~--------------

There is a Chinesa slory 01 a legendary govemmenl ' olñclal Wan-Hoo who equipped a chair wilh 47 large rockels, hoping iI would lake him lo lhe moon. His 47 assistanls igniled Ihe rockets wilh lorches. An enormous explosion Followed. When Ihe smoke cleared Wan·Hoo and Ihe chair were gone never to be seen again.

Tne ~n!:lem Ch¡nesg, 3nd the anciEnl Gr~;ks, Vle¡E f;m.liar '....nI; the ;lc!iQl'-res,,¡o"'\ O!H'fC r'l<, now cal· so N~on'$ Tn,re Law Arl oOle~

~ slJchasarockslcanpushnselffaNl'ard,again;llhelerl!aofÚle? malenal!h-a1!tejects. Weh~'9aIlÍ'la;jfi.nle':tll'lgg~aiff,JlE,¡jb;I!iJ'ons,wtllchprOp91

lh>:ffisel't'?sagainstlheescapmgall Amo¡eelaboraieversionoflhls idea 15 a socia bottle ro~kEt fJ'ed Wi"lh compr€ssed alr Cneck oU! lhe !lel'l]:. 'QCKe-i ir;;!ruMI~M ::.¡ssen¡ed ay no lBS an 3ulhorny on focke¡ flJghllhanN.AS.A_

Unlike airplanes Iha! need air for líft and fel oxygen. roek€1s (01 tirecracke¡s) orry lhe,r CNm oxrgen and IhSlr fuet. They are comple!ely setf..suffielell! ar,o can propellhemse/ves ¡hlough vacuum Climese rockelry was supPOr1ed by anolhu oflh!!Ir clE"Yer trl',ernlOns S::veral huna'red years B.C. (ha ChinEse

.;:

Aplicaciones por Gregor Novak y Andrew Gavrin (Indiana University-Pur­due University, Indianapolis) Los módulos de Aplicaciones contestan la pregunta"¿para qué sirve la física?", relacionando conceptos de física con fenómenos del mundo real y nuevos avances en ciencia y tec­nología. Estos ensayos ilustrados incluyen ,-inculos Web para sitios relacionados, uno por capítulo. Cada ensayo va seguido de preguntas de respuesta corta/ensayo, que los profesores pueden dejar de tarea.

Prefacio xix

Page 20: Fisica_I

Problemas Physlef® por Wolfgang Christian (Davidson College)� Los Problemas Physlet® son problemas multimedia basados� en las galardonadas applets Java de Wolfgang Christian pa­�ra física, llamadas Physlets®. Con estos problemas, los estu­�diantes usan elementos multimedia para observar, aplicar� conceptos apropiados de física y efectuar mediciones de� parámetros que consideren importantes. No se dan cifras,� así que el estudiante se ve obligado a considerar el proble­�ma cualitativamente en vez de limitarse a sustituir valores� en fórmulas.�

.... t<ll """" ,..... , .... t*

~.J!.. ~ ~":'..L ;;;; ~ ! .~~~_

~t.""

r~~~~~"'!>~,"ll'<!:».~::t~.;¡~~;j·tr~..-~t'"~.u-K ~MIC'1t4't<t'~':;"~~:~~! ~,I. ~.~ J .u. .....~If .. "P'""J .. D.~ >;'j

Preguntas de práctica� por Cad Adler (East Carolina University)� Hay dos módulos con 10 a 15 Preguntas de práctica de� opción múltiple para repasar cada capítulo.�

Guía de estudio MCAT por CIen Terrell (University of Texas at Arlington) y Supercurso MCAT de ARCO Para todos los capítulos pertinentes, el módulo de Guía de estudio MCAT ofrece a los estudiantes un promedio de 25 preguntas de opción múltiple sobre los temas y conceptos cubiertos en el examen MCAT. Al igual que en todos los módulos de opción múltiple, la computadora califica auto­máticamente las respuestas del estudiante y proporciona referencias a las secciones correspondientes del texto.

Problemas de práctica por Cad Adler (East Carolina University)� Diez Problemas de práctica numéricos por capítulo, genera­�dos algorítmicamente, permiten a los estudiantes obtener� múltiples iteraciones de cada conjunto de problemas, para� practicar.�

~~t-r__ ~U*

~ ..-,2 ~ 2. ~..!- ~ i' .; ! ~ _:­

.\pL.nctrr.·.~"~ __I"~~''Ul'in''''''.'fl-6IPf111co.t .. dHQc;1U$l~14Ib'('f!l,I: .._istht~or ¡" ..,«knOon;'¡

Destinos Los Destinos son vínculos con sitios Web relacionados con cada capítulo, que tratan el tema del capítulo o presentan aplicaciones relacionadas con él.

Soluciones a ejercicios selectos por Bo Lou (Ferris State University)� Soluciones a seis ejercicios selectos por capítulo, tomados de� la Guía de estudio y manual de soluciones para el estudiante de Bo� Lou.�

Syllabus Manager El Syllabus Manager de Wilson/Buffa ofrece a los profeso­res un sencillo proceso para crear y modificar, paso por pa­so, un programa de estudio con vínculos directos al sitio Web acompañante ya otro material en línea. Con este pro­grama, los profesores pueden asignar tareas y en iar anun­cios al grupo con un solo clie. El programa se alberga en los servidores de Prentice Hall, así que puede actualizarse des­de cualquier computadora con acceso a Internet.

Syllabus Manager

Course lnformation

Ejercicios de clasificación (Ranking Tasks) editados por Tom O'Kuma (Lee College), David Maloney (Indiana University-Purdue University, Fort Wayne) y Curtis Hieggelke (Joliet Junior College)

Los Ejercicios de clasificación, disponibles para casi to­dos los capítulos del texto como archivos PDF en el sitio Web Wilson/Buffa, son ejercicios conceptuales que piden a los estudiantes clasificar varias situaciones o variaciones de una situación. Estos juicios comparativos ayudan a los estudiantes a razonar situaciones físicas y a menudo permi­ten entender mejor las relaciones entre diversos conceptos ~' principios.

Clasificación en línea Los puntajes obtenidos en todas las preguntas y problemas objetivos, así como las respuestas a las preguntas de ensa­yo, se pueden em-iar por correo electrónico al profesor.

xx Prefacio

Page 21: Fisica_I

Introducción a la física ULO

INTRODUCCiÓN

El propósito de escribir el presente libro es el de des­pertar curiosidad en el estudiante que lo lea; que considere a la física como una ciencia de la naturale­

za que se basa en la experimentación, en la reflexión y en la imaginación creadora; ya que su desarrollo es una activi­dad humana en la que han participado muchos hombres

Objetivos del capítulo

Que el alumno

• Identifique los conceptos básicos para el estudio de la física, así como el avance histórico de esta ciencia.

• Conozca las herramientas para su eshldio y adquiera la capacidad de usarlas apoyándose en los conceptos para resolver diferentes problemas.

Objetivos específicos

Al término del presente capítulo, el alumno deberá cumplir con las siguientes metas:

• Identificar los modelos que han sido propuestos para la comprensión del Universo.

• Manejar elementos de juicio que le permitan definir la física.

• Identificar algunas aplicaciones de principios físicos en ciertos fenómenos que le rodean.

• Comprender la importancia de seguir un método para investigar sobre las causas y efectos que acompañan a un fenómeno físico. .

• Manejar con facilidad los conceptos de cantidad física, unidad y medida.

• Identificar con facilidad las unidades correspondientes al Sistema Internacional de Cnidades.

• Realizará conversiones de unidades en el Sistema Internaciona1.

de todas las épocas de diferentes países; por lo que su evolución se ha visto influenciada por las circunstancias históricas; a su vez, ésta ha contribuido a cambiar la con­cepción que el hombre tiene del mundo y de sí mismo. Ha desempeñado un papel fundamental en el progreso tecno­lógico, y es una parte importantísima de la cultura, por eso consideramos su estudio como una de las herramientas que ayudarán al alunmo a comprender su entorno y el de­sarrollo de éste.

1

Page 22: Fisica_I

la parte 1 (La física: historia y aplicaciones), comienza con una bre,'e clasificación de la física para que el estudiante tenga un panorama general. Posteriormente, se presentan los modelos más predominantes que han sen'ido para el desarrollo de la materia. Ense­guida, se dan los lineamientos generales del método científico experimental, el cual se re­forzará con las prácticas correspondientes en el laboratorio. Por último, se presenta una serie de herramientas que vistas desde la perspectiva de la física como ciencia, se utilizan para comprender las relaciones funcionales entre sus diferentes leyes y conceptos.

La parte 2 (Sistemas de unidades), analiza la materia, teniendo en cuenta que la físi­ca se aprende y se aprecia estudiando, comentando y discutiendo las preguntas y pro­blemas que se presentan al final de cada unidad, es recomendable que el estudiante participe en discusiones y comentarios con el profesor y sus compañeros.

Este libro, como todos, no es definitivo, siempre será posible mejorarlo. Pero para ello se necesita la cooperación de profesores y estudiantes, quienes, a través de su lec­tura, pueden decidir si los contenidos son, o no, adecuados para enfrentar los temas de estudio. Siempre serán bienvenidos los comentarios y criticas.

Comité de Física.

PAR T E 1

La física: historia y aplicaciones

111 Fí&ica: clasificación y aplicaciones

Vivimos, la mayoría de nosotros, inmersos en nuestro ambiente diario sin comprender casi nada del mundo que nos rodea ~- dedicamos poco tiempo a pensar en ello, sin em­bargo, han existido hombres con la suficiente curiosidad y empeño que se han consagra­do a estudiar, a desarrollar métodos y herramientas que le sin'an para explicar y tener una mejor comprensión de éste y poder predecir sistemáticamente el movimiento de la materia y la energía, así como conocer de qué está formada la materia y el Universo en el que \'i,'e, lo cual constituye la base fundamental del progreso del ser humano en los ámbitos científico y tecnológico, que se proyectan en una vida con más comodidades.

2 CAPíTULO 1 Introducción a la física

Page 23: Fisica_I

La física es una de las ciencias naturales que estudia la materia, la energía y las re­laciones entre ambas. Su relación con todas las demás ciencias es vital ("'figura 1.1), por ejemplo con la biología, la geología, la química y la astronomía, las cuales utilizan las teorías y las leyes físicas.

La física clásica es una expresión que se refiere a los estudios realizados hasta fina­les del siglo X1X, acerca de la mecánica, la luz, el calor, el sonido, la electricidad y el magnetismo. Los conocimíentos adquiridos en esta rama se basaron en la observación a través de los sentidos y en mediciones directas.

A principios del siglo xx, se desarrolla la teoría de la mecánica cuántica, la cual es­tudia el comportamiento de las partículas y los sistemas microscópicos como las mo­léculas, los átomos y sus componentes. Aquí es cuando el hombre se ve imposibilitado para efectuar mediciones de manera directa y precisa, por lo que las realiza de manera indirecta y probabilística. En esta mísma época, tuvo sus inicios la física relativista, que estudia el movimiento de objetos a velocidades cercanas a las de la luz, y el efecto de estas velocidades sobre la masa, la longitud, el tiempo y la energía. Al igual que en la mecánica cuántica, en la física relativista no se pueden efectuar mediciones directas. Al principio, fue considerada como una propuesta descabellada, pero con el tiempo se han demostrado, a través de experimentos algunas de sus consecuencias, por ejemplo la relación energía-masa dada por E = m¿, en donde E representa la energía, de un ob­jeto de cierta masa (m) y e, la velocidad de la luz.

Se considera que el inicio de la física moderna tiene lugar a partir de la teoría de la relatividad y de la teoría cuántica en la descripción de sistemas microscópicos como los átomos y las moléculas; además de una comprensión detallada de los sólidos, líqui­dos y gases. Con base en estas ideas, se tiene que la Física se puede clasificar en las si­guientes ramas:

Mecánica

Termodinámica

CLÁSICA Óptica

Acústica

Electromagnetismo

FíSICA Relatividad

Mecánica Cuántica

Física Atómica

MODERNA Física Nuclear

Física del estado sólido

Física del plasma

Física de partículas elementales ~ FIGURA 1.1 Ciencias relacionadas con la física.

1.1 Física: clasificación y aplicaciones 3

Page 24: Fisica_I

~ FIGURA 1.2

Con excepción de los fenómenos en el mW1do microscópico y el movimiento de partículas a velocidades próximas a la de la luz, la física clásica describe adecuada­mente el resto de nuestro mundo físico. Los fenómenos que se estudian en las diferen­tes ramas de la física se relacionan entre sí, mediante un pequeño número de principios básicos (leyes generales), integrándola de manera coherente y no como un estudio de hechos aislados. Estos principios básicos pueden ser abordados en el estudio del movi­miento de los cuerpos y prolongarse después a las demás áreas de la materia.

La física ha realizado aportaciones a la tecnología o ciencia aplicada, la cual ofre­ce métodos de solución para los problemas prácticos de nuestro entorno; dichas apor­taciones han sido de gran utilidad para el desarrollo de la humanidad. Por ejemplo, los descubrimientos en el campo de la electricidad produjeron una gran revolución en la transportación terrestre, aérea y marítima; al estudiarse y entenderse mejor la elec­tricidad y el magnetismo se llegó a la industrialización de la energía eléctrica en gran escala, así como a las comwucaciones telegráficas, telefónicas, de radio y de televisión (.figura 1.2).

Para una mejor comprensión de lo anteriormente expuesto, vamos a considerar al­gunas ideas que han predominado en el desarrollo histórico de la física, su aportación a la ciencia así como su método de investigación.

Actividad sugerida

Investiga la relación de la física con otras ciencias y las aportaciones de ésta en el desarrollo tecnológico.

ID Antecedentes históricos En el desarrollo histórico de la física se contemplan tres ideas primordiales, las cuales han sen'ido al hombre para conocer su entorno. Cada una de ellas predominó en cier­ta época, hasta que fueron sustituidas por otras, al no brindar aquéllas una explicación adecuada y precisa del mundo que nos rodea. Es así como la humanidad evoluciona en la satisfacción de sus necesidades, mediante el desarrollo del llamado conocimiento científico.

1.2.1 El modelo aristotélico Aristóteles (384-322 a.e.) (~figura 1.3) trató de dar alguna explicación a cada uno de los aspectos relevantes de la naturaleza y de la vida. Para ello, recopiló y ordenó toda

4 CAPíTULO 1 Introducción a la física

Page 25: Fisica_I

la información necesaria disponible, por lo que es considerado el primero de los enci­clopedistas. Su obra es abundante, pero en nuestro caso nos concretaremos en lo que con frecuencia se acostumbra designar como la "física aristotélica"; en ella estudió es­tos textos utilizados de forma básica por los intelectuales escolásticos medievales. Uno de los trabajos en las universidades fue el de hacer inteligibles estos escritos y aclarar el pensamiento aristotélico.

La materia, su forma, su movimiento y el espacio que ocupa. Sus aportaciones fue­ron más bien filosóficas vistas con el rigor que actualmente se conocen. Los escritos aristotélicos fueron difundidos por los árabes en Europa occidental y traducidos final­mente al latín.

En el pensamiento de Aristóteles se observa que el propósito de su indagación era encontrar el orden de todas las cosas, formulando un gran Universo lógico, en donde cada cosa "conoce" su lugar y tiende a permanecer ahí. Este tipo de proposiciones las formuló con base en sus observaciones, las cuales no verificó experimentalmente, ya que el trabajo manual era considerado como algo indigno y sólo para esclavos. En cambio, la observación sí era aceptada y por eso se realizaron grandes avances en la astronomía (al predecir eclipses, elaborar calendarios, etcétera).

Aristóteles planteó su concepción del mundo, la cual consta de cuatro elementos .... FIGURA 1.3 Aristóteles. superpuestos, dentro de la esfera subhmar, a saber: tierra, agua, aire y fuego, y agrega un quinto elemento, el éter, para las regiones superiores en donde se encontraban los planetas y las estrellas. En este sistema de esferas concéntricas, la Tierra ocupa el cen­tro y además se encuentra estática. En el Universo las cosas tienen su lugar natural y tienden a permanecer en él. Según su cosmología, si un objeto sólido es llevado a cier­ta altura y se suelta, éste tenderá a su posición original y al llegar a ésta permanecerá en estado de reposo. También propuso que un gas ascendería a través de la tierra y del agua hasta ocupar su lugar en la esfera del aire, en donde permanecería en reposo. Lo mismo ocurriría con los demás elementos: al desplazarlos de su lugar natural y soltar­los, tenderían a buscar la esfera a la que correspondían, mediante un movimiento ver­tical hacia arriba o hacia abajo.

En el caso de la esfera celeste, que envolvía a los otros cuatro elementos, los cuer­pos celestes no se encontraban en reposo, sino en un movimiento que ahora conoce­mos como circular uniforme.

Aristóteles llamaba movimiento natural al que realizaba un objeto para regresar a su estado natural (el reposot en la esfera que le correspondía. Por el contrario, al mo­vimiento generado por un factor externo, al que denominó fuerza, le llamó movimien­

.... FIGURA 1.4

1.2 Antecedentes históricos 5

Page 26: Fisica_I

~ FIGURA 1.5

~ FIGURA 1.6 Copémico.

to violento. Tales movimientos deberían de cesar al eliminar la fuerza, que era tempo­ral y contingente, capaz de alterar el estado natural de las cosas, obligándolas a mover­se. Al desaparecer la fuerza, el objeto quedaría en reposo si estaba en la esfera que le correspondía, y si no, adquiriría su movimiento natural, para alcanzar su lugar. Por lo que toca al movimiento en la Tierra, Aristóteles aceptaba el punto de vista "sensato" de que se necesita siempre una fuerza neta para que un objeto se mantenga en movi­miento continuo.

En lo que respecta a la caída de los cuerpos, Aristóteles decía que los más pesados caían más rápido, ya que contenían mayor cantidad del elemento tierra. En dicha caí­da la velocidad adquirida era directamente proporcional al peso (a mayor peso, mayor velocidad y a menor peso, menor velocidad), e inversamente proporcional a la resis­tencia del medio (a mayor resistencia, menor velocidad ya menor resistencia, mayor velocidad). A partir de este movimiento de caída libre, Aristóteles explicaba que el va­cío no existe, ya que en él la resistencia sería cero, adquiriendo el objeto una velocidad infinita, lo cual era inconcebible y absurdo.

La teoría aristotélica empezó a tener contradicciones al tratar de explicar algunos problemas típicos de la época, por ejemplo, el de la flecha que se dispara, la cual, al ce­sar la fuerza que ejerce la cuerda sobre ella, debería caer hacia la Tierra, de acuerdo a su mismo argumento, lo cual no ocurre. Para subsanar este problema los aristotélicos argumentaban que al moverse la flecha hacia adelante, generaba un vacío en la parte de atrás y como éste no estaba permitido en la naturaleza, el aire se apresuraba a llenar el espacio vacío, impulsando a la flecha a continuar su vuelo. Con este argumento se pretendía demostrar que el aire detenía y a la \'ez generaba el movimiento de los cuer­pos (¿figura 1.5).

Otro problema que surgió y que no iba de acuerdo con el razonamiento aristotéli­co, era el de la caída libre de los cuerpos, en donde se establecía que éstos lo hacían a velocidad constante. Al considerar a un mismo objeto que caía de diferentes alturas, se observó que éste lo hacía a mayor velocidad desde una posición más alta. Los aristoté­licos argumentaban que al ir cayendo, era mayor el peso del aire que estaba por enci­ma del objeto que el que iba quedando por debajo de él, por lo que la resistencia era menor, y la fuerza que lo impulsaba, debido al peso del aire de arriba, era cada vez ma­yor, por tanto, la velocidad aumentaba. Por otra parte, se distinguió que si se tienen dos objetos, uno con el doble del peso que el otro, el más pesado no caía en la mitad del tiempo que tardaba el más ligero en hacerlo desde la misma altura. En esta última ob­servación, se planteaban serias dudas acerca de la propuesta aristotélica, la cual afir­maba que la velocidad con la que choca un cuerpo contra el suelo, en caída libre, es proporcional a su peso.

1.2.2 Modelo Clásico (mecanicisfa o newtoniano) En el Renacimiento se generó un desafío total a la concepción que los aristotélicos da­ban del Universo (teoría geocéntrica), siendo las ideas de ~icolás Copérnico (1473­1543) ("figura 1.6), su máxima expresión. Copérnico estableció la teoría heliocéntrica (el Sol en el centro del Universo), en la cual la Tierra, junto con los otros planetas, gira alrededor del Sol. En este sistema la Tierra no está fija, ya que aparte de girar alrededor del Sol, tiene un movimiento de rotación sobre el eje que pasa por los polos.

En el contexto social europeo del siglo x\·1 se presentó el enfoque de la ciencia (el estudio de la naturaleza) como un instrumento para que el hombre dominara el mun­do y acrecentara su poder sobre la Tierra, y no sólo adoptara una actitud contempla­tiva. De esta forma la física de Aristóteles no representaba ninguna utilidad. Este pensamiento se fue desarrollando a partir de la evolución de nuevas técnicas y su apli­cación en la producción capitalista. Es decir, el desarrollo de la nueva ciencia se logra al ir resolviendo problemas técnicos. Estas nuevas técnicas implican mejores medicio­nes y cálculos más precisos, lo cual permitió tener un lenguaje matemático apropiado.

Aparte del nuevo método de la ciencia y de su lenguaje, se desarrollaron nuevas técnicas como la de obserVar, a través de un telescopio, los planetas, el Sol y la Luna, mejorando así sus observaciones.

6 CAPíTULO 1 Introducción a la física

Page 27: Fisica_I

A FONDO

Galileo Galilei y la Torre de Pisa

Galileo Galilei (Fig. 1) nació en Pisa, Italia, en 1564 durante el Re­nacimiento. Hoy día se le conoce en todo el mundo por su nombre de pila y muchos lo consideran el padre de la ciencia moderna o el padre de la mecánica moderna y la física experimental, lo cual da idea de la magnitud de sus aportaciones científicas.

Una de las mayores contribuciones de Galileo a la ciencia fue el establecimiento del método científico, es decir, la inyesti­gación por experimentación. En contraste, el enfoque de _-'l.cistó­teles se basó en la deducción lógica. En el método científico, para que una teoría sea válida, debe predecir o coincidir con resc::ta­dos experimentales. Si no es así, o no es válida o debe modificar­se. Galileo dijo: "Creo que en el estudio de problemas naturales no debemos partir de la autoridad de lugares de las Escrihrras, sino de experimentos sensatos y demostraciones necesarias"!

Tal vez la leyenda más popular y conocida acerca de Gali­leo sea que realizó experimentos dejando caer objetos de la Torre de Pisa (Fig. 2). Se ha puesto en duda que Galileo lo ha a hecho realmente, pero de lo que no hay duda es de que cuestionó la perspectiva de Aristóteles respecto al movimiento de cuerpos que caen. En 1638, Galileo escribió:

Aristóteles dice que tma esfera de hierro de rnillibras que cae de tma altura de cien cúbitos llega al suelo antes que una esfera de una libra haya caído un solo cúbito. Yo digo que llegan al mismo tiempo. Al realizar el experimento, cons­tatamos que la más grande rebasa a la más pequeña por el espesor de dos dedos; es decir, cuando la ma 'or ha llega­do al suelo, la otra está a dos grosores de dedo del suelo; no creo que podamos esconder tras esos dos dedos los no­venta:: nueve cúbitos de Aristóteles.+

*De Grow¡h o Biologicnl T/lOl/g11f: Di¡¡ersity, Evollltim¡ & lnherifnnce, por F. Meyr (Cambridge, MA: Harnrd Unü'ersity Press, 1982).

tDe Aristotle, Gnlileo m¡L! fhe Tower ofPi5n, por lo Cooper (Ithaca, ~1Y: Car­nell University Press, 1935).

FIGURA 1 Galileo Se dice que Galileo realizó experimentos de caída libre dejando caer objetos de la Torre de Pisa.

Éste y otros escritos revelan que Galileo conocía el efecto de la resistencia del aire.

Los experimentos en la Torre de Pisa supuestamente se efectuaron alrededor de 1590. En sus escritos de esa época, Ga­lileo dice haber dejado caer objetos desde una torre alta, pero nunca nombra específicamente la de Pisa. Una carta que otro científico escribió a Galileo en 1641 describe la acción de dejar caer una bala de cañón y una de mosquete desde la Torre de Pi­sa. El primer relato que menciona un experimento similar de Ga­lileo lo escribió Vincenzo Viviani, su último discípulo y primer biógrafo, doce años después de su muerte. o se sabe si Galileo se lo contó a Viviani en sus años postreros o si Viviani creó esta imagen de su antiguo maestro.

Lo importante es que Galileo reconoció (y probablemente demostró experimentalmente) que los objetos en caída libre caen con la misma aceleración, sea cual sea su masa o peso. (Véase la Fig. 2.14.) Galileo no explicó por qué todos los objetos en caída libre tienen la misma aceleración, pero Newton sí lo hizo, como veremos en un capítulo posterior.

FIGURA 2 La Torre de Pisa Construida como campanario de una catedral cercana, se edificó sobre un subsuelo inestable. La construcción se inició en 1173, y comenzó a tender hacia un lado y luego hacia el otro, antes de inclinarse en su dirección actual, Hoy día, la torre diverge unos:; m de la vertical en su parte más alta. Se cerró en 1990 y se hizo un intento por estabilizar la inclinación.

1.2 Antecedentes históricos 7

Page 28: Fisica_I

En el nuevo enfoque de la ciencia se estud'an las causas que producen los fe­nómenos y se enfatiza en aquéllos que permi+en su reproducción de manera experimental así como su cuantificación, éstas son las dos características fun­damentales de la nueva ciencia.

.Á. FIGURA 1.7 Gailleo.

.Á. FIGURA 1.8 Kepler.

~ FIGURA 1.8A

Galileo (~figura 1.7), sentó las bases de esta "revolución científica" al proponer que todo conocimiento de la naturaleza debería establecerse por medio de la experimenta­ción, reproduciendo el fenómeno de manera controlada (midiéndolo o cuantificándo­lo). A partir de estas observaciones particulares se fueron elaborando teorías generales que a su vez se utilizaron para describir casos particulares. De estos casos particulares se llega a casos generales, estableciéndose así el ciclo básico de la física experimental. Este método es llamado inductivo-deductivo. Al mismo tiempo que se desarrollaba un método para la nueva ciencia, se buscaba un lenguaje preciso y capaz de describir los fenómenos, cuantificar las leyes y principios, y comprobar las teorías. Para ello se adoptó el lenguaje matemático por ser éste el más adecuado para dichos fines.

Partiendo de estas características fundamentales de la nueva Física, en donde se adquiere un método basado en la experimentación, un lenguaje claro y preciso con base en las matemáticas y técnicas adecuadas para el estudio de los fenómenos de la naturaleza, se lograron grandes avances en la ciencia, cuya finalidad ya no era la contemplación. sino la satisfacción de la demanda social, a través de su aplicación en el desarrollo tecnológico.

Johannes Kepler (1571-1630) (~figura 1.8) demostró que los planetas giran describien­do una elipse y que el Sol se encuentra en uno de los focos. A su vez Galileo Galilei (1564-1642) mostró argumentos, con base en las observaciones hechas del Sol, en favor del movimiento de la Tierra. El Universo empezó a unificarse y a perder su orden je­rárquico, el cual se consideraba como un conjunto estático de regiones de diferentes categorías, apareciendo como un conjunto de cuerpos en interacción, en movimiento y evolución continuas. Se presentaba como una máquina cuyo funcionamiento había que desentrañar, por medio de encontrar y descubrir las causas que lo gobiernan. En esta nueva concepción de la ciencia, los fenómenos no se explicarían en términos lógi­cos, sino mediante relaciones de causa y efecto mecánicos. De esta forma la contempla­ción sublime cedió su lugar a la acción provechosa.

En este proceso participaron los más grandes talentos de esta época, la cual culmi­na con la formulación de los Principios matemáticos de la filosofía natural de Isaac Newton (1642-1727) (~figura 1.9). Esta obra es considerada como un tratado mecánico-matemá­tico, que representó la base para el desarrollo de la estructura de la Física y de la cien­cia en general. La contemplación aristotélica cedió su lugar a las causas mecánicas de 0Jewton.

El conocimiento se orientó hacia el dominio de las leyes generales de la naturale­za, mediante las cuales se pretendía su control y aprovechamiento.

Como se ha mencionado con anterioridad, fueron los problemas prácticos los que generaron el desarrollo científico, por ejemplo, en el siglo XVI, los problemas plantea­dos en la navegación, hicieron posible el desarrollo conjunto de la mecánica y la astrono­mía, generándose entre ellas las condiciones para la gran síntesis de. ewton. En los trabajos de Kewton los resultados se obtienen por inducción, de los casos particulares a las leyes generales, y de éstas, por deducción, a casos particulares. Mediante este proce­dimiento logrÓ establecer la ley de la gravitación universal, en la cual analiza el compor­tamiento de los cuerpos que caen a la superficie de la Tierra y el comportamiento de los cuerpos celestes, unificando la mecánica y estableciendo el comportamiento dinámico del Universo.

Con la formulación de las leyes de Newton del movimiento, en las cuales se rela­ciona a la fuerza con el cambio en el movimiento, se derrumbó el viejo esquema aristo­télico, en donde de acuerdo al sentido común, la fuerza era necesaria para mantener el movimiento de un cuerpo. Asimismo, Newton mostró que el Universo se rige por le­yes matemáticas simples, como las referidas a la electricidad y al calor, las cuales fue­

8 CAPíTULO 1 Introducción a la física

Page 29: Fisica_I

ron construidas sobre un modelo newtoniano o mecanicista. Es posible deducir enton­ces que, a partir del siglo X'VI se llevó a cabo un desarrollo científico con bases experi­mentales que permitió enunciar las leyes de la naturaleza en las que se describe y se representa de manera matemática el comportamiento de ésta.

1.2.3 Modelo de la física moderna A finales del siglo XIX era una creencia común que todos los fenómenos naturales po­dían describirse mediante las leyes de Newton, los principios de la termodinámica y las leyes del electromagnetismo, los cuales se basaban en una concepción mecanicista del Universo. El desarrollo de la física moderna inicia a principios del siglo xx demos­trando que la mecánica clásica no es siempre aplicable. El estudio del movimiento de las partículas, a velocidades comparables con la de la luz, y la investigación del mun­do microscópico de los átomos, electrones, protones y otras partículas, impulsó el de­sarrollo de algunos campos de la física moderna, como son la relatividad y la mecánica cuántica (~figura 1.10).

La teoría de la relatividad fue desarrollada por Albert Einstein (1879-1955) (T figu­ra 1.11). A partir de la cual se llegaron a establecer algunas proposiCiones teóricas, que fueron demostradas experimentalmente tiempo después. De ellas se deduce que algu­nas cantidades, que en la física clásica se consideran constantes, ahora cambien, tales como la masa, el tiempo y la longitud, cuando las velocidades son cercanas a la de la luz. Otra consecuencia es que algunos conceptos clásicos que eran considerados inde­pendientes, ahora aparecen estrechamente relacionados (espacio-tiempo, masa-ener­gía). Una tercera aportación de la teoría de la relatividad es que la luz se desvía de su trayectoria al pasar junto a cuerpos de gran masa.

En relación con la mecánica cuántica, se realizaron estudios del mundo micros­cópico y se descubrió que en la interacción de la radiación con la materia, el compor­tamiento de los electrones puede ser descrito en términos ondulatorios y, a su vez, la radiación presenta un comportamiento como partícula (dualidad onda-partícula). Por otro lado, se encontró que las cantidades obseryadas no eran independientes del ob­servador y, por tanto, no era posible determinar una medición absoluta en la interac­ción objeto-observador. Esto nos lleva a que en la actualidad el estudio del micro mundo se realice con base en una noción probabilística al pretender pronosticar el comportamiento de la materia. Una consecuencia de ello es que al estudiar el movi­miento de una partícula no es posible determinar, de manera precisa, la posición y la velocidad al mismo tiempo, por lo que se imposibilita la utilización del esquema newtoniano.

Por último, cabe mencionar que cuando un objeto se mueve a velocidades peque­ñas, comparadas con la velocidad de la luz, la teoría de la relatividad se reduce a lasle­yes de la mecánica clásica. Asimismo, al estudiar el comportamiento de los cuerpos en el macro mundo, la mecánica cuántica se reduce también a las leyes de la mecánica clásica.

Podemos concluir que las ideas predominantes en la física son:

1) La del orden, desarrollada por Aristóteles, predominó hasta el siglo XVI, es de­cir que en la antigüedad, la ciencia consistía en ordenar las cosas.

2) La de una causa mecánica surge a partir de Galileo y Ne'Vvton; cuando la cien­cia tuvo como eje principal la búsqueda de la causa mecánica de los fenómenos obsen'ables. Predominó en los siglos :XVII, XVIII YXIX, a la física que se basa en di­cha idea se le conoce como física clásica.

3) La de un comportamiento probabilístico, se desarrolló a partir del inicio de es­te siglo y señala como concepto primordial la probabilidad de que la materia, a nivel microscópico, tiene cierto comportamiento. También se consideran las \'a­riaciones de alglmas cantidades, cuando las partículas se mueven a velocida­des cercanas a la de la luz. Estas cantidades (longitud, masa, tiempo, etc.) eran invariables en la física clásica.

... FIGURA 1.9 Newton.

... FIGURA 1.10 El estudio del mundo microscópico de los átomos, electrones, protones y otras partículas, impulsó el desarrollo de la física moderna.

... FIGURA 1.11 Einstein.

1.2 Antecedentes históricos 9

Page 30: Fisica_I

Actividad sugerida

Buscar las biografías de Aristóteles, Galileo Galilei, Isaac Tewton y Albert Einstein, señalar cuáles fueron sus aportaciones a la física.

Elaborar mapas conceptuales de los modelos de Aristóteles, ewton y de la fí­sica moderna.

Elaborar los mapas en Power Point y hacer una presentación de éstos en equipo.

DI Método científico Si partimos del hecho de que el mundo es ordenado, la ciencia se puede definir como un conjunto de conocimientos ordenados e interrelacionados. El conocimiento científi­co es sistemático, y se emmcia mediante proposiciones dispuestas jerárquicamente, en donde las del ni":el más bajo se refieren a hechos particulares y las del nivel más alto a las leyes que gobiernan el Cni"erso. Para expresar estas proposiciones, la ciencia defi­ne la totalidad de los conceptos que utiliza, de W1a manera concreta y sin ambigüe­dades. A la vez crea lenguajes artificiales como los signos algebraicos, los símbolos químicos, etc., a los que les atribuye W1 significado determinado y reglas para su ade­cuado manejo.

La ciencia trabaja a base de modelos, los cuales son representaciones propuestas para sistemas o relaciones que simplifican la comprensión de un problema dado. Los modelos no pretenden ser una representación ,'erdadera y absoluta, sino que hacen re­ferencia a una situación que conduce a los resultados obseryados. No se espera que el modelo sea absolutamente apegado a la realidad, sino que proporcione predicciones acertadas y que permita a"anzar en el entendimiento del objeto que estudia. Tampoco son estáticos, suelen ser modificados o susti:uidos por otros, cuando ya no dan una ex­plicación completa para lo que fueron diseñados (por ejemplo, el modelo aristotélico que fue reemplazado por el ne\\'tor.iano), o pueden ser restringidos a ciertos campos menos generales de lo que se pensaba originalmente (por ejemplo, la mecánica newto­niana se restringe al mundo macroscópico, ya que no tiene ,'alidez en el micro mundo).

Muchas \'eces el modelo que se constru::e es lma representación simplificada, la cual no se apega del todo a la realidad; posteriormente se van agregando los factores que se saben presentes, pero que de tomarlos en cuenta desde el principio, dificul­tarían el estudio del problema, por ejemplo, cuando se estudia el mm'imiento de un cuerpo que se desliza sobre una superficie, primero se trata el caso en donde se despre­cia la fuerza de fricción y después se aborda otro caso más apegado a la realidad, en donde se considera la fuerza de fricción. Existen modelos físicos y modelos matemá­ticos. Un modelo físico es W1a representación material de algo, como lm dibujo, una construcción a escala, etc. En la construcción de un modelo matemático se le hace co­rresponder a cada característica obsen'able una ,'ariable matemática, las cuales se rela­cionan entre sí mediante una ecuación, ejemplo, L' = d/t.

La herramienta que utiliza la ciencia para comprender las relaciones funcionales entre las cosas, con el mayor apego a la realidad, es el método científico. Este método nos conduce a la adquisición de un conocimiento científico y consta primordialmente de tres etapas:

Primera:,consiste en observar o investigar y describir los hechos. Segunda: plantearse una o varias preguntas o problemas, es decir, postular hipó­

tesis o proponer modelos, que busquen explicar los hechos u observaciones antes mencionadas.

Tercera: diseñar experimentos para probar la(s) hipótesi(s) o modelos, para acep­tar, rechazar o modificar cada uno de éstos.

Al estudiar la naturaleza, pueden postularse hipótesis o modelos tendientes a ex­plicar sus características o a establecer relaciones entre ellas. Al proponerse las hipóte­sis o modelos se debe procurar, primero, que las causas sugeridas para la explicación del fenómeno efecti"amente sean capaces de producirla y, además, que existan en la

1O CAPíTULO 1 Introducción a la física

Page 31: Fisica_I

naturaleza, por ejemplo, al observar que un cuerpo cae libremente, se puede proponer la hipótesis de que todos los cuerpos caen debido a la acción de la gravedad. En esta hipótesis, la causa sugerida de que los cuerpos caigan es la acción de la gravedad, lo cual ha sido demostrado experimentalmente. Por otra parte, cabe señalar que esta causa (la acción de la gravedad) existe en la naturaleza. Si se observa un solo caso en que no se cumple la predicción, la hipótesis o modelo debe ser modificado o rechaza­do. Si se demuestra, a través de experimentos, que son ciertas las predicciones de la hipótesis o modelos, tantas veces como se pruebe, se dice que es sostenible y se agrega al acervo científico.

Cuando un modelo o hipótesis debe ser sustitui­do por otro nuevo, éste debe explicar todos los casos planteados por la hipótesis o modelo anterior, ade­más de los nuevos que motivaron su creación.

Cabe aclarar que no existe ninguna regla para la postulación de hipótesis o la elaboración de mode­los. Éstos van siendo sugeridos por la obsen'ación y son el resultado de la lógica, la intuición, la experien­cia y la habilidad particular del científico. En rigor, el método científico es una especie de guía de proce­dimientos y reglas que sugieren, a grosso modo, cómo se realiza la investigación científica, sin garantizar en absoluto los buenos resultados. ~o existen reglas para generar el conocimiento científico, sólo indi­caciones y criterios para reconocer, como tales, los resultados del proceso. En nuestro curso de física aplicaremos continuamente este método, al construir el soporte teórico y su posterior 'verificación experi­mental. Esperamos que esta metodología le sirva al lector en la búsqueda de nuevos conocimientos en el área de las ciencias naturales y en particular de la física.

.. FIGURA 1.12 Una de las herramientas del método científico es la observación.

T FIGURA 1.13 Todos los cuerpos caen debido a la fuerza de gravedad.

1,3 Método científico 11

Page 32: Fisica_I

PAR T E 2

Sistemas de unidades

Las observaciones y experiencias que interesan a los científicos cubren un dominio in­menso. las distancias se extienden desde las dimensiones increíblemente pequeñas de las partículas subnucleares hasta los miles de años luz que separan a las galaxias del Universo. Los tiempos abarcan aquellos de la e\'olución estelar y los tiempos de \'ida mínimos de algunas "partículas elementales". Para lograr estas mediciones el hombre

~ FIGURA 1.14 El estóndar de masa del SI: el kilogramo El kilogramo se definió originalmente en términos de un "olumen específico de agua, un cubo de 0.10 IT\Por lado, con lo que se asoció el estándar de masa al estándar de longitud. Ahora el kilogramo estándar se define con üI1 cili...r¡dro metálico. El prototipo internacional del kilogramo se consen-a en la Oficina Francesa de Pesos " Medid2.s. Se le fabricó en la década de 1880 con una aleación de 90% platino y 10°'0 iridio. Se han producido copias para usarse como prototipos nacionales de 1 kg, uno de los cuales es el estándar de masa de Estados Unidos, que se guarda en el Instituto I'\acional de J\'ormas v Tecnología e ST) en '.\'ashington, D.C.

12 CAPíTULO 1 Introducción a la física

Page 33: Fisica_I

A FONDO

Nanotecnología

Los prefijos de las unidades métricas se refieren al orden de la dimensión de medición empleada. El prefijo micro- (10-6) se ha usado comúnmente desde hace mucho, como en microscopio, microchip, microbiología, microquímica, etc. Ahora se está co­menzando a usar el prefijo nano-, y seguramente nos toparemos a menudo con él cuando se hable de nanotecnología.

En general, nanotecnología es cualquier tecnología que se practica a escala de nanometros. Un nanometro es una milmillo­nésima (10-9

) de metro, aproximadamente la anchura de tres o cuatro átomos. Básicamente, la nanotecnología implica la fabri­cación o construcción de cosas átomo por átomo o molécula por molécula, así que el nanometro es la escala apropiada. Esto po­dría parecer increíble, pero no lo es. El físico Richard Feynman sugirió la posibilidad en 1956: "Los principios de la física, hasta donde los entiendo, no descartan la posibilidad de manipular las cosas átomo por átomo". Esto se ha logrado con la ayuda de un microscopio especial a muy bajas temperaturas, "empujan­do" átomos individuales (Hg. 1).

En cierto sentido, se usa nanotecnología natural en las cé­ FIGURA 1 Hombre molecular Esta figura se creó despla­lulas de nuestro cuerpo. Es en los ribosomas (diminutas estruc­ zando 28 moléculas, una por una. Cada saliente dorada es la turas presentes en todas las células vivas) donde la información imagen de una molécula de monóxido de carbono. Las molécu­del código genético se convierte en moléculas de proteína. Los las descansan en la superficie de un solo cristal de platino. El ribosomas son como minúsculas "máquinas", de sólo unos cuan­ "hombre molecular" mide 5 nm a lo alto y 2.5 nm a lo ancho tos nanometros de longitud, que leen las instrucciones del Dj A (de mano a mano). Se necesitarían más de 20 000 figuras, uni­para ensamblar enzimas y otras proteínas molécula por molécula. das de la mano, para abarcar un solo cabello humano. Las mo­

Quizá ahora el lector perciba el potencial de la nanotecnolo­ léculas de la figura se acomodaron empleando un microscopiogía. Conocemos bien las propiedades químicas de los átomos y especial a temperaturas muy bajas.moléculas. Por ejemplo, un reacomodo de los átomos de la hulla puede producir un diamante. (Ya hacemos esto sin nanotecnolo­gía, utilizando calor y presión.) La nanotecnología presenta la posibilidad de construir novedosos dispositivos o "máquinas" moleculares con propiedades y capacidades extraordinarias. He aquí algunas posibles aplicaciones que los científicos querrían desarrollar: Microscopio de barrido por túnei

1+-------- Una oscilación -------.-1

D Cesio 133 1 s = 9192631 no oscilaciones Detector

(a) de radiació'1

lb)

... FIGURA 1.15 El estándar de tiempo en el 51: el segundo El segundo se llegó a definir en términos del día solar medio. (a) Ahora se define con base en la frecuencia de la radiación asociada a una transición atómica. (b) El "reloj" atómico de fuente que se muestra aquí, en ell\lST, es el estándar de tiempo para Estados Unidos. La variación de este reloj es de menos de un segundo cada 20 millones de años.

1.3 Método científico 13

Page 34: Fisica_I

A FONDO .

• Suministro de medicamentos. Se podrían inyectar nanoestruc­ mundo se talló de silicio cristalino. ¿Cómo se hizo esto? La clave turas en el cuerpo para que se dirijan a sitio específicos, di­ es la tecnología de haz de electrones. La litografía por haz de gamos un tumor canceroso, y suministren un medicamento electrones es una técnica para crear patrones extremadamente fi­directamente. Así, los demás órganos del cuerpo no sufri­ nos, mucho más pequeños de lo que el ojo puede distinguir sin rían los efectos del medicamento. (Este proceso podría lla­ ayuda. Se desarrolló a partir de la antigua tecnología de los rrú­marse nanoquimioterapia.) croscopios electrónicos. La nanoguitarra se hizo por diversión y

como ilustración; no produce música. (Tal vez los científicos no• NmlOcomputación. Pronto se necesitará una nueva tecnolo­

han podido hacer una "nanouña" con la cual pulsadas cuerdas.)gía para producir dúps de computadora más pequeños o De hecho, las cuerdas, que tienen 1ma anchura de apenas 50 na­

más rápidos. A medida que se incluyen más y más transis­nometros (que equivale a unos 100 átomos), podrían pulsarsetores en los chips, los "alambres" rrúcroscópicos que conec­con técnicas especiales de microscopía y vibrarían, pero a fre­

tan a los transistores deben hacerse más pequeños y delgados. cuencias inaudibles. (Ya han llegado a un espesor de unos cuantos centenares de

Es difícil captar o visualizar el nuevo concepto de nanotec­átomos.) Cuando los alambres tienen un espesor de unos nología. No obstante, debemos tener presente que un nanome­cuantos átomos, el paso de señales eléctricas los destruye tro es una milrrúllonésima de metro. El diámetro de un cabelloexplosivamente. La salvación podría ser el "nanotubo", humano es de unos 200 000 nanometros: enorme en compara­una nanoestructura de átomos de carbono, con forma de tu­ción con las nuevas nanoaplicaciones. El nanofuturo segura­bo hueco, que puede transportar las señales eléctricas sin mente será interesante. destruirse. Ya se han creado nanotubos, y se está inyesti­

gando su aplicación.

• Mejoramiento de telas. Una empresa de investigación está de­sarrollando nanoestructuras que se pueden unir a fibras. Un tipo, al UTÚrse a fibras de algodón, hace que los líquidos derramados formen gotas en la superficie de modo que puedan limpiarse rápidamente, lo que hace a la tela tanto repelente al agua como resistente a las manchas. Otro tipo se aplica a fibras sintéticas, como el poliéster. La nanoes­tructura hace que la tela actúe como lo hacen las fibras na­turales de la ropa, desplazando la humedad del cuerpo de la persona hacia la superficie de la tela, para que se evapore

FIGURA 2 Nanoguitarra La guitarra más pequeña del mun­rápidamente y la ropa se sienta más cómoda. do rrúde 10 [Lm de largo --el tamaño de una célula- y tiene A fin de ilustrar la nueva tecnología, ciertos científicos crea­ seis cuerdas, cada una con una anchura de unos 50 nm (100 áto­

ron una "nanoguitarra" (Fig. 2). La guitarra más pequeña del mos). Está hecha de silicio cristalino.

ha desarrollado, en el transcurso de su historia, algunos sistemas de unidades median­te los cuales ha pretendido llevar a cabo dichas mediciones, es decir, comparar la mag­nitud de un objeto con otra que le sirve de base o patrón. Uno de los puntos fundamentales en el que nos centraremos en este capítulo, es el de encontrar el patrón de medida.

111 Cantidad tísica Se llama cantidad física a todo aquello que puede ser medido y que tiene una repre­sentación en el mundo real. Ejemplos de cantidades físicas son la longitud, la masa, el volumen, la \'elocidad, etcétera.

Para efectuar la medición de alguna cantidad física, primero debemos fijar, de ma­nera arbitraria o convencional, nuestra unidad o patrón de medida.

Una unidad o patrón de medida es toda magnitud de valor conocido y per­fectaMente definido que se toma como referencia para medir y expresar el valor de otras magnitudes de la misma especie,

Para medir una cantidad física, se compara esta cantidad con la unidad correspondiente. La medición es una descripción cuantitativa de dicha cantidad, mediante la asigna­ción de un número.

14 CAPíTULO 1 Introducción a la física

Page 35: Fisica_I

ID Clasificación de las cantidades físicas Las cantidades físicas se dividen en fundamentales y derivadas.

Fundamentales: son aquellas que se seleccionan de manera arbitraria y que no se definen en función de otras magnitudes físicas: longitud¡ masa, tiempo, carga eléctrica, corriente eléctrica, intensidad luminosa, etcétera.

Cantidades físicas

Derivadas: éstas se forman a partir de las fundamentales: velocidad, aceleración, fuerza, impulso, cantidad de movimiento, peso, etcétera.

Además, las cantidades físicas pueden ser escalares o vectoriales.

Las cantidades escalares son aquellas que sólo tienen magnitud (número y unidad).

Longitud

Masa

Tiempo

Cantidades escalares

Temperatura Distancia

Rapidez Trabajo

Energía T FIGURA 1.16

Potencia

Las cantidades vectoriales además de magnitud poseen dirección y sentido. Las cuales se representan gráficamente, por medio de segmentos de rectas terminados en punta de flecha, donde el tamaño del segmento indica su magnitud, y su posición su dirección y la punta de flecha el sentido, algunos ejemplos son:

Desplazamiento (D)

Velocidad (v)

Aceleración (a) Cantidades

Fuerza (F)vectoriales Peso (iD)�

Impulso (I)�

Cantidad de movimiento (ji)�

A continuación se describe un ejemplo para distinguir la diferencia entre dirección y sentido.

1,5 Clasificación de las cantidades físicas 15

Page 36: Fisica_I

Ejemplo 1

En una pista recta horizontal orientada norte-sur, usted camina 2.5 km al norte y su hermano camina 1.8 km al sur, en este ejemplo ambos caminarán en la misma dirección (norte-sur) pero en diferente sentido.

Sur Norte

ID Sistema Internacional de Unidades (SI) Para describir y caracterizar fenómenos, los científicos deben estar de acuerdo en un conjunto de unidades con el cual se comparen las mediciones y se evite la con­fusión, Esto trajo como consecuencia un acuerdo internacional llamado Sistema Internacional de Unidades: SI.

Longitud: es la distancia que cubre un segmento de línea para unir dos puntos,� Maso: es la propiedad que tiene un cuerpo de resistirse a cambiar su estado� de reposo o movimiento,� Tiempo: íntervalo que transcurre entre dos sucesos determinados.�

Una de las características más convenientes del Sistema Internacional de Unidades es su base decimal (su antecesor es el llamado Sistema :Jétrico Decimal, adoptado desde 1795 en Francia). Los kilómetros, microgramos, nanosegundos o megawatts se derivan de las unidades básicas o patrón a tra\'és de la multiplicación de potencias enteras de 10, existen prefijos estandarizados para cada potencia significativa de 10.

Unidades fundamentales

Medición Unidad Símbolo

Longitud :\1etro m

Masa Kilogramo kg

Tiempo Segundo s

Temperatura Kelvin K

Corriente eléctrica I Ampere A

Intensidad luminosa Candela Cd

Cantidad de sustancia Mol mol

Unidades derivadas

Medición M.K.5. Símbolo

Velocidad Metros / segundos • mis Aceleración Metros / segundos al cuadrado m/s2

)Área :Jetros cuadrados m-

Volmnen Metros cúbicos m 3

Fuerza Kilogramo metro/ N (Newton)

segundo al cuadrado

Trabajo Kilogramo metro al cuadrado/ J aoule)

Potencia /watt Kilogramo metro al cuadrado/ W (Watts)

segundo al cubo

Campo eléctrico eléctrico Kilogramo metro / segundo al

cuadrado / Coulomb

16 CAPíTULO 1 Introducción a la física

N/C

Page 37: Fisica_I

ID Equivalencias En virtud de la existencia de diferentes unidades (múltiplos y submúltiplos), es común tener una cantidad física expresada en dos o más unidades diferentes. Para ello vamos a mencionar algunas equivalencias entre unidades del Sistema Internacional.prefijos y sus respectivos símbolos usados para designar múltiplos y submúltiplos decimales.

Múltiplos Submúltiplos

Prefijo Símbolo Factor Prefijo Símbolo Factor

exa E 1018 deci d 10-1

peta P 1015 centi c 10-2

tera T 1012 mili m 10 3

giga G 109 micro m 106

mega M 106 « nano n 109 »

kilo K 103 pico P 10-12

hecto H 102 femto f 10-16

deca Da 101 atto a 10-18

Ejemplos

3 -9 91 km = 10 m, 1 nm = 10 m, 1 ns = 10- s

En cuanto al tiempo, tenemos que la unidad es el segundo, la misma unidad para los tres sistemas que se están considerando. Algunas de las equivalencias más comu­nes son:

Horas Minutos Segundos

Día 24 1440 86400

Hora 1 60 3600

Minuto 1/60 1 60

Para el caso de la medición de ángulo plano, que se manejará con cierto rigor en este módulo, tenemos las siguientes equivalencias:

Radián (rad) Grados (f") Revolución (rev)

Radián 1 57.3 1

Grado 1/57.3 1 1/360

Revolución 271 360 1

Se pueden establecer también equivalencias para las unidades derivadas, al igual que para las fundamentales. Observe los siguientes ejemplos.

Ejemplo 2

Obtener la equivalencia de metros cuadrados a centímetros cuadrados.

1 m = 100 cm

(1 m)2 = (100 cm)2 elevando al cuadrado ambos de la igualdad 2 2 21 m = 10 000 cm equivalencia entre m2y cm .

1.7 Equivalencias 17

Page 38: Fisica_I

Ejemplo 3

Obtener la equivalencia de decímetros cuadrados en centímetros cuadrados.

1 dm = 10 cm

(1 dm)3 = (10 cm)3 elevando al cubo tiene que este volumen de 1000 cm3

equivale también ambos lados de la igualdad, sea 1 litro,

1 dm3 = 1000 cm3 es decir, 1 litro = 1000 cm3

A FONDO

¿Es importante la conversión de unidades? Júzguelo usted

La respuesta a esta pregunta es: ¡claro que sí! He aquí lU1 par de casos que lo demuestran. En 1999, la nave I\Iars Climate Orbiter, que costó 125 millones de dólares, iba rumbo a Marte para in­vestigar su atmósfera (Fig. 1). La nave se acercó al planeta en septiembre, pero de repente se perdió el contacto entre ella y el personal en la Tierra, y nlU1ca se volvió a saber del Orbiter.

Las investigaciones revelaron que el Orbiter se había acer­cado a Marte a lU1a altura mucho menor que la planeada. En lu­gar de pasar a 147 km (87 mi) de la superficie marciana, los datos de rastreo mostraron que la trayectoria de la nave la habría lle­vado quizá a 57 km (35 mi) de la superficie: lU10S 80 km (50 mi) más cerca del planeta que lo planeado. El resultado fue que la nave se quemó en la ahnósfera marciana o bien se estrelló contra la superficie.

¿Cómo pudo pasar esto? Las investigaciones revelaron que el fallo del Orbiter fue primordialmente lU1 problema de con-

FIGURA 1 Mars elimate Orbiter Concepción artística del Orbiter cerca de la superficie de Marte. El Orbiter real se quemó en la atmósfera marciana o bien se estrelló con la superficie. Se atribuyó la causa a lU1a confusión con las unidades, y se perdió lU1a nave de 125 millones de dólares.

versiones de unidades, o de no efectuarlas. En Lockheed Martin Astronautics, que construyó la nave, los ingenieros calcularon la información de navegación en unidades inglesas. Cuando los científicos del jet Propulsion Laboratory de la NASA recibieron los datos, supusieron que estaban en unidades métricas, como se estipulaba en las especificaciones de la misión. No se con­virtieron las unidades, y se perdió lU1a nave de 125 millones de dólares en el Planeta Rojo, dejando muchos rostros rojos de ver­güenza.

Un vuelo no tan memorable, más cercano a la Tierra, fue el del "Planeador Gimli" en 1983. El vuelo 143 de Air Canada via­jaba de Montréal a Edmonton, Canadá, con 61 pasajeros en un t1amante Boeing 767, que en ese entonces era el jet comercial más avanzado del mundo. El avión había cubierto casi la mitad de su trayecto cuando se encendió una luz de ad,-ertencia de una bomba de combustible; luego se encendió otra, y finalmente las de las cuatro bombas. Los motores se pararon, convirtiendo al moderno avión en un planeador, a lU1as 100 millas del aeropuer­to grande más cercano, en Winnipeg.

Sin motores, el descenso del vuelo 143 lo haría tocar tierra a 10 millas del aeropuerto, por lo que se le desvió a un viejo cam­po de aterrizaje de la Real Fuerza Aérea Canadiense en Gimli, que se había convertido en lU1a pista de carreras de automóviles con barreras en los extremos. El piloto maniobró el "planeador" para aterrizarlo, logrando pararlo justo antes de chocar con la barrera, de ahí el nombre Planeador Gimli. ¿Las bombas de com­bustible eran defectuosas? o, ¡el avión se había quedado sin combustible!

Este conato de desastre se debió a otro problema de conver­sión. Parece que las computadoras del sistema de combustible no estaban funcionando bien, así que los mecánicos habían usa­do el antiguo procedimiento de medir el combustible en los tan­ques con una varilla. La longitud de la varilla que se humedece permite a los mecánicos estimar el volumen de combustible empleando factores de conversión tabulados. Durante años, Air Canada había calculado cantidades de combustible en libras, mientras que el consumo de combustible del nuevo 767 se ex­presaba en kilogramos. Peor aún, el uso de la varilla daba la cantidad de combustible cargado en litros, no en libras ni en ki­logramos.

Estos incidentes destacan la importancia de usar las unida­des apropiadas, realizar conversiones correctas de unidades y trabajar de forma consistente en el mismo sistema de unidades. Varios problemas al final del capítulo obligarán al lector a afinar su destreza para convertir unidades con exactitud (véase Ejerci­cios, sección 1.5, Conversiones de unidades).

18 CAPíTULO 1 Introducción a la física

Page 39: Fisica_I

ID Factor de conversión A partir de una equivalencia se obtienen dos factores de conversión, que se generan al dividir ambos lados de la igualdad, por uno de los dos términos de la equivalencia.

Con base en este ejemplo se deduce que a partir de una equivalencia, se obtie­nen dos factores de conversión, al dividir primero por uno y luego por el otro lado de la igualdad. Otra observación importante es que un factor de conversión siempre es igual a uno.

111 Conversión de unidades En ocasiones, es necesario expresar una cantidad física en otras unidades, bien sea del mismo sistema o de otro.

Para efectuar una conversión de unidades, se deberán realizar los siguientes pasos:

a) Escribir la cantidad que se va a convertir con sus unidades. b) Seleccionar la equivalencia que relacione las unidades dadas con las unidades

deseadas. c) Formar el factor de conversión adecuado, según las unidades deseadas, a partir

de la equivalencia. d) Multiplicar la cantidad original por el factor de conversión adecuado y cancelar

las unidades no deseadas.

lII!l Resolución de problemas OBJETIVOS:� (a) Establecer un procedimiento para resolver problemas y (b) apli­

carlo a problemas representativos.

Un aspecto importante de la física es la resolución de problemas. En general, ello im­plica aplicar principios y ecuaciones de física a los datos de una situación específica para encontrar el valor de una cantidad desconocida o deseada. No existe un método universal para atacar un problema que produzca automáticamente W1a solución.

No obstante, vale la pena tener presentes unos cuantos pLmtos generales:

•� Asegúrese de entender el problema. la base del éxito al resolver problemas es enten­derlo perfectamente. Antes de comenzar a resolver un problema, acostúmbrese a hacer una lista de todo lo que se da o se conoce (como haremos en casi todos los ejemplos de este texto). Es muy fácil pasar por alto información crucial.

•� Para llegar de los datos dados a la solución, podría ser necesario idear una estrate­gia o plan. Muchos problemas no pueden resolverse encontrando simplemente una ecuación e "insertando" en ella las cantidades dadas para obtener la solución. En muchos casos, será preciso realizar pasos intermedios, cada W10 de los cuales lo acercará a la solución final.

•� Recuerde que las ecuaciones son expresiones de principios físicos. Resolver problemas implica traducir los principios al "idioma" de las ecuaciones, pero también hay que saber examinar una ecuación y determinar que relación física encarna.

•� Una causa común de fracaso al resolver problemas es aplicar indebidamente principios o ecuaciones de física. los principios y ecuaciones generalmente están sujetos a ciertas condiciones o tienen aplicabilidad limitada a situaciones físicas.

•� Muchos problemas pueden resolverse con dos o más métodos. Si entiende bien el proble­ma y los principios pertinentes, probablemente podrá encontrar la forma más rá­pida y fácil de resolverlo.

Aunque no existe una fórmula mágica para resolver problemas, si hay varias prác­ticas recomendadas que pueden ser muy útiles. los pasos del procedimiento que sigue pretenden ofrecer un marco general que se puede aplicar a la resolución de la mayor

..� FIGURA 1.17 Pesas.

1 cIT3 = 1 mL = 1 cc�

DJ� (1 cm3)� 1000 cm3 = 1 L

~ T 10 cm

1 f--l0cm~cm

(a) Volumen

Masa de 1 mL de agua = 1 g

DJ Masa de 1 L agua = 1 kg

T 10 cm

Agua 1 (=4°C)

1--10cm~cm (bl Masa

.. FIGURA 1.18 El litro y el kilogramo Otras unidades métricas se derivan del metro. (a) Una unidad de \'olumen (capacidad) es el \'olumen de un cubo de 10 cm (0.01 m) por lado, y se llama litro (L). (b) la masa de un litro de agua (a su densidad máxima) se definió como 1 kg. Obsén'ese que el cubo de decímetro contiene 1000 cm3, o 1000 mL Así, 1 cm3

, o 1 ml, de agua tiene una masa de 1 g.

1.10 Resolución de problemas 19

Page 40: Fisica_I

1. Haga una lista de los datos y lo que se pide

� 2. Haga un diagrama

(si es apropiado)

Ü 3. Determine qué principios�

y ecuaciones son pertinentes y� cómo usarlos�

� 4. Simplifique ecuaciones

algebraicamente y despeje incógnitas en forma simbólica

� 5. Verifique unidades y haga conversiones si es necesario

Ü 6. Inserte valores numéricos y haga

cálculos, observando cifras significativas

Ü 7. Compruebe la respuesta:�

¿Es razonable?� (¿ Es lógica físicamente 7)�

... FIGURA 1.10 Diagrama de flujo del procedimiento sugerido para resolver problemas

parte de los problemas que se plantean en el texto. En general, seguiremos estos pasos al resolver los problemas de ejemplo del texto. Se darán más sugerencias útiles para re­solver problemas en los puntos apropiados de los capítulos subsiguientes.

Procedimiento sugerido para resolver problemas 1.� Lea el problema detenidamente y analícelo. Anote los datos que se dan y lo que se pide. Al­

gunos datos quizá no se den explícitamente en forma numérica. Por ejemplo, si un automóvil "parte del reposo", su velocidad inicial es cero (vo = O). En algunos ca­sos, se espera que el lector conozca ciertas cantidades o que las busque en tablas.

2.� Dibuje un diagrama para visualizar y analizar la situación física del problema, si resulta apropiado. Este paso quizá no sea necesario en todos los casos, pero suele ser útil.

3.� Determine qué princípio(s) y ecuación(es) se pueden aplicar a esta situación y cómo pue­den llevarlo de la información dada a la pedida. Podría ser necesario idear una estrate­gia de varios pasos.

4.� Simplifique las expresiones matemáticas lo más posible por manipulación algebraica antes de insertar valores. Las relaciones trigonométricas (que se resumen en el apéndice 1) pueden servir para simplíficar ecuaciones. Cuantos menos cálculos realice, menos probable será que se equivoque, así que no inserte los números antes de tiempo.

5.� Verifique las unidades antes de efectuar los cálculos. Efectúe conversiones de unidades si es necesario para que todas las unidades estén en el mismo sistema (de prefe­rencia unidades estándar). Esta práctica evita la mezcla de unidades y facilita el análisis de unidades. (La verificación y conversión de unidades generalmente se efectúa al anotar los datos en el paso 1.)

6.� Sustituya las cantidades dadas en la(s) ecuación(es) y efectúe los cálculos. Informe el resul­tado en las unidades apropiadas y con el número correcto de cifras significativas.

7.� Decida si el resultado es razonable o no. ¿La respuesta tiene una magnitud razonable? (Es decir, ¿es del orden correcto?) Por ejemplo, si la masa calculada para una per­sona resulta ser 4.60 X 102 kg, hay que dudar del resultado, pues 460 kg es un peso muy alto.

20 CAPíTULO 1 Introducción a la física

Page 41: Fisica_I

Ejemplo 4

Convertir 13 200 m a km. (Siguiendo los pasos citados anteriormente.)

a) 13 200 m b) 1 km = 1000 m Equivalencia.

c) 1 km = 1 p.e. de metros a kilómetros. 1000 m

d) 13200 m[ 1 km = 1] o Realizando operaciones 1000 m

13 200 km = 13.20 km� 1000�

Ejemplo 5

Expresar 2.4 m2 en cm2.

a) 2.4 m2�

b) 1 m2 = 10000 cm2 Equivalencia.�

c) 10000 cm2

= 1� p. e. de metros2 a cenmnetros2

1 m 2

Ejemplo 6

Convertir 90 km/h a mis.

a) 90 km/h Equivalencia.� b) 1 h = 3600 s; 1 km = 1000 m�

p. e. de segundos a horas yc) Y[3~0~ s] =1 U:] =1 kilómetros a metros.�

d) [90km][~][1000 m]= 90000 =2S m�

h 3600 s 1 km 3600 s s

1.10 Resolución de problemas 21

Page 42: Fisica_I

Autoevaluación� 1.

2.

3.

4.

5.

6.

Los estudios realizados hasta finales del siglo XIX acerca de la mecánica, la luz, el ca­�lor, el sonido y el electromagnetismo forman parte de lo que comÚlUnente se llama:�

a) Física moderna.�

b) Física clásica.�

c) Física relativista.�

d) Física cuántica.�

e) Física aristotélica.�

Los estudios reaüzados desde principios del siglo xx acerca del comportamiento� de las partículas microscópicas y del movimiento de objetos a velocidades cerca­�nas a la de la luz, forman parte de lo que comÚlUnente se llama:�

a) Física moderna.�

b) Física clásica.�

c) Física relativista.�

d) Física cuántica.�

e) Física aristotélica.�

La idea en la ciencia de buscar una causa mecánica de los fenómenos observables� surge con:�

a) Sócrates y Aristóteles.�

b) Pascal y Arquímedes.�

c) Newton y Galileo.�

d) Orested y Faraday.�

e) Einstein y Planck.�

La idea de orden en la ciencia, es decir, "la ciencia consiste en ordenar las cosas",� fue desarrollada principalmente por:�

a) Aristóteles.�

b) Sócrates.�

c) Newton.�

d) Galileo.�

e) Einstein.�

Es una de las etapas primordiales en el método científico.�

a) Optimización.�

b) Memorización.�

c) Observación.�

d) Graficación.�

e) ConfJ.guración.�

Es otra de las etapas primordiales en el método científico.�

a) Optimización.�

b) Memorización.�

c) Postulación de hipótesis.�

d) Graficación.�

e) Configuiación.�

22 CAPíTULO 1 Introducción a la ñsica

Page 43: Fisica_I

7.� Alas unidades de medición que no se definen en función de otras magrútudes físi­cas se les llama:

a) Fundamentales.

b) Compuestas.

c) Derivadas.

d) Dimensionales.�

e) Adimensionales.�

8.� A las unidades de medición que se forman a partir de las magrútudes físicas prin­cipales se les llama:

a) Fundamentales.

b) Compuestas.

c) Derivadas.

d) Dimensionales.�

e) Adimensionales.�

9.� Es una unidad derivada:�

a) cm b) min c) g d) mis e) h�

10.� Es una unidad fundamental:

a) cm3 b) m 2 c) cmls d) m/s2 e) km

11.� 135 km/h equivalen a:

a) 225 mis b) 2250 mis c) 375 mis d) 22.5 mis e) 37.5 mis 12.� 15 mi s equivalen a:

a) 4.17 km/h b) 9 km/h c) 54 km/h d) 0.9 km/h e) 90 km/h

13.� Las magrútudes físicas que son vectoriales poseen:

a) Sólo magrútud.

b) Sólo dirección.

c) Sólo sentido.�

d) Todo lo anterior.�

14.� Las magrúhldes físicas que son escalares poseen:

a) Sólo magrútud.

b) Sólo dirección.

c) Sólo sentido.�

d) Todo lo anterior.�

15.� Es una cantidad física escalar:

a) Desplazamiento.

b) Velocidad.

c) Aceleración.

d) Peso.�

e) Rapidez.�

16.� Es una cantidad física vectorial:

a) Distancia.

b) Área.

c) Volumen.�

d) Peso.�

e)� ~apidez.

Autoevaluación 23

Page 44: Fisica_I

17. Para que el vector resultante de la suma de dos vectores sea igual a cero, se reque­riría que estos vectores tuvieran:�

a) La misma magnitud, dirección y sentido.�

b) Diferente magnitud y la misma dirección y sentido.�

c) La misma magnitud y dirección, pero sentidos contrarios.�

d) La misma magnitud y sentido, pero diferente dirección.�

e) Ningw1a de las anteriores.�

18.� Al sumar dos vectores, para que la magnitud del vector resultante sea el doble de la magnitud de cada vector original, se requeriría que estos dos vectores tuvieran:

a) La misma magnitud, dirección y sentido.

b) Diferente magnitud y la misma dirección y sentido.

c) La misma magnitud y dirección, pero sentidos contrarios.�

d) La misma magnitud y sentido, pero diferente dirección.�

19.� Si a un desplazamiento de 35 m al norte se le suma 55 m al sur, el desplazamiento resultante sería:

a) 20 m al sur.

b) 90 m al norte.

c) 20 m al norte.

d) 90 m al sur.�

e) -90 m al sur.�

20.� Si a un desplazamiento de 20 m al este se le suman 35 m al oeste, el desplaza­miento resultante sería:

a) 55 m al este.

b) 15 m al oeste.

c) 55 m al oeste.

d) 15 m al este.�

e) -55 m al este.�

24 CAPíTULO 1 Introducción a la física

Page 45: Fisica_I

Ejercicios�

I.� Efectúe las conversiones de unidades requeridas en los siguientes problemas y resuelve los problemas:

1. 1250 km a metros.

2. 80 cm a kilómetros.

3.� En zona escolar el límite de velocidad es de 30 km/h, ¿se estará rebasando� dicho límite si viajamos a 12 mis?�

4. 90 km/h a mi s.

5. 20 m a milímetros.

6.� Una cancha de fútbol tiene 100 m de largo y 60 m de ancho. ¿Cuáles son la� longitud y la anchura de la cancha en kilómetros?�

7. 2.5 kg a gramos.

8. 3 kg a toneladas.

9. 0.56 toneladas a gramos.

10. 3.5 horas a minutos.

11. 1210 minutos a horas.

12. 79 300 segundos a días.

Ejercicios 25

l .�

Page 46: Fisica_I

Cinell1ática: descripción LO delll1ovill1iento

2.1 Distancia y rapidez: cantidades escalares

2.2 Desplazamiento unidimensional v velocidad: cantidades vectoriales

2.3 Aceleración

2.4 Ecuaciones de cinemática (aceleración constante)

2.5 Caída libre

AFONDO • Galileo Galilei y la Torre

de Pisa

Aprender dibujando Coordenadas cartesianas y desplazamiento unidimenSional Signos de la velocidad y la aceleración

E l guepardo ,-a a todo galope. Es el más rápido de los animales terrestres, y puede alcanzar ,-elocidades de hasta 113 km/h (70 mi/h). la impresión de moúmien­

to es tan marcada en esta fotografía, que casi podemos sentir el paso del aire. Sin embargo, esta sensación de mm-imiento es una ilusión. El moYimiento se da en el tiempo, pero la fo­to sólo puede "congelar" un instante. Veremos que, sin la dimensión del tiempo, prácticamente es imposible descri­bir el movimiento.

la descripción del mm'imiento implica representar un mundo inquieto. :'\ada está perfectamente inmóvil. Ellec­tor podría sentarse, aparentemente en reposo, pero su sangre fluye, y el aire entra y sale de sus pulmones. El aire se com­pone de moléculas de gas que se muewn con diferente ra­pidez y en diferentes direcciones. Y, aunque experimente quietud, el lector, su silla, la casa o el edificio en que está y

26

el aire que respira están girando en el espacio junto con la Tierra, parte de un sistema solar en una galaxia de movi­miento espiral dentro de un uni"erso en expansión.

la rama de la física que se ocupa del estudio del movi­miento, lo que lo produce y lo afecta se llama mecánica. los orígenes de la mecánica y del interés humano en el movi­miento se remontan a las primeras civilizaciones. El estudio de los mm'imientos de los cuerpos celestes (la mecánica ce­lestial) nació de la necesidad de medir el tiempo y la ubi­cación. Varios científicos de la antigua Grecia, destacando entre ellos Aristóteles, propusieron teorías del movimien­to que eran descripciones útiles, aunque después se demos­tró que eran incorrectas o estaban incompletas. Galileo (1564-1642) e Isaac ?\Tewton (1642-1727) formularon buena parte de los conceptos sobre el moúmiento que actualmen­te gozan de aceptación.

Page 47: Fisica_I

La mecánica suele dividirse en dos partes: (1) cinemática y (2) dinámica. La cine­mática se ocupa de describir el movimiento de los objetos, sin considerar qué lo causa. La dinámica analiza las causas del movimiento. Este capítulo cubre la cinemática y reduce la descripción del movimiento a sus términos simples considerando el movimiento en línea recta. Aprenderemos a analizar los cambios de movimiento: aceleración, frenado, parado. Al hacerlo, nos ocuparemos de un caso especialmente interesante de movi­miento acelerado: caída libre bajo la influencia únicamente de la gravedad. En el ca­pítulo 3 nos concentraremos en el movimiento en dos dimensiones (que fácilmente puede extenderse a tres dimensiones).

DI Distancia y rapidez: cantidades escalares OBJETIVOS: (a) Definir distancia y calcular rapidez, y (b) explicar qué es una

cantidad escalar.

Distancia Podemos observar muchos casos de movimiento a nuestro alrededor. ¿Qué es movi­miento? Esta pregunta parece sencilla pero el lector podría tener problemas para dar una respuesta inmediata (y no se vale usar formas del verbo "mover" para describir el movimiento). Después de meditarlo un poco, seguramente llegará a la conclusión de que el movimiento (o moverse) implica un cambio de posición. El movimiento puede describirse en parte especificando qué tan lejos viaja algo al cambiar de posición; es decir, qué distancia recorre. Distancia es simplemente la longitud total del trayecto reco­rrido al moverse de un lugar a otro. Por ejemplo, el lector podría viajar en automóvil de su pueblo natal a la universidad y expresar la distancia recorrida en kilómetros o mi­llas. En general, la distancia entre dos puntos depende del camino seguido ~ Fig. 2.1).

Igual que muchas otras cantidades en física, la distancia es una cantidad escalar. Una cantidad escalar es una cantidad que sólo tiene magnitud, o tamaño. Es decir, un escalar sólo tiene un valor numérico, como 160 km o 100 mi. (Cabe señalar que la mag­nitud incluye unidades.) La distancia es una cantidad escalar; únicamente nos dice la magnitud: qué tan lejos, pero no qué tan lejos en alguna dirección. Otros ejemplos de escalares son cantidades como 10 s (tiempo), 3.0 kg (masa) y 20°C (temperatura).

Rapidez Cuando algo se mueve, su posición cambia con el tiempo. Es decir, el objeto se mueve cierta distancia en cierto tiempo. Por consiguiente, tanto la longitud como el tiempo son cantidades importantes para describir el movimiento. Por ejemplo, imaginemos a un auto y un peatón que van por una calle y recorren una distancia (longitud) de una cuadra. Es de esperar que el automóvil viaje con mayor rapidez, y cubra la distancia en menos tiempo, que la persona. Esta relación puede expresarse utilizando longitud y tiempo para dar la rapidez con que cada uno cubre la distancia.

Rapidez media (s) es la distancia d recorrida; es decir, la longitud real del camino dividida entre el tiempo total ~ t que tomó recorrer esa distancia:

'd d' distancia recorrida \rapl ez me la = (2.1)

tiempo total para recorrerla _ d d 5=-=-­

..':lt t2-tl

Unidad SI de rapidez: metros por segundo (mis)

Nota: Un símbolo con una raya encima suele denotar un promedio. Se usa la letra grie­ga ~ para representar un cambio o diferencia en una cantidad; en este caso, el cambio de tiempo entre el inicio (t¡) y el final (t2) de un viaje, o el tiempo transcurrido.

Universidad estatalLejostlán ____

48 km (30 mI)

t

... FIGURA 2.1 Distancia: longitud total del trayecto Al ir de Natal a la Universidad estatal, un estudiante podría tomar la ruta más corta y recorrer una distancia de 81 km (SO mi). Otro estudiante sigue una ruta más larga para visitar un amigo en Lejostlán antes de volver a la escuela. El viaje más largo tiene dos segmentos, pero la distancia recorrida es la longitud total, 97 km + 48 km = 145 km. (90 mi).

Nota: Una cantidad escalar tiene magnitud pero no dirección.

Definición de: Rapidez media

2,1 Distancia y rapidez: cantidades escalares 27

Page 48: Fisica_I

¿ FIGURA 2.2 Rapidez instantánea El \'elocírnetro de W1 automóvil da la rapidez en W1 intervalo de tiempo muy corto, así que su lectura se aproxima a la rapidez instantánea.

¿ FIGURA 2.3 ¡En marcha! Sojourner corre a 0.60 mlmin (0.010 mis) por la superficie de Marte. Véase el ejemplo 2.1.

Cálculo de rapidez media

la unidad estándar de rapidez en el sr es metros por segundo (longitud/tiempo), aunque en muchas aplicaciones cotidianas se usa kilómetros por hora. La unidad in­glesa estándar es pies por segundo, pero también se usa mucho millas por hora.

Dado que la distancia es un escalar (igual que el tiempo), la rapidez también es es­calar. la distancia no tiene que ser en línea recta. (Véase la Fig. 2.1.) Por ejemplo, el lec­tor seguramente ha calculado la rapidez media de un viaje en automóvil calculando la distancia a partir de las lecturas inicial y final del odómetro. Supongamos que dichas lecturas fueron 17455 km Y17 775 km, respectivamente, para un viaje de cuatro horas. (Supondremos que el odómetro del automóvil marca kilómetros.) La resta de las lectu­ras da una distancia recorrida d de 320 km, así que la rapidez media del viaje es d/t =

320 km/4.0 h = 80 km/h (unas 50 rni/h). La rapidez media da una descripción general del movimiento en un intervalo de

tiempo, ilt. En el caso del viaje en auto con una rapidez media de SO km/h, la rapidez del vehículo no fue siempre 80 km/h. Con las diversas detenciones durante el viaje, el automóvil se debe haber estado moviendo a menos de 80 km/h una parte del tiempo. Por tanto, tu\'O que haberse estado moviendo a más de la rapidez media otra parte del tiempo. Una rapidez media no nos dice realmente con qué rapidez se estaba moviendo el automóvil en un instante dado durante el viaje. De forma similar, el puntaje medio que un grupo obtiene en un examen no nos dice el puntaje de un estudiante en par­ticular.

Si el intervalo de tiempo considerado, j,t se hace muy pequeño, cercano a cero, el cálculo de rapidez nos da una rapidez instantánea. Esta cantidad es la rapidez con que algo se está moviendo en un instante dado. El velocímetro de un auto da una rapidez instantánea aproximada. Por ejemplo, el velocímetro de la "Fig. 2.2 indica una rapi­dez de unas 44 rni/h, o 70 km/h. Si el automóvil viaja con rapidez constante (de modo que la lectura del velocímetro no cambie), la rapidez media y la instantánea serán iguales. (¿Está de acuerdo? Piense en la analogía de los puntajes de examen.)

Ejemplo 2.1 • Cámara lenta: Sojourner avanza

E14 de julio de 1997, el Pathfinder Lander bajó a la superficie de Marte, y de él salió rodando el explorador Soiourner ("Fig. 2.3).

Sofourner podía moverse con W1a rapidez máxima de 0.60 m/min. Con esta rapídez, ¿qué tiempo mínimo tarda el explorador en \'iajar 3.0 m para llegar a otra piedra y analizarla?

Razonamiento. Conocemos la rapidez media v la distancia, así que calculamos el tiempo con la ecuación de rapidez media.

Solución. Anotamos en forma sírnbólica los datos y lo que se pide:

1 ._)Dado: 5" = 060 ~ jJlli1 Hallar: '::"t (tiempo para recorrer la distancia d)(mi:rl 60 s

= omo mIs d = 3.0 m

En los datos, la unidad de metros por minuto se com'irtió a la unidad estándar de metros por segundo.

De la ecuación 2.1, tenemos

_ d s =­

'::"t Reacomodcm'do,

d 3.0m , . '::"1 = - = O I = 3.0 X 10- s (= 5.0 mm)

5" 0.01 m s

Ejercicio de refuerzo. (a) ¿Fue necesario convertir metros por minuto a metros por se­gundo? Explique. (b) Suponga que Sojoltmer tardó 15.0 min en recorrer los 3.00 m. ¿Qué rapi­dez media tuvo el explorador en este caso? (Las respuestas de todos los Ejercicios de refuerzo se dan al final del libro.)

28 CAPíTULO 2 Cinemática: descripción del movimiento

//

Page 49: Fisica_I

ID Desplazamiento unidimensional y velocidad: cantidades vectoriales

OBJETIVOS: (a) Definir desplazamiento y calcular velocidad, y (b) explicar la di­ Aprender dibujando ferencia entre cantidades escalares y vectoriales.

Desplazamiento En el movimiento en línea recta, o rectilíneo, conviene especificar la posición empleando el conocido sistema bidimensional de coordenadas cartesianas, con ejes x y y perpen­diculares. Una trayectoria recta puede tener cualquier dirección, pero por conveniencia solemos orientar los ejes de coordenadas de modo que el movimiento siga uno de ellos. (Véase Aprender dibujando.)

Como ya vimos, la distancia es una cantidad escalar que sólo tiene magnitud (y unidades). Sin embargo, al describir un movimiento podemos dar más información si especificamos una dirección. Esta información tiene especial utilidad cuando el cam­bio de posición es en línea recta. Definimos desplazamiento como la distancia en línea recta entre dos puntos, junto con la dirección del punto de partida a la posición final. A diferencia de la distancia (un escalar), el desplazamiento puede tener valores positi\'os o negativos, y el signo indica la dirección a lo largo del eje de coordenadas.

Por tanto, el desplazamiento es una cantidad vectorial. Un vector tiene tanto mag­nitud como dirección. Por ejemplo, cuando describimos el desplazamiento de un avión como 25 kilómetros al norte, estamos dando una descripción vectorial (magnitud y di­rección). Otras cantidades vectoriales son velocidad y aceleración.

Podemos aplicar álgebra a los vectores, pero necesitamos saber cómo especificar y manejar la parte de dirección del vector. Este proceso es relativamente sencillo en una dimensión. Para ilustrar esto con respecto a calcular desplazamientos, consideremos la situación que se muestra en la ~Fig. 2.4, donde x; y X2 indican posiciones en el eje x. Gn estudiante se mueve en línea recta de los casilleros al laboratorio de física. Como pue­de verse en la Fig. 2.4a, la distancia escalar entre los dos puntos es de 8.0 m. Para espe­cificar desplazamiento (un vector) entre Xl y x2, usamos la expresión

(2.2)

LABOR~ TORIO DE FISIC.t..

- ~' 1

,

x' X2 I x

1.0 20 30 4.0 50 60 7.0 80 9.0 ~ 0.0 11.0 12. O(metros)

l· 8.0 'li ·1 ..

(a) Distancia (magnitud o valor numérico)

X1 x2

x 10 20 30 4.0 11.0 12.0 (metros)

lit = x2 - x, = 9.0 m -1.0 m =+8.0 rl

lb) Desplazamiento (magnitud y dirección)

Coordenadas cartesianas y des­

plazamiento unidimensional

y

--::oP--------x (origen)

(a) Siste a bidimensional de coordenadas cartesianas. Un vector I

desplazamiento d ubica un punto Ix,

Y\ d,/',Y \ ;01'01')(,/

lb) En ovimiento unidimensional. o -ectilíneo, conviene orientar uno de los ejes de coordenadas en la

dirección del movimiento.

~ FIGURA 2.4 Distancia (escalar) y desplazamiento (vector) (a) La distancia (camino en línea recta) entre el estudiante v el laboratorio de física es de 8.0 m v es una cantidad escalar. (b) Para indicar desplazamiento, Xi y x, especifican las posiciones inicial y final, respecti\·amente. El desplazamiento es entonces ~ = x, - Xl = 9.0 m -1.0 m = -8.0 m; es decir, 8.0 ID en la dirección +x.

2.2 Desplazamiento unidimensional y velocidad: cantidades vectoriales 29

Page 50: Fisica_I

Noto: Cl siempre significo final menos inicial, osí como un cambio en un saldo bancario es el saldo final menos el saldo inicial.

Noto: Si el desplazamiento es en una dirección, la distancio es la magnitud del desplazamiento.

Noto: No confunda velocidad (un vector) con rapidez (un escalar).

Velocidad y rapidez

Definición de: Veiocidad media

donde ~ representa, una vez más, un cambio o diferencia en una cantidad, Entonces, como en la Fig. 2.4b, tenemos

~x = X2 - Xl = 9.0m - 1.0m = +8.0m

Así, el desplazamiento (magnitud y dirección) del estudiante es de 8.0 m en la direc­ción X positiva, como irldica el resultado (+) de la Fig. 2.4b. (Al igual que en las mate­máticas "normales", suele omitirse el signo más, pues se sobreentiende, así que este desplazamiento se puede escribir como .:lx = 8.0 m en lugar de ~x = +8.0 m.)

En este libro, las cantidades vectoriales por lo regular se irldican con negritas; por ejemplo, un vector de velocidad se irldica con v. No obstante, cuando se trabaja en una sa­la dimensión esa notación no es necesaria y se puede simplificar usando signos más y me­nos para irldicar las únicas dos direcciones posibles. Por lo regular se usa el eje x para los movimientos horizontales, y un signo más (..L) indica la dirección a la derecha, o en la "di­rección X positiva", mientras que un signo menos (-) irldica la dirección a la izquierda, o la "dirección x negativa". Tenga presente que estos signos sólo "apuntan" en direcciones espe­cíficas, Un objeto que se mueve sobre el eje x negatiyo hacia el origen se estaría moviendo en la dirección -r, aunque su posición x sea negativa. ¿Y un objeto que se mueve sobre el eje +x hacia el origen? Si el lector dijo "dirección negativa" (-), está en lo correcto.

Supongamos que la otra estudiante de la Fig. 2.4 camirla del laboratorio de física (la posición irlicial es diferente y Xl = 9.0 m) al firlal de los casilleros (la posición firlal ahora es X2 = 1.0 m). Su desplazamiento sería

~x = X2 - Xl = 1.0m - 9.0m = -8.0m

El signo menos irldica que la dirección del desplazamiento fue en la dirección x nega­tiva. En este caso, decimos que los desplazamientos de los dos estudiantes son iguales (en magnitud) y opuestos (en dirección).

Velocidad Como hemos visto, la rapidez, al igual que la distancia que implica, es una cantidad escalar: sólo tiene magnitud. Otra cantidad que se usa para describir movimiento es la velocidad. En la conversación cotidiana, solemos usar los términos rapidez y velocidad como sirlónirnos, pero en física tienen distinto significado. La rapidez es un escalar y la velocidad es un vector: tiene magnitud y dirección. A diferencia de la rapidez (pero igual que el desplazamiento), la velocidad puede tener valores positivos y negativos, que irldican direcciones.

La velocidad nos dice qué tan rápidamente se está moviendo algo y en qué direc­ción se está moviendo. Así corno podernos hablar de rapidez media e instantánea, te­nemos yelocidades media e instantánea que implican desplazamientos vectoriales. La velocidad media es el desplazamiento dividido entre el tiempo total de recorrido.

desplazamientovelocidad media (2.3)*

tiempo total de recorrido _ ~x X2 - Xl V = - =

M t2 - tl

Unid~d sr de \'elocidad: metros por segundo (mis)

'Otra forma muv utilizada de esta ecuación es

(x - .lo)

(t - t )o

que, después de reacomodar, queda así:

(2.3)

donde .lo es la posición inicial, x es la posición final y ':"f = t con to = Q. En la Sección 2.3 se explica es­ta notación.

30 CAPíTULO 2 Cinemática: descripción del movimiento

Page 51: Fisica_I

En esta ecuación, Dox es el desplazamiento y D.t es el intervalo de tiempo, donde tI es el tiempo inicial y t2 es el tiempo final.

En el caso de más de un desplazamiento (desplazamientos sucesivos), la velocidad media para todos los desplazamientos es igual al desplazamiento total o neto dividido entre el tiempo total. El desplazamiento total se obtiene sumando algebraicamente los desplazamientos según los signos de dirección.

Quizá el lector se pregunte si hay una relación entre rapidez media y velocidad media. Un vistazo a la Fig. 2.4 muestra que, si todo el movimiento es en la misma di­rección, es decir, si nunca se invierte la dirección, la distancia es igual a la magnitud del desplazamiento, y la rapidez media es igual a la magnitud de la velocidad media. ~o

obstante, hay que tener cuidado. Este conjunto de relaciones no se cumple si hay inver­sión de dirección, como en el ejemplo 2.2.

Ejemplo 2.2 • Ida y vuelta: velocidades medias

Un deportista trota de un extremo al otro de una pista recta de 300 m en 2.50 min Yluego tro­ta de regreso al punto de partida en 3.30 mino ¿Qué velocidad media tuvo el deportista al (a) trotar al final de la pista, (b) regresar al punto de partida y (c) el trote total?

Razonamiento. Las velocidades medias se calculan a partir de la ecuación de definición. Ca­

be señalar que los tiempos dados son los .lt asociados a los desplazamientos en cuestión.

Solución. El problema nos dice que:

Dado: ~x¡ = 300 m (tomando la dirección irúcial Hallar: Velocidades como positiva) medias

~X2 = -300 m (tomando la dirección de regreso para (a) el primer como negativa) tramo,

D.t¡ = 2.50 min (60 s/min) = 150 s }(conversión a (b) el tramo de = 3.30 min (60 s/min) = 198 s unidades estándar) reareso v_~t2 b

(c) el trote total

(a) La velocidad media al trotar hasta el final de la pista se calcula con la ecuación 2.3:

.lX1 300 m V. = -' = -- = -2 00 m/s

, .lt: 150 s .

(b) De forma similar, para el trote de regreso, tenemos

_ .lX2 -300 m V2 = - = = -1.52m/s

.lt2 198 s (e) Para el recorrido total, debemos considerar dos desplazamientos, de ida y vuelta, así que los sumamos para obtener el desplazamiento total, que luego dividirnos entre el tiempo total;

v.lX¡ + .lx¿ 300 m + (-300 m)

3 = = = Omis .lt¡ + '::'tL 150 s + 198 s

¡La velocidad media para el trote total es cero! ¿Ve el lector por qué? La defirúción de despla­zamiento dice que la magnitud del desplazamiento es la distancia en linea recta entre dos puntos. El desplazamiento de un punto a ese mismo punto es cero, asi que la velocidad me­dia es cero. (Véase la ~ Fig. 2.5.)

Podríamos haber calculado el desplazamiento total con sólo calcular '::'x = Xfinal - Xinidal = O- O= O, pero lo hicirnos en partes como ilustración.

Ejercicio de refuerzo. Calcule la rapidez media del trotador en cada caso del ejemplo, y compárela con las ,-elocidades medias respecti,-as. [¿La rapidez media en (c) será cero?] (Las respuestas de todos los Ejercicios de refuerzo se dan al final dellibro.J

Como muestra el ejemplo 2.2, la "elocidad media sólo ofrece una descripción muy general del movimiento. Una forma de estudiar más de cerca el movimiento es consi­derando intervalos de tiempo más pequeños, es decir, haciendo que el tiempo de ob­servación (D.t) sea cada \'ez más pequeño. Al igual que la rapidez, cuando .::. t se aproxima a cero, obtenemos la velocidad instantánea, que describe qué tan rápida­mente y en qué dirección se está moviendo algo en un instante dado.

A FIGURA 2.5 ¡De vuelta a home! Pese a haber cubierto casi no m entre las bases, en el momento en que el corredor se barre en la caja de bateo (su posición original) para llegar a ¡lOme, su desplazamiento es cero, al menos si es un bateador derecho. Por más rápidamente que haya corrido las bases, su velocidad media para todo el recorrido también es cero.

Noto: En el caso de desplazamientos tanto en la dirección ~ como - (inversión de dirección), la distancia no es la magnitud del desplazamiento total.

2.2 Desplazamiento unidimensiona y velocidad: cantidades vectoriales 31

Page 52: Fisica_I

La velocidad instantánea se define matemáticamente así:

Noto: Lo palabro uniforme significo constante.

~ FIGURA 2.6 Movimiento rectilíneo uniforme: velocidad constante En el movimiento rectilíneo uniforme, un objeto viaja con velocidad constante, cubriendo la misma distancia en íntervalos iguales de tiempo. (a) Aquí, un automóvil recorre SO km cada hora. (b) Una gráfica de'x contra 1 es una línea recta, pues se cubren desplazamientos iguales en tiempos iguales. El yalor numérico de la pendiente de la línea es igual a la magnitud de la velocidad, y el signo de la pendiente da su dirección. (La velocidad media es igual a la \"elocidad ínstantánea en este caso. ¿Por qué?)

Lit v = lím-' (2.4)

..It~o~t

Esta expresión se lee "la velocidad instantánea es igual al límite de ~x/ ~t cuando M se aproxima a cero". El intervalo de tiempo nunca llega a cero (¿por qué?), pero se apro­xima a cero. La velocidad instantánea es una velocidad media, pero en un ~t tan pe­queño que es prácticamente un promedio "en un instante de tiempo", y es por ello que la llamamos velocidad instantánea.

En una dimensión, movimiento uniforme se refiere a un moúmiento con veloci­dad constante (magnitud constante y dirección constante). Por ejemplo, el automóvil de la "t' Fig. 2.6 tiene una \"elocidad uniforme (así como una rapidez uniforme). Recorre la misma distancia en interyalos de tiempo iguales (50 km en cada hora) r la dirección de su movimiento no cambia.

Análisis gráfico El análisis gráfico a menudo es útil para entender el moúmiento y las cantidades rela­cionadas con él. Por ejemplo, el mm"imiento del auto de la Fig. 2.6a podría representar­se en una gráfica de posición contra tiempo, o x contra t. Como se aprecia en la Eg. 2.6b, se obtiene una línea recta para una velocidad uniforme, o constante, en una gráfica así.

Recordemos que en las gráficas cartesianas de y contra x la pendiente de una lí­nea recta está dada por ~Y/ ~x. Aquí, con una gráfica de x contra t, la pendiente de la

Distancia

50 km 100 km 150i<IlJ..n~ [ I o o

o 'o h 2.0 ", 30­

Tiempo

!:"t (k:n) /::,.t (h)

50 1.0 50 km/l.0 h = 50 km/h 100 20 100 km/2.0 h = 50 km/h 150 3.0 150 m/30 = 50 kmjh

(a)

Pe1diente = u (= v)200 O !:..Ix 50 KM Ok Ihend·lente = - = -- = 5 m

!:..It 1.0 r E 150 ­~

·3 el) º 100 X2

c... :> I

i!:"t = X2 - X, = 100 - 50 = 50 kIT'

50 Xl ---- ....I f:..It=t2 -t1 =20-'.0=10h

11 t2IO

1.0 2.0 30 4.0

Tiempo (h)

Velocidad uniforme

(b)

32 CAPíTULO 2 Cinemática: descripción del movimiento

Page 53: Fisica_I

línea, 6.xjM, es igual a la velocidad media v = !1xj 6.t. En movimiento uniforme, este valor es igual a la velocidad instantánea. Es decir, v = v. (¿Por qué?) El valor numérico de la pendiente es la magnitud de la velocidad, y el signo de la pendiente da la direc­ción. Una pendiente positiva indica que x aumenta con el tiempo, así que el movimien­to es en la dirección x positiva. (El signo más suele omitirse, porque se sobreentiende.)

Supongamos que una gráfica de posición contra tiempo para el movimiento de un auto es una línea recta con pendiente negativa, como en la .... Fig. 2.7. ¿Qué indica esta pendiente? Como se aprecia en la figura, los valores de posición (x) disminuyen con el tiempo de manera constante, lo que indica que el auto está viajando con movimiento uniforme en la dirección x negativa.

En la mayor parte de los casos, el movimiento de un objeto no es uniforme, lo que implica que se cubren diferentes distancias en intervalos iguales de tiempo. Una gráfica de x contra t para un movimiento así en una dimensión es una línea curva, como la de la .... Fig. 2.8. La velocidad media del objeto en un intervalo de tiempo dado es la pen­diente de una línea recta que pasa por los dos puntos de la curva que corresponden a los tiempos inicial y final del intervalo. En la figura, la velocidad media para todo el viaje es la pendiente de la línea recta que une los puntos inicial Yfinal de la curva (t¡ y t2).

La velocidad instantánea es igual a la pendiente de una línea recta tangente a la curva en un punto específico. En la Fig. 2.8 se muestran cinco líneas tangentes. En (1), la pendiente es positiva, y el movimiento es en la dirección x positiva. En (2), la pen­diente de una línea tangente horizontal es cero, así que no hay movimiento. Es decir, el objeto se ha detenido instantáneamente (v = O). En (3), la pendiente es negativa, así que el objeto se está moviendo en la dirección x negativa. Así pues, el objeto se detu,·o y cambió de dirección en el punto (2). ¿Qué está sucediendo en los puntos (4) Y(5)?

Si dibujamos diversas líneas tangentes a lo largo de la curva, vemos que sus pen­dientes varían, lo que indica que la wlocidad instantánea está cambiando con el tiempo. Un objeto en movimiento no uniforme puede acelerarse, frenarse o cambiar de direc- Deurminación gráfica de velocidad

T FIGURA 2.8 Gráfica de posición contra tiempo para un objeto en movimiento rectilíneo no uniforme Si la velocidad no es uniforme, una gráfica de x contra t es una curva. La pendiente de la línea entre dos puntos es la velocidad media entre esas dos posiciones, y la velocidad instantánea es la pendiente de una línea tangente a la CULTa en cualquier punto. Se muestran cinco líneas tangentes, y se da el valor !:;.x/ M para la quinta. ¿Puede el lector describir el mo\'imíento del objeto con palabras?

x T FIGURA 2.7 Gráfica de posición contra tiempo para un objeto que se mueve uniformemente en la dirección - x Una linea recta con pendiente negativa en una gráfica .~-----------------------de x contra t indica movimiento uniforme en la dirección -x. Obsérvese que la posición del objeto cambia de forma constante. En t = 4.0 h, el objeto está en x = O. ¿Qué aspecto tendría la gráfica si el movimiento continuara durante t > 4.0 h?

x e

·C

200 P d · -en lente = v= ¡}.X -50 km _. h - =--.­ =-00 Km/

¡}.t 1.0 h ". .¿¿ (j)

C

E CL

::::s 150 Xl e '0 Ü

'(j) 100 x2 O

CL ¡}.t = t2 ­ t1 = 2.0 - 1.0 = 1.0 h

50 ~I.--------tit --------'

O 30 40

TieMpo (h) Tiempo

2,2 Desplazamiento unidimensional y velocidad: cantidades vectoriales 33

Page 54: Fisica_I

Definición de: Aceleración media

Nota: En unidades compuestas, la multiplicación se indica con un punto centrado.

ción. La forma de describir un movimiento con velocidad cambiante es el tema de la sección 2.3.

DI Aceleración OBJETIVOS: (a) Explicar la relación entre velocidad y aceleración y (b) realizar

un análisis gráfico de la aceleración.

La descripción básica del movimiento implica la tasa de cambio de posición con el tiem­po, que llamamos velocidad. Podemos ir un poco más lejos y considerar cómo cambia esa tasa de cambio. Supongamos que algo se está moviendo a velocidad constante y lue­go la velocidad cambia. Semejante cambio de velocidad se denomina aceleración. En un automóviL llamamos acelerador al pedal de la gasolina. Cuando pisamos el acelerador, el auto aumenta su velocidad; si levantamos el pie, el automóvil baja de velocidad. En ambos casos, hay un cambio de velocidad con el tiempo. Definirnos aceleración como la tasa de cambio de la velocidad con el tiempo.

La aceleración media es análoga a la velocidad media; es el cambio de velocidad dividido entre el tiempo que tardó en efectuarse ese cambio:

cambio de velocidadaceleración media = ----------- ­ (2.5)

tiempo que toma el cambio

t1v (i=­

t1t

Unidad sr de aceleración: metros por segundo al cuadrado (m/s2).

Obsérvese que hemos escrito las variables inicial y final con una notación más común. va y to son la velocidad y el tiempo iniciales u originales, respectivamente, y v y t son la velocidad y el tiempo generales en algún momento futuro, cuando queremos conocer la velocidad v después de cierto tiempo t.

Las dimensiones de aceleración son ([L]/[T])/[n de D.v/M. Por tanto, las unida­des sr de aceleración son metros por segundo por segundo, es decir, (m/s)/s o m/s· s, que comúnmente se expresa como metros por segundo al cuadrado (m/52). En el siste­ma inglés, las unidades son pies por segundo al cuadrado (tt/s2

).

Dado que la velocidad es una cantidad vectoriaL también lo es la aceleración, pues representa un cambio de velocidad. Al ser una cantidad vectorial, la velocidad tiene tanto magnitud como dirección, así que un cambio de velocidad podría implicar a cualquiera de estos factores, o ambos. Por tanto, una aceleración podría deberse a un cambio de rapidez (la magnitud), un cambio de dirección o un cambio en ambas cosas, co­mo se muestra en la ~ Fig. 2.9.

En el caso del movimiento rectilíneo, usaremos signos más y menos para indicar las direcciones de velocidad y aceleración, como hicimos con el desplazamiento lineal. La ecuación 2.5 suele simplificarse a:

a = v - va (2.6) t

donde se ha supuesto que to = O. (va podría no ser cero, así que no podemos omitirla en general.)

La aceleración instantánea, análoga a la velocidad instantánea, es la aceleración en un instante dado. Esta cantidad se expresa matemáticamente así:

. ..iv (27)a = lím- . .l~~O ..it

Las condiciones del intervalo de tiempo cercano a cero son las que se describieron pa­ra la velocidad instantánea.

34 CAPíTULO 2 Cinemática: descripción del movimiento

Page 55: Fisica_I

lo 01

I I a • a ~

tAce~e'aC:Ó;1 I )esaceleración I.. .. ~l· (aumenta .'a magnitud) (disminuye la magnitud)

~ ..... ~---+ ~ • ~

~ V = 20 km/h v = 30 k /h v = 40 km/h IV = O II I I

O t = 2.0 s t = 4.0 s 1=60s

(a) Cambio en la magnitud de la velocidad pero no en su dirección

ji 11 o

(b) Cambio en la dirección de la velocidad (e) Cambio en la magnitud y la dirección pero no en su magnitud de la velocidad

... FIGURA 2.9 Aceleración: la tasa de cambio de la velocidad con el tiempo Puesto que la ,-elocidad es una cantidad vectorial, con magnitud y dirección, puede haber una aceleración cuando hay (a) lm cambio de magnitud, pero no de dirección, (b) un cambio de dirección, pero no de magnitud o (e) un cambio tanto de magnitud como de dirección.

Ejemplo 2.3 • Frenado: aceleración media

Un matrimonio viaja en una camioneta a 90 km/h por una carretera recta. Ven lm accidente a lo lejos, así que el conductor baja su velocidad a 40 km/h en 5.0 s. ¿Qué aceleración media tu­vo la camioneta?

Razonamiento. Para calcular aceleración media, se necesitan las variables definidas en la ecuación 2.6, y se han dado.

Solución. Del planteamiento del problema, tenemos estos datos:

0.278 m S)Dado: Vo = (90 kmfl1) ( Hallar: a (aceleración)

1.km-tí = 25 mis

( S)0.278 m

v = (40 kmfl1) 1.km-tí

= 11 mis t = 5.0 s

[En movimiento rectilíneo, suponemos que las velocidades tienen la dirección positiva, y se efectúan de inmediato las conversiones a unidades estándar (kilómetros por segundo a me­

2.3 Aceleración 35

Page 56: Fisica_I

tros por segundo), pues el tiempo se dio en segundos. En general, siempre trabajamos con • aceleración en unidades estándar.] Dadas las velocidades inicial y final y el intelTalo de tiempo, podemos calcular la acele­

ración media con la ecuación 2.6:Aprender dibujándo

v - V 11 mis - 25 mis ,Signos de la velocidad y la a = --o = = -2.8m:s­

t 5.0 saceleración

El signo menos indica la dirección (del vector) de aceleración. En este caso, la dirección es

a,poritÚ!rJ., opuesta a la dirección del movimiento (-i-v), y el automóvil se frena. A veces llamamos desace­leración a una aceleración negativa.

---~-~ irporitÚ!rJ.,

Ejercicio de refuerzo. ¿Una aceleración negativa necesariamente implica que el objeto -x +x en movimiento está desacelerando, o que su rapidez está disminuyendo? Sugerencia: Véase al

margen. (Las respuestas de todos los Ejercicios de refuerzo se dan al fina? dellibro.J

RMu.i:taM:a, lte!JtJ.i:iv¡¡, mM Lei1i:o et&

----,~ w., d¡n"caó~ +XirporitÚ!rJ., Aceleración constante

-x +x Aunque la aceleración puede variar con el tiempo, generalmente restringiremos nues­

.­a,poritÚ!rJ.,

irlte!JatÚ!rJ.,

RMu.i:taM: mM texto et&

w., &ecció~ -x

tro estudio del movimiento a aceleraciones constantes, por sencillez. (Una aceleración constante importante es la debida a la gravedad cerca de la superficie terrestre, que ve­remos en la sección que sigue.) Puesto que en el caso de una aceleración constante el promedio es igual al valor constante (a = a), podemos omitir la raya sobre la acelera­ción en la ecuación 2.6. Así, para una aceleración constante, la ecuación que relaciona

-x +x velocidad, aceleración y tiempo suele escribirse como sigue (reacomodando la ecua­

~ ción 2.6):

a,lte!JatÚ!rJ.,

ir lte!JtJ.i:iv¡¡, v = Vo + at (sólo aceleración constante) (2.8)

-x +x (Cabe señalar que el término at representa el cambio de 'I'elocidad, ya que at v - va = -1v.)

Ejemplo 2.4 • Arranque rápido, detención lenta: movimiento con aceleración constante

Un automóvil para "arrancones" que parte del reposo acelera en línea recta con una tasa constante de 5.5 m/s2 durante 6.0 s. (a) ¿Qué velocidad tiene el vehículo al cabo de ese perio­do? (b) Si en ese momento el carro despliega un paracaídas que lo frena con una tasa unifor­me de 2.4 m/s2

, ¿cuánto tardará en detenerse?

Razonamiento, El ,'ehículo primero acelera y luego frena, por lo que debemos fijamos bien en los signos de dirección de las cantidades vectoriales. Escogemos un sistema de coor­denadas con la dirección positiva en la dirección de la velocidad inicial. (Diagrame la situa­ción.) Entonces podremos obtener las respuestas usando las ecuaciones apropiadas.

Solución. Tomando el movimiento inicial en la dirección positiva, tenemos estos datos:

Dado: (a) Uo = O (en reposo) Hallar: (a) v (wlocidad final)

a = 5.3 In /52 (b) t (tiempo)

t = 6.0 s (b) Vo = [' [de la parte (a)]

u = O (se detiene) 11 = -2.4 m/s2 (dirección opuesta <:)0)'

Hemos presentado los datos en dos partes. Esto ayuda a no confundir los símbolos. Obsérve­se que la velocidad final que se calculará en la parte (a) será la velocidad inicial en la parte (b).

36 CAPíTULO 2 Cinemática: descripción del movimiento

Page 57: Fisica_I

(a) Para obtener la velocidad final, v, usamos la ecuación 2.8 directamente:

v = Vo + at = O -+- (5.5 mN)(6.0 s) = 33m/s

(b) Aquí, queremos el tiempo, así que despejamos t de la ecuación 2.6 y usamos V = 33 miso

de la parte (a) para obtener

v - V o O - 33 mis t = -- = = 14s

a -2.4 m/s2

Obsérvese que el tiempo es positivo, como tendría que ser. Iniciamos implícitamente en el tiempo cero (tomando to = Ocuando se despliega el paracaídas) y el tiempo siempre avan­za, es decir, tiene "dirección positiva".

Ejercicio de refuerzo. ¿Qué velocidad instantánea tiene el carro 10 segundos des­pués de desplegar el paracaídas? (Las respuestas de todos los Ejercicios de refuerzo se dan al final del libro.)

Es fácil representar gráficamente movimientos con aceleración constante grafican­do la velocidad instantánea contra el tiempo. Cna gráfica de v contra t es 1.U1a línea rec­ta cuya pendiente es igual a la aceleración, como se muestra en la ... Fig. 2.10. Obsérvese que la ecuación 2.8 se puede escribir como v = at -,- va' que, como reconocerá el lector, tiene la forma de 1.U1a ecuación de línea recta, y = mx + b (pendiente m y ordenada al origen b). En la Fig. 2.10a, el movimiento es en la dirección positiva, y la aceleración Determinación gráfica de aceleración

T FIGURA 2.10 Gráficas de velocidad contra tiempo para movimientos con aceleración constante La pendiente de una gráfica de v contra t es la aceleración. (a) L'na pendiente positiva indica un aumento de velocidad en la dirección positiva. Las flechas verticales a la derecha indican cómo la aceleración añade velocidad a la velocidad inicial "o' (b) 'Cna pendiente negativa indica una disminución de la velocidad inicial vo' o sea, una desaceleración. (c) Aquí, una pendiente negativa indica una aceleración negativa, pero la velocidad inicial es en la dirección negativa, -vo' así que la rapidez del objeto aumenta en esa dirección. (d) La situación inicial aquí es similar a la de (b), pero termina pareciéndose a la de (c). ¿Puede el lector explicar qué sucedió en el tiempo t]?

-al 'D va + at- v = ro 'D '0 o

1at

~ - V = ve - al

o Tiempo o Tiempo

(a) Movimiento en dirección positiva-acelera (b) Movimiento en dirección positiva-frena

of----------.----.------- ­Tiempo

'D

'D'" -at '[5o OI-__"""""~--___,___,___-L-+-___,------

-al ~ Tiempo -v = vo- al2

_ -[1 = -vo-at

(e) Movimiento en dirección negativa-acelera (d) Cambio de dirección

2.3 Aceleración 37

Page 58: Fisica_I

aumenta la velocidad durante un tiempo t, como indican las flechas verticales a la de­recha de la gráfica. Aqut la pendiente es positiva, -'-a. En la Fig. 2.10b, la pendiente ne­gativa (-a) indica una aceleración negativa que produce un frenado o desaceleración. En cambio, la Fig. 2.10c ilustra cómo una aceleración negati\'a puede aumentar la velo­cidad (cuando el movimiento es en la dirección negativa). La situación en la Fig. 2.10d es un poco más compleja. ¿Puede el lector explicar qué está sucediendo ahí?

Cuando un objeto se mueve con aceleración constante, su velocidad cambia en la misma cantidad en cada unidad de tiempo. Por ejemplo, si la aceleración es de 10 mi S2

en la misma dirección que la velocidad iniciat la velocidad del objeto aumentará en 10 mis cada segundo. Supongamos que el objeto tiene una velocidad inicial Vo de 20 mis en to =O. Entonces, para t = O, 1.0, 2.0, 3.0 Y4.0 s, las velocidades son 20, 30,40,50 Y60 mis, respectivamente. La velocidad media en el intervalo de cuatro segundos es v = 40m/s.

Esta velocidad media podría calcularse de la forma acostumbrada (Ecuación 2.3), pero también podríamos reconocer de inmediato que la serie uniformemente creciente de números 20, 30, 40, 50 Y60 tiene un valor medio de 40 (el valor que está en el punto medio de la serie). Cabe señalar que el promedio de los valores extremos (inicial y fi­nal) también da el promedio de la serie; es decir, (20 + 60)/2 = 40. Cuando la velocidad cambia de manera uniforme debido a una aceleración constante, v es el promedio de las velocidades inicial y final:

_ v + Vov=--- (sólo aceleración constante) (2.9)2

Ejemplo 2.5 • En el agua: uso de múltiples ecuaciones

Una lancha de motor que parte del reposo en un lago acelera en línea recta con una tasa cons­tante de 3.0 m/s2 durante 8.0 s. ¿Qué distancia recorre en ese tiempo?

Razonamiento. Sólo tenemos una ecuación para distancia (Ecuación 2.3, x = .lo -'- vi), pe­ro no podemos usarla directamente. Primero debemos calcular la velocidad media, así que necesitaremos múltiples ecuaciones.

Solución. Después de leer el problema y resumir los datos y lo que se pide, tenemos:

Dado: .lo = O Hallar: x (distancia) Vo = O

a = 3.0 m/s2

I = 8.0 s

(Observe que todas las unidades son estándar.) Al analizar el problema, podríamos razonar como sigue: para obtener x, tendremos que

usar la ecuación 2.3, .lo + vi. (Debemos usar la velocidad media v porque la velocidad está cambiando, así que no es constante.) Como se nos dio el tiempo 1, ya sólo nos falta obtener v. Por la ecuación 2.9, v = (v + vo )/2, y, con Vo =Osólo necesitamos la velocidad final v para re­solver el problema. La ecuación 2.8, v =Vo + al, nos permite calcular v a partir de los datos. Así pues, tenemos:

La velocidad de la lancha al término de 8.0 s es

v = Vo -r al = O - (3.0 m/s2)(8.0s) = 24 mIs

La velocidad media en ese intervalo de tiempo es

v + V 24 mis O-Í­

V =--o = . = 12 mIs 2 2

Por último, la magnitud del desplazamiento, que en este caso es igual a la distancia recorrida, está dada por la ecuación 2.3 (con .lo = O):

x = vi = (12 m/s)(8.0 s) = 96 m

38 CAPíTULO 2 Cinemática: descripción del movimiento

Page 59: Fisica_I

Ejercicio de refuerzo. (Avance.) En la sección 2.4 deduciremos la siguiente ecuación: x = vot + ~at2. Utilice los datos de este ejemplo para ver si esta ecuación da la distancia reco­rrida. (Las respuestas de todos los Ejercicios de refuerzo se dan al final dellíbro.)

DI Ecuaciones de cinemática (aceleración constante)

OBJETIVOS: (a) Explicar las ecuaciones de cinemática para aceleración cons­tante y (b) aplicarlas a situaciones ñsicas.

Sólo necesitamos tres ecuaciones básicas para descnbir los movimientos en una di­mensión con aceleración constante. En las secciones anteriores vimos que esas ecuacio­nes son

x = Xo + Vt (2.3)

- v+vov=--- (sólo aceleración constante) (2.9)2

v = Vo + at (sólo aceleración constante) (2.8)

(Cabe señalar que la primera ecuación, ecuación 2.3, es general y no está limitada a sí­tuaciones de aceleración constante, como las otras dos ecuaciones.)

Sin embargo, como vimos en el ejemplo 2.5, la descripción del movimiento en al­gunos casos requiere aplicar varias de estas ecuaciones, lo cual podria no ser obvio. Sería útil reducir el número de operaciones que deben efectuarse para resolver problemas de cinemática, y podemos lograrlo combinando ecuaciones algebraícamente.

Por ejemplo, supongamos que queremos una expresión que dé la ubicación x en términos del tiempo y la aceleración, no en términos del tiempo y la velocidad media (como en la ecuación 2.3). Podemos eliminar v de la ecuación 2.3 sustituyendo v de la ecuación 2.9 en la ecuación 2.3:

V + v)x = Xo + VI: = Xo + ( ~ t

y

x = Xo + ~(v + vo)t (sólo aceleraóán constante) (2.10)

Entonces, al sustituir v de la ecuación 2.8 obtenemos

x = Xo + ~(vo + at + vo)t Al simplificar,

x = Xo + vat + ~af (sólo aceleración mnstante) (2.11)

En esencia, realizamos esta serie de pasos en el ejemplo 2.5. La ecuación combinada permite calcular directamente la distancia recorrida por la lancha de ese ejemplo:

x - Xo = .!lx = vot + ~af = O + h3.0 m/s2)(8.0 S)2 = 96 m

Es mucho más fácil, ¿no? Podriamos querer una expresión que dé la velocidad en función de la posición x,

no del tiempo (como en la ecuación 2.8). Podemos eliminar t de la ecuación 2.8 usando la ecuación 2.3 en la forma t = (x - xo)/v:

( x)X ­

v = Vo +a T

Nota: Áx = x - Xo es desplazamiento, pero con xo = o, corno suele ser, Áx = X, y el valor de Jo posición x es el mismo que el del desplazamiento. Esto nos ahorra tener que escribir siempre Áx = x - Xo '

2.4 Ecuaciones de cinemática (aceleración constante) 39

Page 60: Fisica_I

Entonces sustituimos V, utilizando la ecuación 2.9:

2a(x - xo ) o o

v + v

Simplificamos utilizando la relación algebraica (v + [10)( v - va) v2 - v~, para ob­

tener

(sólo aceleración constante) (2.12)

Sugerencia para resolver problemas

Los estudiantes de cursos de introducción a la física a veces se sienten abrumados por las di­versas ecuaciones de cinemática. 'o hay que olvidar que las ecuaciones y las matemáticas son las herramientas de la física. Todo mecánico o carpintero sabe que las herramientas facilitan el trabajo si uno las conoce y sabe usarlas. L.o mismo sucede con las herramientas de la física.

En las secciones 2.3 y 2.4 hemos presentado las ecuaciones de cinemática. Usamos el conjunto siguiente de ecuaciones para movimiento rectilineo con aceleración constante pa­ra resolver la mayor parte de los problemas de cinemática:

v = va + at (2.8)

(2.10)

(2.11)

(2.12)

(Ocasionalmente, nos interesará una rapidez o velocidad media pero, como ya señalamos, los promedios generalmente no nos dicen mucho.) Obsérvese que todas las ecuaciones de la lista tienen cuatro o cinco variables. Es preciso conocer todas las variables de una ecua­ción, menos una, para poder calcular lo que nos interesa. Es decir, para despejar y calcular una variable incógnita de la ecuación 2.8, hay que conocer las otras tres, y en las demás ecuaciones hay que conocer las otras cuatro.

Siempre hay que tratar de entender y visualizar los problemas. Una lista de los datos, como la que se describe en el procedimiento para resolver problemas sugerido en el capítu­lo 1, podría ayudamos a decidir qué ecuación usar, pues nos indica las variables conocidas y las incógnitas. Recuerde esta estrategia al resolver los demás ejemplos de este capítulo. También es importante no pasar por alto datos implícitos, error que ilustra el ejemplo 2.6.

Ejemplo integrado 2.6 • Separación: ¿dónde están ahora?

Dos pilotos de carritos están separados por 10 m en una pista larga y recta, mirando en direc­ciones opuestas. Ambos parten al mismo tiempo y aceleran con una tasa constante de 2.0 miS2.

(a) Haga un diagrama de la situación para resolver problemas. (b) ¿Qué separación tendrán los carritos al término de 3.0 s?

(a) Razonamiento conceptual. Sólo sabemos que los carritos tienen una separación ini­cial de 10 m, así que popemos colocarlos en cualquier punto del eje x. Nos conviene colocar uno en el origen para que una posición inicial (xo) sea cero. En la ~ Fig. 2.11 se muestra un dia­grama de la situación.

(b) Razonamiento. El diagrama nos indica que tenemos estos datos:

Dado: xo, = O Hallar: separación en t = 3.0 s

a~ = -2.0 m/s2

t = 3.0 s

x", = 10 m

aB = 2.0 m/s2

40 CAPíTULO 2 Cinemática: descripción del movimiento

Page 61: Fisica_I

- .......-----"".~ +

A B

I I I I 1 I

x=O x=10m I I Separación I I IL.o....__ inicial __""",.~I I

II = 10 rl I

I

.----------------- Separación final = ?----------------..~I1

I

A FIGURA 2.11 ¡Allá van! Dos carritos aceleran en direcciones opuestas. ¿Qué separación tienen en un momento posterior? Véase el Ejemplo integrado 2.6.

La distancia que cada vehículo recorre está dada por la ecuación 2.11 [la única ecuación de desplazamiento (x) que incluye la aceleración (a)]: x = .lo + vot -'- ~af. Pero hay un proble­ma: Va no está en la lista "Dado". Quizá pasamos por alto algún dato implícito. Nos damos cuenta de que va = Opara ambos \'ehículos, así que:

x~ = .lo, -+- '-'o.t + ia\t2 = O -¡- O ..,. ~(- 2.0 m/s2)(3.0 S)2 = -9 m

y .lB = .lo, + vo./ + iaBf = 10 m ~ O - i(2.0 m/s2)(3.0 S)2 = 19 m

¿Qué nos dice x\ = -9 m? Que el \'ehículo A está en la posición -9 m respecto al origen, so­bre el eje - x, mientras que el \'ehículo B está en la posición 19 m sobre el eje..,. x. Por tanto, la separación entre los dos carritos es de 28 m.

Ejercicio de refuerzo. ¿Sería diferente la separación si hubiéramos tomado la posición inicial del vehículo B como el origen, en lugar de la del \'ehículo A? (Las respuestas de todos los Eiercicios de refuerzo se dan al final del libro.)

Ejemplo conceptual 2.7 • Dos autos de carreras: el efecto de cantidades al cuadrado

Durante unas pruebas contra reloj, el carro A, que parte del reposo, acelera de manera unifor­me por una pista recta y plana durante cierto tiempo. El carro B, que también parte del repo­so, acelera con la misma tasa, pero durante el doble de tiempo. Al término de sus respecti\'os periodos de aceleración, ¿qué afirmación es cierta? (a) El carro A recorrió una distancia ma­yor; (b) el carro B recorrió el doble de la distancia que el carro A; (c) el carro B recorrió cuatro veces la distancia que el carro A; (d) ambos carros han recorrido la misma distancia. Plantee claramente el razonamiento y los principios defísica que usó para llegar a su respuesta, antes de leer el párrafo siguiente. Es decir, ¿por qué escogió esa respuesta?

Razonamiento y respuesta. :\:os dicen que Vo = O, Yla aceleración es la misma para am­bos carros. Para calcular las distancias recorridas en un tiempo t, usaríamos la ecuación 2.11 que, con va = OYtomando Xo = O, se con\'ierte en x = ~ at"' Lo importante aquí es obserTar que la distancia aumenta en función de t2 Es decir, si aumentamos el tiempo al doble, la distancia se cuadruplica (es decir, aumenta en un factor de 4). '.

En este ejemplo, el carro B acelera el doble de tiempo que el carro A, o sea que ts =21.\, de modo que el carro B recorre cuatro veces la distancia que recorre el A, y la respuesta es (c). La expresión matemática de la solución es

x.\ = ~at~ \' Xs = ~at§ = ~a(2tS = ~a( 4t~) = 4(iat~) = 4x\

Compare el recorrido de los carros si tB =3tA.

Ejercicio de refuerzo. Compare la rapidez de los dos carros al término de su periodo de aceleración. (Las respuestas de todos los Ejercicios de refuerzo se dan al final del libro.)

2.4 Ecuaciones de cinemática (aceleración constante) 41

Page 62: Fisica_I

Ejemplo 2.8 • Frenado: distancia en que un vehículo para

La distancia en que W1 vehículo para es W1 factor importante para la seguridad en las carrete­ras. Esta distancia depende de la velocidad inicial (+"0) Yde la capacidad de frenado, o desa­celeración, -a, que suponemos constante. (Recordemos que el signo menos indica que la aceleración es en la dirección negativa. En este caso, el signo de la aceleración es opuesto al de la velocidad, que suponemos positivo. Así pues, el \"ehículo disminuye su \'elocidad hasta pa­rar.) Exprese la distancia de detención x en términos de estas cantidades.

Razonamiento. Una vez más, necesitamos W1a ecuación de cinemática, y W1a lista de lo que se da y lo que se pide indica cuál es la apropiada. Se nos pide W1a distancia, y no intervie­ne el tiempo.

~ FIGURA 2.12 Distancia en que po a un vehiculo Díbujo para - .......-------. + visualizar la situación del ejemplo 2.8. vo Autorróvil detenido

v=O....­~ ---- f2:;;;;.:::-. -=~

¡..I·------x=7-----------1.1 xo =O (Distancia de detención)

Solución. Estamos trabajando con variables, así que sólo podemos representar las cantida­des en forma simbólica.

Dado: V o (dirección positiva) Hallar: x (en términos de las -a (dirección opuesta de vol \-ariables dadas) v = O (el automóvil se detiene) X = O (el origen es la posición inicial del auto)o

Aquí también ayuda diagramar la situación, sobre todo porque intervienen cantidades vecto­riales (.Fig. 2.12). (Dado que la ecuación 2.12.) Tiene las \"ariables que queremos, nos deberá permitir encontrar la distancia de detención x. Si expresamos la aceleración negativa explíci­tamente y suponemos Xo = O, tendremos

7l- = é~ -'-- 2( -a)x = I~ - 2ax

Puesto que el vehículo se para (v =O), podemos despejar x:

v;x =­

2n

Esta ecuación nos da x en términos de la velocidad inicial del vehículo y la aceleración de fre­nado.

Observemos que la distancia de detención x es proporcional al cuadmdo de la rapidez ini­cial. Por tanto, si la rapidez inicial es el doble, la distancia de detención aumentará en W1 fac­tor de 4 (con la misma desaceleración). Es decir, si la distancia de detención es Xl con W1a velocidad inicial de V¡, con W1 aumento del doble en la \"elocidad inicial (U2 = 2V1) la distancia de detención aumentará cuatro veces:

ui Y. =­.~ 1

2a

if, (2v1)2 (~'~) X2 = 2~ = ---;;- = -± 2,~ = -le)

Podemos obtener el mismo resultado usando cocientes directamente:

X2 v~

x) d

42 CAPíTULO 2 Cinemática: descripción del movimiento

Page 63: Fisica_I

¿Será importante esta consideración para fijar límites de velocidad, digamos en zonas es­colares? (También habría que considerar el tiempo de reacción del conductor. En la sección 2.5 se da un método para aproximar el tiempo de reacción de una persona.)

Ejercicio de refuerzo. Las pruebas han demostrado que el Chevy Blazer tiene una desa­celeración media de frenado de 7.5 m/s2

, mientras que la de un Toyota Celica es de 9.2 m/s2

Supongamos que dos de estos vehículos se están conduciendo por un camino recto y plano a 97 km/h (60 mi/h), con el Celica adelante del Blazer. Un gato cruza el camino frente a ellos, y ambos conductores aplican los frenos al mismo tiempo y se detienen sin percance (sin arrollar al gato). Suponiendo que ambos conductores tienen el mismo tiempo de reacción, ¿a qué dis­tancia minima debe ir el Blazer del Celica para que no choque con éste cuando los dos vehícu­los se detienen? (Las respuestas de todos los Ejercicios de refuerzo se dan al final del libro.)

Análisis gráfico de ecuaciones de cinemática Como se mostró en la Fig. 2.10, las gráficas de v contra t dan una línea recta cuya pen­diente es la aceleración constante. Las gráficas de v contra t tienen otro aspecto intere­sante. Consideremos la que se muestra en la .Fig. 2.13a, en especial el área sombrada bajo la curva. Supongamos que calculamos el área del triángulo sombreado donde, en general, A = ~ab [área = ~(altura)(base)].

En la gráfica de la Fig. 2.13a, la altura es v y la base es t, así que A = ~vt. Por la ecuación v = va + at, tenemos v =at, donde Uo =O(la recta pasa por el origen). Por tanto,

, ¡ ¡ ,2A = ~vt = :2(at)t = la, = 6x = x

(La ecuación x = ~at2 surge de la ecuación 2.11 cuando va = OYXo = O, con el objeto ini­cialmente en el origen.) Por tanto x, la distancia cubierta, es igual al área bajo una cur­va de v contra t.

Examin.emos ahora la Fig. 2.13b. Aquí, va tiene un valor distinto de cero en t = O, o sea que el objeto ya se está moviendo. Consideremos las áreas sombreadas. Sabemos que el área del triángulo es A 2 = ~at2, y puede verse que la del rectángulo es (con Xo =

O) A¡ = vot. Si sumamos estas áreas para obtener el área total, tenemos

La ecuación 2.11 está a la derecha, así que también en este caso x, la distancia cubierta, es igual al área bajo la cunTa de v contra t.

ID Caída libre OBJETIVO: Usar las ecuaciones de cinemática para analizar la caída libre.

Uno de los casos más comunes de aceleración constante es la aceleración debida a la gravedad cerca de la superficie terrestre. Cuando dejamos caer un objeto, su velocidad inicial (en el momento en que se suelta) es cero. En un momento posterior, mientras cae, tiene una velocidad distinta de cero. Ha habido un cambio en la velocidad y, por tanto (por definición), una aceleración. Esta aceleración debida a la gravedad (g) tiene una magnitud aproximada de

aceleración debida a la gravedad

(o sea, 980 cm/s2) y está dirigida hacia abajo (hacia el centro de la Tierra). En unidades inglesas, el valor de g es de aproximadamente 32.2 ft/s2

Los \'alores que damos aquí para g son aproximados porque la aceleración debida a la gravedad varía un poco en los diferentes lugares como resultado de diferencias en la altura sobre el nivel del mar y en la densidad media regional de masa de la Tierra. En este libro haremos caso omiso de esas pequeñas variaciones a menos que se indique

v

Tiempo (b)

Á FIGURA 2.13 Gráficas de v contra t, otra vez (a) En la recta de aceleración constante, el área bajo la ClinTa es igual a x, la distancia recorrida. (b) Aunque V no sea cero, o

la distancia está dada por el área bajo la Clin!a, que aquí se dividió en dos partes, las áreas Al y A2.

2.5 Caída libre 43

Page 64: Fisica_I

otra cosa. La resistencia del aire es otro factor que afecta la aceleración de un objeto que cae, pero también haremos caso omiso aquí de ella, por sencillez.

Decimos que los objetos que se mueven únicamente bajo la influencia de la gravedad están en caída libre. Las palabras "caída libre" nos hacen imaginar objetos que se han dejado caer y se mueven hacia abajo bajo la influencia de la gravedad (g = 9.80 miS2 en ausencia de resistencia del aire), pero el término se puede aplicar en general a cualquier movi­miento vertical bajo la influencia exclusiva de la gravedad. los objetos que se sueltan desde el reposo o que se lanzan hacia arriba o hacia abajo están en caída libre una vez que se sueltan. Es decir, después de t = O(el momento de liberación), sólo la gravedad influye en el movimiento. (Incluso cuando un objeto proyectado hacia arriba está as­cendiendo, está acelerando hacia abajo.) Por tanto, podemos usar el conjw1to de ecuacio­nes para movimiento en una dimensión, para describir la caída libre generalizada.

La aceleración debida a la gravedad, g, es la aceleración constante de todos los ob­jetos en caída libre, sin importar su masa o peso. Antes se pensaba que los cuerpos más pesados caen más rápidamente que los más ligeros. Este concepto formó parte de la teoría del movimiento de Aristóteles. Es fácil observar que una moneda cae más rápi­damente que una hoja de papel cuando se dejan caer simultáneamente de la misma al­tura. Sin embargo, en este caso la resistencia del aire es muy importante. Si el papel se aprieta hasta formar una bolita compacta, dará más batalla a la moneda. Asimismo, una pluma "flota" hacia abajo mucho más lentamente que una moneda que cae. o obstan­te, en un vacío aproximado, donde la resistencia del aire es insignificante, la pluma y la moneda caen con la misma aceleración: la aceleración debida a la gra,'edad (... Fig. 2.14).

El astronauta David Scott realizó un experimento similar en la Luna en 1971, de­jando caer simultáneamente una pluma y un martillo desde la misma altura. :\:'0 nece­sitó una bomba de vacío: la luna no tiene atmósfera y por tanto no hay resistencia del aire. El martillo y la pluma tocaron la superficie lunar juntos, pero ambos cayeron más lentamente que en la Tierra. la aceleración debida a la gra\'edad cerca de la superficie lunar es aproximadamente la sexta parte de la que experimentamos cerca de la super­ficie terrestre (g,,¡ ;::::; gI6).

Las ideas que gozan actualmente de aceptación en lo tocante al movimiento de cuerpos que caen se deben en gran medida a Galileo. Él puso en duda la teoría de iuis­tóteles e investigó experimentalmente el mm'imiento de los objetos. Según la leyenda, Galileo estudió la aceleración de cuerpos que caen dejando caer objetos de diferente peso desde lo alto de la Torre de Pisa. (Véase el ensayo A fondo sobre Galileo.)

~ FIGURA 2.14 Caída libre y resistencia al aire (a) Cuando se dejan caer simultáneamente de la misma altura, una pluma cae más lentamente que una moneda, por la resistencia del aire. En cambio, cuando ambos objetos se dejan caer en un recipiente en el que se ha hecho un buen vacío parcial, en el que la resistencia del aire es insignificante, la pluma y la moneda caen juntas con aceleración constante. (b) Demostración real con fotografía de destello múltiple: una manzana y una pluma se sueltan simultáneamente a través de un escotillóYl en una cámara de \'acío grande, y caen juntos... o casi. Dado que el vacío es sólo parcial, todada hay cierta resistencia del aire. (¿En qué se nota?)

(a) (b)

44 CAPíTULO 2 Cinemática: descripciór del movimiento

Page 65: Fisica_I

A FONDO

La torre inclinada de Pisa: un ejercicio en estabilidad

La torre inclinada de Pisa es un famoso monmnento italiano en el que supuestamente Galileo realizó experimentos. (Véase el ensa­yo del capítulo 2 en la p. 51.) Hace poco, la torre ha estado en las noticias por los esfuerzos que se han hecho para evitar que se caiga. La torre comenzó a inclinarse antes de que terminaran de construirla en 1350, debido a lo blando del subsuelo. Se cerró al público en 1990, cuando tenía una inclinación de 5.5" respecto a la vertical (unos 5 m en la parte más alta) y un incremento medio anual de la inclinación de 1.2 mm.

Se inyectó cemento en la base en la década de 1930, pero la inclinación siguió amnentando. En la última década, se inició un esfuerzo internacional para salvar la torre. Entre 1993 y 1995, se construyó un anillo de concreto en la base y se colocaron sobre él 620 toneladas de plomo como contrapeso (Fig. 1). En cuestión de meses, la inclinación se corrigió en 2-3 cm. Sin embargo, la cons­trucción de un segundo anillo sacudió a la torre, y en una sola noche se inclinó 2.5 mm. Después de ese susto, se añadieron otras 230 toneladas de plomo al contrapeso.

Se logró una mejora importante extrayendo suelo de abajo del lado alto de la torre. Taladros penetraron diagonalmente ba­jo los cimientos, creando cavidades pequeñas de las cuales se extrajo suelo. Esto permitió a la torre enderezarse un poco al asentarse (y mantener el centro de gravedad más adentro de la

FIGURA 1 ¡Estabilícenla! Se usaron toneladas de plomo como contrapeso para corregir la inclinación de la torre.

base de soporte). Por si acaso, se sujetaron cables de acero a la to­rre para sostenerla en caso de haber un desplazamiento en la di­rección equivocada (Fig. 2). Gracias a las perforaciones, la torre regresó a una inclinación de unos 5", lo que correspondió a un desplazamiento de 40 = en la parte más alta.

La torre inclinada de Pisa abrió otra vez sus puertas a los turistas en diciembre de 2001, después de 12 años de labores pa­ra reducir la inclinación. Antes de que la torre se cerrara en 1990, cerca de un millón de visitantes trepaban cada año la escalera de caracol de 293 escalones hasta el piso más alto. Ahora, sin em­bargo, esto se limitará a grupos de 30 personas en una \'isita guiada de 40 minutos. Las filas y la espera van a ser muy largas.

Arena Arcilla

Arena comprimida

FIGURA 2 Mantener el centro de gravedad sobre la base de soporte. Se buscó estabilizar la torre extrayendo suelo bajo el lado alto. Por seguridad en caso de desplazamientos repentinos, se sujetaron cables de acero a la torre.

2.5 Caída libre 45

Page 66: Fisica_I

Se acostumbra usar y para representar la dirección \-ertical y considerar positivo hacia arriba (como en el eje y vertical de las coordenadas cartesianas). Dado que la ace­leración debida a la gravedad siempre es hacia abajo, está en la dirección y negativa.

Nota: Al subir (+ v), un objeto lanzado Esta aceleración negativa, a = -g = -9.80 m/s2, se sustituye en las ecuaciones de movi­

está acelerando hacia abajo, es decir, miento, aunque la relación a = -g se puede expresar explícitamente en las ecuaciones tiene una aceleración en la dirección hacia abajo (-g). de movimiento rectilíneo, por conveniencia:

! ¡ -,_.~~-,,-I

II(1N, : I ,

¡, InJf3rR~:g1 ' ,,®= -pR,'-tC'lrCrr-"I'.~. i~tt~--

¡ 1 ! I M

Caída libre

v = V - gt (2.8')o

Ecuaciones de caída libre con (2.11')- g expresada explícitamente

(2.12')

La ecuación 2.9 también es válida, pero no contiene a g:

(2.10')

Por lo regular se toma el origen (y = O) del marco de referencia como la posición inicial del objeto. Puesto que casi siempre la dirección positi\'a es hacia arriba (eje +y de las gráficas), el hecho de escribir explícitamente -g en las ecuaciones nos recuerda las di­ferencias de dirección, e ínsertamos el valor de 9.80 m/s2 en \'ez de g. No obstante, la decisión es arbitraria. Las ecuaciones se pueden escribir con a =g; por ejemplo, v =V o

-r gt, asociando el signo menos directamente a g. En este caso, sustituiremos siempre -9.80 m/s2 por g.

Siempre debemos indicar explícitamente las direcciones de las cantidades vecto­riales. La posición y y las velocidades v y V o podrían ser positivas o negativas, depen­diendo de la dirección del movimiento. El uso de estas ecuaciones y la convención del signo se ilustran en los ejemplos que siguen.

Ejemplo 2.9 • Piedra lanzada hacia abajo: repaso de ecuaciones de cinemática

Un niño parado en W1 puente lanza W1 piedra verticalmente hacia abajo con W1a velocidad inicial de 14.7 mis, hacia el río que pasa por abajo. Si la piedra choca con el agua 2.00 s des­pués, ¿a qué altura está el puente sobre el agua?

Razonamiento. Es W1 problema de caída libre, pero hay que observar que la yelocidad ini­cial es hacia abajo, o negativa. Es importante expresar explícitamente este hecho.

Solución. Como siempre, primero escribimos lo que nos dan y lo que nos piden:

Dado: va = -14.7 mis (se toma hacia abajo Hallar: y (altura)

t = 2.00 s como dirección negativa) g (= 9.S0 m/s2

)

ObséI';ese que g se toma como número positivo, porque el signo menos direccional ya se in­cluyó en las ecuaciones,de moyimiento anteriores. Con el tiempo, es probable que el lector simplemente escriba el símbolo g, pues ya conocerá su valor numérico. En esta ocasión, con­Yiene hacer W1 dibujo para analizar la situación.

¿Qué ecuación(es) da(n) la solución con los datos proporcionados) Debe ser obvio que la distancia que la piedra recorre en W1 tiempo t está dada directamente por la ecuación 2.11' (tomando Yo =O):

y = '-'ot - tgt2 = (-14.7 mis) (2.00 s) - h9.S0m s1)(2.00s)2

= -29.4 m - 19.6 m = -49.0 m

El signo menos indica que el desplazamiento es hacia abajo, lo cual coincide con lo que sabe­mos por el planteamiento del problema. Así pues, la altura es 49.0 m.

46 CAPíTULO 2 Cinemática: descripción del movimiento

Page 67: Fisica_I

Ejercicio de refuerzo. ¿Cuánto más tardaría la piedra de este ejemplo en tocar el agua si el niño la hubiera dejado caer en vez de lanzarla? (Las respuestas de todos los Ejercicios de refuer­zo se dan al final del libro.)

Ejemplo 2.10 • Medición del tiempo de reacción: caída libre

El tiempo de reacción es el tiempo que necesita una persona para notar, pensar y actuar en res­puesta a una situación; por ejemplo, el tiempo que transcurre entre que se observa por prime­ra vez una obstrucción en el camino cuando se conduce un automóvil, y se responde a ella. El tiempo de reacción varía con la complejidad de la situación (y con el individuo). En general, la mayor parte del tiempo de reacción de una persona se dedica a pensar, pero la práctica en el manejo de una situación dada puede reducir ese tiempo.

El tiempo de reacción de una persona puede medirse pidiendo a otra persona que deje caer una regla (sin previo aviso) cuya base está a la altura del pulgar y el indice de la primera persona, y entre ellos, como se muestra en la ~ Fig. 2.15. La primera persona sujeta lo antes po­sible la regla que cae, y se toma nota de la longitud de la regla que queda por debajo del dedo superior. Si, en promedio, la regla desciende 18.0 cm antes de ser atrapada, ¿qué tiempo de reacción medio tiene la persona?

Razonamiento. Intervienen tanto la distancia como el tiempo. Esta observación indica la ecuación de cinemática que debe usarse.

Solución. Observamos que sólo se da la distancia de caída. Sin embargo, sabemos algunas cosas más, como va y g, así que, tomando Yo = O:

Dado: y = -18.0 cm = -0.180 m Hallar: t (tiempo de reacción)

va = O

g (= 9.80 m/s2)

(Obsérvese que la distancia Y se convirtió directamente en metros.) Vemos que la ecuación pertinente es la ecuación 2.11', que da

o sea

Despejando t,

2y = j2( -0.180 ~) = 0.192 st = -g \ -9.80 mis

Pruebe este experimento con un compañero y mida su tiempo de reacción. ¿Por qué cree que debe ser otra persona la que deje caer la regla?

Ejercicio de refuerzo. Un truco popular consiste en usar un billete nuevo de dólar en lugar de la regla de la Fig. 2.15, Ydecir a la persona que puede quedarse con el billete si lo pue­de atrapar. ¿Es buen negocio la propuesta? (La longitud de un billete de dólar es de 15.7 cm.) (Las respuestas de todos los Ejercicios de refuerzo se dan al final del libro.)

Ejemplo 2.11 • Caída libre hacia arriba y hacia abajo: uso de datos implícitos

Un trabajador parado en un andamio junto a una valla lanza una pelota ,'ertica1mente hacia arriba. La pelota tiene una velocidad inicial de 11.2 mis cuando deja la mano del trabajador en la parte más alta de la \'alla (~Fig. 2.16). (a) ¿Qué altura máxima alcanza la pelota sobre la valla? (b) ¿Cuánto tarda en llegar a esa altura? (c) ¿Dónde está la pelota en t = 2.00 s?

Razonamiento. En (a), sólo hay que considerar la parte ascendente del movimiento. La pe­lota se detiene (velocidad cero) en la altura máxima, lo que nos permite determinar esa altu­ra. (b) Conociendo la altura máxima, podemos determinar el tiempo de ascenso. En (c), la ecuación distancia-tiempo (Ecuación 2.11') es válida para cualquier tiempo y da la posición (y) de la pelota relativa al punto de lanzamiento.

.& FIGURA 2.15 Tiempo de reacción El tiempo de reacción de una persona puede medirse pidiéndole que sujete una regla que se deja caer. Véase el ejemplo 2.10.

2,5 Caída libre 47

Page 68: Fisica_I

v=o - - - - - - - - c·.-.-:- - - - - y =Ymáx

!/máx

~ FIGURA 2.16 Caída libre hacia arriba y hacia abajo Obsérvese la longitud de los \'ectores de velocidad y aceleración en diferentes tiempos. (Las trayectorias ascendente y descendente de la pelota se han desplazado horizontalmente para hacer más clara la ilustración.) Véase el ejemplo 2.11.

------::1;0'-------1

Q]

Solución. Podría parecer que lo único que se da en el problema general es la velocidad ini­cial Vo' Sin embargo, se sobreentiende un par de datos más. Uno es la aceleración, g, y el otro es la velocidad en la altura máxima, donde la pelota se detiene. Aquí, al cambiar de dirección, la \'elocidad de la pelota es momentáneamente cero, así que tenemos (tomando Yo = O):

Dado: Vo = 11.2 mis Hallar: (a) !/má' (altura máxima)

g (= 9.80 mjsl) (b) ta (tiempo de ascenso)

v = O (enYmá.J (c) y (en t = 2.00 s)

t = 2.00 s [para la parte (c)]

(a) Referimos la altura a la parte más alta de la \'aila (yo = O). En esta parte del problema só­lo nos ocupamos del mo\'irniento ascendente: se lanza una pelota hacia arriba y se detiene en su alhtra máxima !lIT.á" Con t' = Oa esta altura, podemos obtener !lm¿x directamente de la ecua­ción 2.12':

Así que,

v~ (11.2 m/s)2 !lmá.' = - = = 6.40 m

2g 2(9.80 m S2)

relati\'a al borde superior de la \'alla (yo = O; \'éase la Fig. 2.16). (b) Llamamos ta al tiempo en que la pelota sube. Éste es el tiempo que la pelota tarda en al­canzar !lmá" donde [' = O. Puesto que conocemos Vo y v, podemos obtener el tiempo t~ directa­mente de la ecuación 2.?,:

Entonces,

l'o 11.2 m s t~ = --; ---- = 1.14 s

ó 9.80 m S2

(e) La altura de la pelota en t = 2.00 s está dada directamente por la ecuación 2.11':

!I = Pot - ~g¡2

= (11.2 m/s)(2.00 s) - ~(9.80 mjsl)(2.00 S)2 = 224 m - 19.6 m = 2.8 m

48 CAPíTULO 2 Cinemática: descripción de movimiento

Page 69: Fisica_I

Cabe señalar que esta altura de 2.8 m se IlÚde hacia arriba desde el punto de referencia % = O). La pelota alcanzó su altura máxima y viene de bajada.

Considerada desde otro punto de referencia, la situación se parece a la de dejar caer una pelota desde una altura de !/má.' sobre la valla con Vo = OYpreguntar qué distancia cae en un tiempo t =2.00 5 - t. =2.00 5 - 1.14 s =0.86 s. La respuesta es (con Yo =Oen la altura máxima):

y = vot - ~gt2 = O - ~(9.80 mjs2)(0.86 5)2 = -3.6 m

Esta altura es la IlÚsma que la posición que obtuvimos antes, sólo que se IlÚde con respecto a la altura máxima como punto de referencia; es decir,

Ymáx - 3.6 m = 6.4 m - 3.6 m = 2.8 m

Ejercicio de refuerzo. ¿A qué altura la pelota de este ejemplo tiene una rapidez de 5.00 mis? (Sugerencia: La pelota alcanza esta altura dos veces, W1a de subida y otra de baja­da.) (Las respuestas de todos los Ejercicios de refuerzo se dan al final del libro.)

He aquí un par de hechos interesantes relacionados con el movimiento vertical de proyectil de un objeto lanzado hacia arriba en ausencia de resistencia significativa del aire. Primero, los tiempos de ascenso y descenso son iguales. Es decir, el tiempo que el objeto tarda en alcanzar su altura máxima es el mismo que el que tarda en caer de la altura máxima hasta el punto de partida. Recordemos que, en la cúspide de la trayec­toria, la velocidad del objeto es cero durante un instante, pero la aceleración se mantiene, incluso ahí, en el valor constante de 9.8 m/s2 hacia abajo. Si la aceleración se volúera cero, el objeto permanecería ahí, iY la gravedad habría dejado de actuar l

Segundo, cuando el objeto vuelve al punto de partida, tiene la misma rapidez con la que se lanzó. (Las velocidades tienen la misma magnitud, pero dirección opuesta.)

Estos hechos pueden demostrarse matemáticamente. Véase el ejercicio 89.

Sugerencia para resolver problemas

Al resolver problemas de proyección vertical en que intervienen movimientos ascenden­tes y descendentes, suele ser conveniente dividir el problema en dos partes y considerar­las por separado. Como vemos en el ejemplo 2.11, en la parte ascendente del movimiento la velocidad es cero en la altura máxima. Una cantidad de cero simplifica los cálculos. Asi­mismo, la parte descendente del movimiento es análoga a la de un objeto que se deja caer desde la altura máxima con velocidad inicial cero.

No obstante, como muestra el ejemplo 2.11, podemos usar directamente las ecuacio­nes apropiadas para cualquier posición o tiempo del movimiento. Por ejemplo, en la parte (c) vemos que la altura se obtuvo directamente para un tiempo después de que la pelota ha­bía alcanzado la altura máxima. También podríamos haber calculado directamente la velo­cidad de la pelota en ese momento, con la ecuación 2.8', v = V - gt.o

Observemos también que la posición inicial siempre se tomó como Yo = O. Este su­puesto generalmente es válido y se acepta por conveniencia cuando en la situación sólo interviene un objeto (entonces, Yo = Oen to = O). Esta convención puede ahorrar mucho tiempo al plantear y resolver ecuaciones.

Lo IlÚsmo se cumple con un solo objeto en movimiento horizontal: generalmente po­demos tomar Xo = Oen to = O. Sin embargo, este caso tiene un par de excepciones: primera, si el problema especifica que el objeto está situado inicialmente en una posición distinta de Xo =O; segunda, si en el problema intervienen dos objetos, como en el Ejemplo integrado 2.6. En este caso, si consideramos que un objeto inicialmente está en el origen, la posición inicial del otro no será cero.

En nuestros estudios, estas situaciones no son muy comunes, pero deben tenerse pre­sentes. Habrá ocasiones en que escribamos las ecuaciones de cinemática sin Xoy Yo, supo­niendo que están en el origen y son cero en to = O.

Como último ejemplo de este capítulo, consideremos lo siguiente.

2.5 Caída libre 49

Page 70: Fisica_I

Ejemplo 2.12 • Caída libre en Marte

El Mars Polar Lander se lanzó en enero de 1999 y se perdió cerca de la superficie marciana en diciembre de 1999. No se sabe qué pasó con la nave. Supongamos que se dispararon los retro­cohetes y luego se apagaron, y que la nave se detu,"o para después caer hasta la superficie desde una altura de 40 m. (Muy improbable, pero supongamos que así fue.) Considerando que la na'"e está en caída libre, ¿con qué velocidad hizo impacto con la superficie?

Razonamiento. Esto parece análogo a un problema sencillo de dejar caer un objeto desde una altura. Y lo es, sólo que sucede en Marte. Ya vimos en esta sección que la aceleración de­bida a la gravedad en la superficie de la Luna es la sexta parte de la que experimentamos en la Tierra. La aceleración debida a la gravedad también varía en otros planetas, así que necesi­tamos conocer g"larte'

Solución,

Dado: y = -40 m (Yo = Ootra "ez) Hallar: L' (magnitud, rapidez)

V = Oo

g,larte = (0.379)g = (0.379) (9.8 m/s2)

= 3.7 m/s~ (del apéndice ID)

Así que podemos usar la ecuación 2.12':

[,2 = v~ - 2g,larteY = O - 2(3.7 m/s2) ( -40 m).

Entonces,

y

Ésta es la velocidad, que sabemos es hacia abajo, por lo que escogemos la raíz negativa: ¡; = -17 mis. Puesto que la rapidez es la magnitud de la velocidad, es 17 mis.

Ejercicio de refuerzo. Desde la altura de 40 m, ¿cuánto tardó el descenso del Lander? Calcúlelo empleando dos ecuaciones de cinemática distintas, y compare las respuestas. (Las respuestas de todos 105 Ejercicios de refuerzo se dall al p.llal del libro.)

Repaso del capítulo

Conceptos y ecuaciones importantes

• El movimiento implica un cambio de posición; se puede • La velocidad instantánea (un vector) describe con qué ra­describir en términos de la distancia recorrida (un escalar) pidez y en qué dirección se está moviendo algo en un ins­o del desplazamiento (un vector). tante dado.

• Una cantidad escalar sólo tiene magnitud (valor y múda­ • La aceleración es la tasa de cambio de la velocidad con el des); una cantidad vectorial tiene magnitud y dirección. tiempo, así que es una cantidad vectorial:

• La rapidez (un escalar) es la tasa de cambio de posición con cambio de velocidadel tiempo: aceleración media = --------- ­

tiempo que tarda el cambio 'd di distancia recorrida rapl ezme a = _ t:.v V2 - VI

tiempo total de recorrido a=-=--- (2.5)M t2 - tI

o sea • Las ecuaciones de cinemática para aceleración constante:d d

5 = (2.1).1t t2 - t: _ v + Vo v=--- (2.9)

• La velocidad media (un vector) es el desplazamiento divi­ 2 dido entre el tiempo total de recorrido: v = Vo + at (2.8)

desplazamientovelocidad media = ---------- X = Xo + ~(v + vo)t (2.10)

tiempo total de recorrido X = Xo + vot + ~af (2.11)

_ ~x X2 - Xl v=-=--- o sea (2.3) [12 = ifc T 2a(x - xo ) (2.12)

~t t: - t:

50 CAPíTULO 2 Cinemática: descripción del movimiento

Page 71: Fisica_I

• Un objeto en caída libre tiene una aceleración constante de magnitud g = 9.80 m/s2 (aceleración debida a la gravedad) cerca de la superficie de la Tierra.

• Si expresamos a = -g en las ecuaciones de cinemática para aceleración constante en la dirección y tenemos:

Ejercicios

2.1 Distancia y rapidez: cantidades escalares y 2.2 Desplazamiento unidimensional y velocidad:

cantidades vectoriales

1. Identifique las siguientes cantidades como escalares o vectoriales: (a) distancia, (b) velocidad, (c) rapidez y (d) desplazamiento.

2. PC Dos personas escogen diferentes puntos de referen­cia para especificar la posición de un objeto. ¿Esta dife­rencia afecta sus descripciones de las coordenadas del objeto? ¿Y su determinación del desplazamiento del objeto? Ex­plique.

3. Una cantidad escalar tiene (a) sólo magnitud, (b) sólo di­rección o (c) tanto dirección como magnitud.

4. Una cantidad vectorial tiene (a) sólo magnitud, (b) sólo dirección o (c) tanto dirección como magnitud.

5. PC ¿El desplazamiento de una persona en un viaje pue­de ser cero, aunque la distancia recorrida en el viaje no sea cero? ¿Es posible la situación in\'ersa? Explique.

6. PC Le dicen que una persona caminó 500 m. ¿Qué puede decir con certeza acerca de la posición final de la persona relativa al punto de partida?

7. PC Un objeto viaja a velocidad constante. ¿Qué relación hay entre la rapidez del objeto y su velocidad?

8. PC la rapidez es la magnitud de la velocidad. ¿la rapidez media es la magnitud de la velocidad media? Explique.

9. PC Si el desplazamiento de un objeto es 300 m al norte, ¿qué puede decir acerca de la distancia recorrida por el objeto?

10. • ¿Qué magnitud tiene el desplazamiento de un auto­móvil que recorre media vuelta de una pista circular con 150 m de radio? ¿Y cuando recorre una vuelta completa?

11. • Un estudiante lanza una piedra verticalmente hacia arriba desde su hombro, que está 1.65 m sobre el suelo. ¿Qué desplazamiento tendrá la piedra cuando caiga al suelo?

12. • En 1999, el corredor marroquí Hicham El Guerrouj co­rrió la milla en 3 min, 43.13 s. ¿Qué rapidez media tuvo durante la carrera?

v = va - gt (2.8')

y = Yo -+- ~(v + vo)t (2.10')

y = Yo + vot - ~gP (2.11')

¡) = ~ - 2g(y - Yo) (2.12')

13. • Un autobús viaja con una rapidez media de 90 km/h. En promedio, ¿qué distancia recorre en 20 min? ¿Esta dis­tancia es la magnitud del desplazamiento real del auto­bús? Explique.

14. • Un automovilista conduce 150 km de una ciudad a otra en 2.5 h, pero hace el viaje de regreso en sólo 2.0 h. Calcu­le la rapidez media en (a) cada mitad del viaje redondo y (b) el \'iaje redondo.

15. • Una anciana camina 0.30 km en 10 min, dando la vuel­ta a un centro comercial. (a) Calcule su rapidez media en mi s. (b) si quiere aumentar su rapidez media en un 20% al dar una segunda vuelta, ¿en cuántos minutos deberá caminarla?

16. El. Un automóvil de carreras da una vuelta a una pista circular de 500 m de radio en SO s. (a) La velocidad media del carro es (1) cero, (2) 100 mis, (3) 200 mis o (4) ningu­na de las anteriores. ¿Por qué? (b) Calcule la rapidez me­dia del carro.

17. El •• L'n estudiante corre 30 m al este, 40 m al norte y 50 m al oeste. (a) la magnitud del desplazamiento neto del estudiante es (1) entre Oy 20 m, (2) entre 20 y 40 m o (3) en­tre 40 y 60 m. (b) Calcule el desplazamiento neto.

18. •• Un estudiante lanza una pelota verticalmente hacia arriba de modo que sube 7.1 m hasta su altura máxima. Si la pelota se atrapa en la altura inicial 2.4 s después de ser lanzada, (a) ¿qué rapidez media tuvo?, (b) ¿qué velocidad media tu\'o?

19. •• Un insecto repta por el borde de una piscina rectan­gular de 27 m de longitud y 21 m de anchura TCFig. 2.17). Tarda 30 min en reptar de la esquina A a la esquina B. Calcule (a) su rapidez media y (b) la magnitud de su ve­locidad media.

... FIGURA 2.17 Rapidez vs. velocidad Véase el ejercicio 19. (No está a escala, se ha desplazado al insecto por claridad.)

Ejercicios 51

Page 72: Fisica_I

20. •• La distancia de una vuelta a una pista ovalada de tie­rra para motociclismo es de 1.50 km. Un motociclista que va con rapidez constante da una vuelta en 1.10 mino Calcule su rapidez en mis. ¿La velocidad de la moto también es constante? Explique.

21. •• (a) Dado que la rapidez del sonido es de 340 mis, y la de la luz, de 3.00 X 108 mis (186 000 mi/s), ¿cuánto tiem­po transcurrirá entre un relámpago y el trueno resultante si el rayo cae a 2.50 km del observador? (b) ¿Cambia la respuesta si se supone que la luz viaja con rapidez infini­ta (es decir, que el relámpago se ve instantáneamente)?

22. •• En la TFig. 2.18 se muestra una gráfica de posición contra tiempo para un objeto en movimiento rectilíneo. (a) Calcule las velocidades medias para los segmentos AB, BC, CD, DE, EF, FG Y BG. (b) Diga si el movimiento es uniforme o no uniforme en cada caso. (c) Calcule la ve­locidad instantánea en el punto D.

x

10.0 -----r---1c---,---D--,--'----.-1--'---1--'- ­

9.0-t----i¡--'----.....¿'--+'----i----:

8.0 I : / l\ I I ! L \~ i

7. O '---+-----fl,F-­v+----'---t\t-F--1¡--+----+-----'

e ~ 60

1/ I'0 5.0 .¡¡:¡'C:; y I O 4.0 o...

3.0 Ji I

2.0 J A

1.0 1B

\i 1\ I \

----r--,-~---+---

\.F G

O 20 40 6.0 80 100 12.0 Tiempo (s)

... FIGURA 2.18 Posición contra tiempo Véase el ejercicio 22.

23. •• Al demostrar un paso de baile, una persona se mueve en una dimensión, como se muestra en la TFig. 2.19. Calcu­

x

4.0

30

2.0~ e '0 10'C:;.¡¡:¡ O o... O

-1 .O 1---+--1--I---+--1--f-\--+-+--f---1

- 2 O '-----'---'--'-----'---'--'-----'---'---'----' Tiempo (s)

... FIGURA 2.19 Posición contra tiempo Véase el ejercido 23.

le (a) la rapidez media y (b) la velocidad media en cada fase del mmimiento. (c) Calcule la velocidad instantánea en t = 1.0 s, 2.5 SO, 4.5 s V 6.0 s. (d) Calcule la velocidad media para el inten'alo entre t =4.5 s y t =9.0 S. [Sugerencia: Re­cuerde que el desplazamiento total es el desplazamiento entre el punto de partida y el punto final.]

24. •• Podemos determinar la rapidez de un automóvil mi­diendo el tiempo que tarda en viajar entre dos mojones de milla en una carretera. (a) ¿Cuántos segundos deberá tardar el automóvil en viajar entre dos mojones consecu­tivos si su rapidez media es de 65 mi/h? (b) Calcule la ra­pidez media si el carro tarda 65 s en viajar entre los mojones de milla.

25. •• Un terremoto genera dos tipos de ondas progresivas, llamadas "transversales" y "longitudinales". La rapidez media de las ondas sísmicas trans\'ersales y longitudinales en roca es de 8.9 km/s y 5.1 km/s, respectivamente. Un sismógrafo registra la llegada de las ondas transversales 73 s antes que la de las ondas longitudinales. Suponiendo que las ondas viajan en línea recta, ¿a qué distancia del centro del terremoto está el sismógrafo?

26. •• Una aerolínea opera dos tipos de aviones. El más veloz tiene una rapidez de crucero de 565 mi/h; el más lento, de 505 mi/h. Si el avión más rápido tarda 4.50 ho­ras en cubrir una ruta establecida, ¿cuántos minutos más tardará el más lento en hacer el mismo viaje?

27. • •• Un eshldiante que regresa a casa en automóvil en Navidad parte a las 8:00 A .:-L para hacer el viaje de 675 km, que efectúa casi en su totalidad en autopistas interestata­les no urbanas. Si quiere llegar a casa antes de las 3:00 P.M., ¿qué rapidez media deberá mantener? ¿Tendrá que exceder el límite de nlocidad de 65 mi/h?

28. ••• Dos corredoras se aproximan una a la otra en una pis­ta recta con rapideces constantes de 4.50 mis y 3.50 mis, respectivamente, cuando están separadas 100 m (T Fig_ 2.20). ¿Cuánto tardarán en encontrarse y en qué posición lo harán si mantienen su rapidez?

450 mIs 3.50 mIs.....­•

---""'----~ __1 l. ""r----l00 rr -----J""r----_./

... FIGURA 2.20 ¿Cuándo y dónde se encontrarán? Véase el ejercicio 28.

29. • •• Al conducir por su ruta acostumbrada a la univer­sidad, un estudiante calcula que su rapidez media es de 30 km/h. Esa tarde, con prisa por llegar a casa, quiere pro­mediar 60 km/h en el viaje redondo. ¿Qué rapidez media deberá mantener durante el regreso por la misma ruta pa­ra lograrlo?

52 CAPíTULO 2 Cinemática: descripción del movimiento

Page 73: Fisica_I

v

2.3 Aceleración uniformemente su velocidad hasta parar. (a) ¿La direc­

30. PC El pedal de la gasolina de un automóvil se conoce co­mo acelerador. ¿Qué podría producir también aceleración? (a) Los frenos; (b) el volante; (c) la palanca de velocida­des; (d) las tres cosas. Explique.

31. PC Un automóvil viaja con rapidez constante de 55 mi/h en una pista circular. ¿Está acelerando? Explique.

32. Una gráfica de posición contra tiempo para un objeto que tiene aceleración constante es (a) una línea horizontal, (b) una línea recta no horizontal ni vertical, (c) una línea ver­tical o (d) una curva.

33. PC en objeto que viaja a velocidad constante Va experi­menta una aceleración constante en la misma dirección durante un tiempo t.luego experimenta una aceleración de igual magnitud en la dirección opuesta a va durante el mismo tiempo t. ¿Qué velocidad final tendrá el objeto?

34. PC ¿Un objeto que se mueve rápidamente siempre tiene una mayor aceleración que uno que se mueve lentamen­te? Dé unos cuantos ejemplos, y explique.

35. PC ¿Un objeto puede tener velocidad positiva y acelera­ción negativa? Apoye su respuesta con un ejemplo.

36. PC Describa los movimientos de los dos objetos cuyas gráficas de velocidad contra tiempo se muestran en la ... Fig.2.21.

(b)

"D ro

"D'u o Qi> 0iL---------.fIC----------- ­

.A. FIGURA 2.21 Descripción de movimiento Véase el ejer­cicio 36.

37. • Un automóvil que viaja a 25.0 km/h por un camino recto y plano acelera a 65.0 km/h en 6.00 s. Calcule la Illflg­nitud de la aceleración media del automóvil.

38. • Un auto deporti\'o puede acelerar de Oa 60 mi/h en 3.9 s. Calcule la magnitud de su aceleración media en,m/s2

.

39. • Si el automóvil del ejercicio 38 puede acelerar a 7.2 m/s2

, ¿cuánto tardará en acelerar de Oa 60 mi/h?

40. El. Un matrimonio viaja en auto a 40 km/h por una ca­rretera recta. Ven un accidente en la distancia, así que el conductor aplica los frenos y en 5.0 s el automóvil baja

ción del vector de aceleración es (1) en la misma dirección, (2) en la dirección opuesta o (3) a 90~ del vector de veloci­dad? ¿Por qué? (b) ¿Cuánto debe cambiar la velocidad cada segundo entre el inicio del frenado y el alto total?

41. •• Una lancha que parte del reposo en un lago acelera en línea recta con una tasa constante de 2.0 m/s2 durante 6.0 s. ¿Qué distancia recorre en ese tiempo?

42. •• El conductor de un auto que viaja con velocidad ini­cial de 3.5 mis apaga el motor y pone la transmisión en neutral. Si el efecto combinado de la resistencia del aire y la fricción de rodamiento causa una desaceleración de 0.50 m/s2, ¿cuánto tiempo tardará el auto en detenerse?

43. •• Calcule la aceleración para cada segmento de la gráfi­ca de la ... Fig. 2.22. Describa el movimiento del objeto du­rante el intervalo total de tiempo.

V

100 f----;--.-----.-1-----.--!---+----, ~ (40: 80) pO.O.8.0)E S.Of----- ---¡-~--+-__+---1

"D 6.O!------.(--+----+-~-~---+--~'--iro "D ü 4 OI---f-----f----+-+-------'l<--+-----io

Qi> 2.0 f-I.----4----+-+---f---'!r----i

O'-----'-----'------L----'------'---""'-- ­4.0 SO 120 160

Tiempo (s)

.A. FIGURA 2.22 Velocidad contra tiempo Véanse los ejer­cicios 43 y 66.

44. •• La ... Fig. 2.23 muestra una gráfica de velocidad con­tra tiempo para un objeto en movimiento rectilíneo. (a)

V

10.0 ! I !

I I I ¡ I I !

¡ SO

!¡ 1/\ I I6.0

I I 4.0 1 V \ ! ,

Vi i'> ! / \ I I I i .s 20 ,

; "D \ ;

I1ro O V

"D 20 4:0 \6:0 S.O 10.0 12.0'u ,o -20 !

Qi ¡>; \ I I i

-40

-6.0 I i \ I I /, I I I \-S.O I

¡ 1Y+11\ :1 : !-10.0 t

, ¡

,

¡I

! vlTTli

! I i !!-12.0 Tiempo (s)

.A. FIGURA 2.23 Velocidad contra tiempo Véanse los ejer­cicios 44 y 67.

Ejercicios 53

Page 74: Fisica_I

Calcule la aceleración para cada fase del movimiento. (b) Describa el movimiento del objeto durante el último seg­mento de tiempo.

45. • •• Un tren normalmente viaja con rapidez uniforme de 72 km/h por un tramo largo de vía recta y plana. Cier­to día, el tren debe hacer una parada de 2.0 min en una es­tación sobre esta vía. Si el tren desacelera con una tasa uniforme de 1.0 m/S2 y, después de la parada, acelera con una tasa de 0.50 m/S2, ¿cuánto tiempo habrá perdido por pa­rar en la estación?

2.4 Ecuaciones de cinemática (aceleración constante)

46. la gráfica de velocidad contra tiempo para una acelera­ción rectilínea constante es (a) una línea horizontal, (b) una línea vertical, (c) una línea recta no horizontal ni ver­tical o (d) una línea curva.

47. la gráfica de posición contra tiempo para una aceleración rectilínea constante sería (a) una línea horizontal, (b) una línea vertical, (c) una línea recta no horizontal ni vertical o (d) una curva.

48. PC Si la gráfica de velocidad contra tiempo de un objeto es una línea horizontal, ¿qué puede decir acerca de la ace­leración del objeto?

49. Un objeto acelera uniformemente desde el reposo duran­te t segundos. la rapidez media del objeto en este inter­valo de tiempo es ~at, (b) ~at2, (c) 2at o (d) 2at2

50. PC Un compañero dice que una aceleración negativa siempre implica que un objeto en movimiento está desa­celerando. ¿Es verdad esto? Explique.

51. • En un rally de autos deportivos, un auto que parte del reposo acelera uniformemente con una tasa de 9.0 m/s2 a lo largo de una distancia recta de 100 m. El tiempo a supe­rar en este evento es de 4.5 s. ¿Lo logra el conductor? ¿Qué aceleración mínima se requiere para hacerlo?

52. • Un auto acelera desde el reposo con tasa constante de 2.0 m/s2 durante 5.0 s. (a) ¿Qué rapidez tendrá al término de ese lapso? (b) ¿Qué distancia recorrerá en ese tiempo?

53. • Un automóvil que viaja a 35 mi/h debe parar en'un tramo de 35 m de una carretera. (a) ¿Qué magnitud míni­ma debe tener su aceleración? (b) ¿Cuánto tiempo tardará en detenerse el auto con esa desaceleración?

54. • Una lancha de motor que viaja por una pista recta fre­na uniformemente de 75 a 40 km/h en una distancia de , 50 m. Calcule la aceleración de la lancha.

55. •• El conductor de una camioneta que va a 100 km/h aplica los frenos y el vehículo desacelera uniformemente a 6.50 m/s2 en una distancia de 20.0 m. (a) ¿Qué rapidez en km/h tiene la camioneta al término de esta distancia? (b) ¿Cuánto tiempo ha transcurrido?

56. •• Un carro cohete experimental que parte del reposo al­canza una rapidez de 560 km/h después de un recorrido recto de 400 m en una llanura plana. Suponiendo que la

54 CAPíTULO 2 Cinemática: descripción del movimiento

aceleración fue constante, (a) ¿qué tiempo tardó el reco­rrido? (b) ¿Qué magnitud tuvo la aceleración?

57. •• Un carro cohete viaja con rapidez constante de 250 km/h por una llanura. El conductor imparte al vehículo un empuje en re\'ersa y el carro experimenta una desace­leración continua y constante de 8.25 m/s2 ¿Cuánto tiem­po transcurre hasta que el vehículo está a 175 m del punto en el que se aplicó el empuje en reversa? Describa la si­tuación en su respuesta.

58, •• Dos automóviles idénticos que pueden acelerar a 3.00 m/s2 compiten en una pista recta con arranque en movimiento. El carro A tiene una rapidez inicial de 2.50 m/s; el B, de 5.0 mis. (a) Calcule la separación de los dos auto­móviles después de 10 S. (b) ¿Qué automóvil se mueve a mayor velocidad después de 10 s?

59, El •• en objeto se mueve en la dirección ~x con una ra­pidez de 40 mis. Al pasar por el origen, comienza a expe­rimentar una aceleración constante de 3.5 m/s2 en la dirección -x. (a) ¿Qué sucederá después? (1) El objeto in­vertirá su dirección de movimiento en el origen. (2) El objeto seguirá \'iajando en la dirección +X. (3) El objeto viajará en la dirección +x y luego invertirá su dirección. ¿Por qué? (b) ¿Cuánto tiempo transcurre antes de que el objeto vuelva al origen? (c) ¿Qué velocidad tiene el objeto al volver al origen?

60. •• Cna bala de rifle cuya rapidez al salir del cañón es de 330 m/s se dispara directamente a un material denso es­pecial que la detiene en 30 cm. Suponiendo que la desace­leración de la bala fue constante, ¿qué magnitud tuvo?

61. •• Gna bala que viaja horizontalmente con una rapidez de 350 mis golpea una tabla perpendicular a la superfi­cie, la atraviesa y sale por el otro lado con una rapidez de 210 mis. Si la tabla tiene 4.00 cm de espesor, ¿cuánto tar­dó la bala en atravesarla?

62. •• Una piedra golpea el piso con una rapidez de 10 mis y deja un agujero de 25 cm de profundidad. ¿Qué magni­tud tuvo su desaceleración?

63. •• LTn avión a reacción que se lanza desde un portaavio­nes acelera desde el reposo sobre una pista de 94 m du­rante 2.5 s. (a) Calcule su aceleración, suponiendo que es constante. (b) Calcule la rapidez con que es lanzado el avión.

64. •• El límite de velocidad en una zona escolar es de 40 km/h. Cn conductor que viaja a esta velocidad ve que un nIDo cruza corriendo la calle 13 m adelante de su auto­móvil. Aplica los frenos, y el auto desacelera con una tasa uniforme de 8.0 m/s2

. Si el tiempo de reacción del con­ductor es de 0.25 s, ¿el auto se detendrá antes de golpear al nIDo?

65. •• Suponiendo un tiempo de reacción de 0.50 s para el conductor del ejercicio 64, ¿el automóvil se detendrá an­tes de golpear al nIDo?

66. •• (a) Demuestre que el área bajo la curva de una gráfica de velocidad contra tiempo, con aceleración constante, es igual al desplazamiento. [Sugerencia: El área de un trián­

Page 75: Fisica_I

gula es abl2, la mitad de la altura multiplicada por la ba­se.] (b) Calcule la distancia recorrida en el movimiento re­presentado en la Fig. 2.22.

67. • •• La Fig. 2.23 muestra una gráfica de velocidad con­tra tiempo para un objeto en movimiento rectilíneo. (a) Calcule las velocidades instantáneas a t = 8.0 s y t = 11.0 s. (b) Calcule el desplazamiento final del objeto. (c) Calcule la distancia total que el objeto recorre. (Véase el ejercicio 66(a).)

68. El... (a) Un automóvil que viaja con rapidez v puede frenar para hacer un alto de emergencia en una distancia x. Suponiendo que las demás condiciones de manejo son similares, si la rapidez del automóvil es el doble, la dis­tancia de detención será (1) V2x, (2) 2x 0(3) 4x. (b) Un conductor que viaja a 40.0 kmlh en una zona escolar pue­de frenar para hacer un alto de emergencia en 3.00 m. Calcule la distancia de frenado si el automóvil estuviera viajando a 60.0 km/h.

69. • •• Dé las cuatro ecuaciones de cinemática del texto. Para cada una, conteste estas preguntas: (a) ¿Qué repre­senta cada símbolo? (b) ¿En qué condiciones se cumplen las ecuaciones?

70. • •• Un objeto se mueve en la dirección x positiva con aceleración constante. En x = 5.0 m, su rapidez es de 10 mis. 2.5 s después, el objeto está en x = 65 m. Calcule su aceleración.

2.5 Caída libre (sin considerar resistencia del aire)

71. Un objeto se lanza verticalmente hacia arriba. ¿Cuál de estas afirmaciones es cierta? (a) Su velocidad cambia de manera no uniforme; (b) su altura máxima es indepen­diente de la velocidad inicial; (c) su tiempo de ascenso es un poco mayor que su tiempo de descenso; (d) la rapidez al volver a su punto de partida es igual a su rapidez inicial.

72. El movimiento de "caída libre" descrito en esta sección es válido para (a) un objeto que se deja caer del reposo, (b) un objeto que se lanza verticalmente hacia abajo, (c) un objeto que se lanza verticalmente hacia arriba o (d) todos los casos anteriores.

73. Pe Cuando una pelota se lanza hacía arn'ba, ¿qué veloci~

dad y aceleración tiene en su punto más alto?

74. Pe Un objeto que se suelta en caída libre (a) cae 9.8 m ca­da segundo, (b) cae 9.8 m durante el primer segundo, (c) tiene un incremento de velocidad de 9.8 mis cada segundo o (d) tiene un incremento de aceleración de 9.8 mis cada segundo.

75. Pe Imagine que está en el espacio lejos de cualquier pla­neta, y lanza una pelota como haría en la Tierra. Describa el movimiento de la pelota.

76. Pe Usted deja caer una piedra desde la ventana de un edificio. Después de un segundo, deja caer otra piedra. ¿La distancia que separa las dos piedras (a) aumenta, (b) disminuye o (c) se mantiene?, o (d) no se da suficiente in­formación para saberlo. Explique.

77. • Si un objeto que se deja caer cae 19.6 m en 2.00 s, ¿qué distancia caerá en 4.00 s?

78. • Un estudiante deja caer una pelota desde la azotea de un edificio alto; la pelota tarda 2.8 s en llegar al suelo. (a) ¿Qué rapidez tenía la pelota justo antes de tocar el suelo? (b) ¿Qué altura tiene el edificio?

79. El. El tiempo que un objeto que se deja caer desde el acantilado A tarda en chocar con el agua del lago que está abajo es el doble del tiempo que tarda en llegar al lago otro objeto que se deja caer desde el acantilado B. (a) La altura del acantilado A es (1) la mitad, (2) el doble o (3) cuatro veces la del acantilado B. (b) Si el objeto tarda 1.8 s en caer del acantilado A al agua, ¿qué altura tienen los dos acantilados?

80. • Para el movimiento de un objeto que se suelta en caída libre, dibuje la forma general de las gráficas de (a) v con­tra t y (b) y contra t.

81. • Un truco conocido consiste en dejar caer un billete de dólar (a lo largo) entre el pulgar y el índice de un compa­ñero, diciéndole que lo sujete lo más rápidamente posible para quedarse con él. (La longitud del billete es de 15.7 cm, y el tiempo de reacción medio del ser humano es de unos 0.2 s. Véase la Fig. 2.15.) ¿Esta propuesta es un buen negocio? Justifique su respuesta.

82. • Un niño lanza una piedra verticalmente hacia arriba con una rapidez inicial de 15 mis. ¿Qué altura máxima alcanzará la piedra antes de descender?

83. • En el ejercicio 82, ¿qué altura máxima alcanzaría la pie­dra si el niño y la piedra estuvieran en la superficie de la Luna, donde la aceleración debida a la gravedad es de sólo 1.67 miS2?

84. •• Un rifle de resorte lanza una bala de 0.0050 kg verti­calmente hacia arriba con una velocidad inicial de 21 mis. (a) ¿Qué altura tendrá la bala 3.0 s después de ser dispa­rada? (b) ¿En qué tiempos la bala está 12 m arriba de la boca del cañón?

85. •• El techo de un aula está 3.75 m sobre el piso. Un estu­diante lanza una manzana verticalmente hacia arriba, sol­tándola a 0.50 m sobre el piso. Calcule la rapidez inicial máxima que puede impartirse a la manzana sin que to­que el techo.

86. •• Las Torres Gemelas Petronas de Malasia y la Torre Sears de Chicago tienen alturas de 452 y 443 m, respecti­vamente. Si se dejaran caer objetos desde la punta de ca­da una, ¿con qué diferencia de tiempo llegarían al suelo?

87. •• Una piedra se lanza verticalmente hacia abajo con una rapidez inicial de 14 mis desde una altura de 65 m. (a) ¿Qué distancia recorre la piedra en 2.0 s? (b) ¿Qué ve­locidad tiene justo antes de chocar con el suelo?

88. •• Una flecaa se dispara verticalmente hacia arriba. Tres segundos después, está a una altura de 35 m. (a) ¿Qué ra­pidez inicial tenía la flecha? (b) ¿Cuánto tiempo está en

Ejercicios 55

Page 76: Fisica_I

vuelo la flecha desde que se lanza hasta que vuelve a la altura original? (Suponga que la flecha cae verticalmente.)

89. •• Remitiéndonos a la pelota del ejemplo 2.11 y la Fig. 2.16, (a) compare el tiempo de ascenso (ta) con el tiempo de descenso (td) que la pelota tarda en volver al punto de partida, y (b) compare la velocidad inicial de la pelota con la que tiene al volver al punto de partida.

90. •• Usted lanza una piedra verticalmente hacia arriba con una rapidez inicial de 6.0 mis desde la ventana de una oficina del tercer piso. Si la ventana está 12 m sobre el sue­lo, calcule (a) el tiempo que la piedra está en el aire y (b) la rapidez que tiene la piedra justo antes de tocar el suelo.

91. El •• Una pelota Superball™ se deja caer desde una al­tura de 4.00 m. Suponiendo que la pelota rebota con el 95% de su rapidez de impacto, (a) ¿rebotaría a (1) menos del 95%, (2) el 95.0% o (3) más del 95% de la altura inicial? (b) ¿Qué altura alcanzará la pelota?

92. •• Dos pelotas se lanzan verticalmente, ambas con una rapidez inicial de 10.0 mis desde una altura de 60.0 m so­bre el suelo. La pelota A se lanza hacia arriba, mientras que la B se proyecta hacia abajo. (a) ¿Con qué diferencia de tiempo tocan las dos pelotas el suelo? (b) ¿Las masas de las pelotas afectan el resultado?

93. • •• En la ..Fig. 2.24, un estudiante en una ventana del segundo piso de una residencia ve que su profesora de matemáticas camina por la acera junto al edificio. Deja caer un globo lleno de agua desde 18.0 m sobre el suelo cuando la profesora está a 1.00 m del punto que está di­rectamente abajo de la ventana. Si la estatura de la profe­sora es de 1.70 m y camina con una rapidez de 0.450 mis, ¿la golpeará el globo? Si no, ¿qué tan cerca pasará de ella?

Tres

I

I I I I I

180 m I

j 1.70 m --~

1.00 m

.. FIGURA 2.24 Déle a la profe Véase el ejercicio 93. (Esta figura no está a escala.)

94. ••• Un fotógrafo en un helicóptero que asciende vertical­mente con una tasa constante de 12.5 mi s deja caer acci­dentalmente una cámara por la ventana cuando el helicópte­ro está 60.0 m sobre el suelo. (a) ¿Cuánto tardará la cámara en llegar al suelo? (b) ¿Con qué rapidez chocará?

56 CAPíTULO 2 Cinemótlca: descripción del movimiento

95. El ••• La aceleración debida a la gravedad en la Luna es la sexta parte que en la Tierra. (a) Si un objeto se dejara caer desde la misma altura en la Luna v en la Tierra, el tiempo que tardaría en llegar al suelo ser'ía (1) \16, (2) 6 o (3) 36 veces mayor que el que tardaría en la Tierra. (b) Para el caso de un proyectil con una velocidad inicial de 18.0 mis hacia arriba, calcular la altura máxima y el tiempo total de vuelo en la Luna v en la Tierra.

96. • •• Un objeto que se dejó caer tarda 0.210 s en pasar por una ventana de 1.3 m de altura. ¿Desde qué altura arriba del borde superior de la ventana se soltó el objeto? (Véase la ..Fig. 2.25.)

~EJ T, ~ 1.35 m

L...J..f------'-¡ _J

.. FIGURA 2.25 ¿De dónde vino? Véase el ejercicio 96.

Ejercicios adicionales

97. Al lanzar un objeto verticalmente hacia arriba con una ra­pidez de 7.25 mis desde la azotea de un edificio alto, un estudiante se inclina sobre el borde del edificio para que el objeto no choque con el edificio al descender. (a) ¿Qué ve­locidad tendrá el objeto cuando haya viajado una distan­cia total de 25.0 m? (b) ¿Cuánto tardará en viajar esa distancia?

98. Después de aterrizar en una pista recta, un avión a reac­ción rueda hasta detenerse. Su velocidad media es de -35.0 km/h. Si el avión tarda 7.00 s en parar, ¿qué veloci­dad y aceleración iniciales tenía?

99. Un proyectil que se mueve verticalmente alcanza una al­tura máxima de 23 m sobre su punto de partida. (a) ¿Qué velocidad inicial tuvo? (b) ¿Qué altura sobre el punto de partida alcanzó en t = 1.3 s?

100. Un carro de arrancones que viaja a 200 kmlh en una pis­ta recta despliega un paracaídas y frena uniformemente hasta una rapidez de 20 kmlh en 12 s. (a) Calcule la ace­leración del carro. (b) ¿Qué distancia recorre el carro en el intervalo de 12 s?

Page 77: Fisica_I

101. En un viaje de un lado a otro del país, un matrimonio conduce 500 mi en 10 h el primer día, 380 mi en 8.0 h el segundo día y 600 mi en 15 h el tercer día. Calcule la rapi­dez media durante todo el viaje.

102. Un automóvil que viaja a 25 mi/h tiene 1.5 s para dete­nerse al acercarse a un semáforo en rojo. La ma~tud de la desaceleración máxima del auto es de 7.0 mis . ¿Logra­rá detenerse antes de llegar al semáforo?

103. El Un auto da tres cuartos de vuelta en una pista circular de radio R. (a) La magnitud del desplazamiento es (1) menor que R; (2) mayor que R, pero menor que 2R; o (3) mayor que 2R. (b) Si R = 50 m, ¿qué magnitud tuvo el desplazamiento?

104. Un policía vigila desde detrás de una valla para ah'apar a quienes pasen una intersección (x = O) con exceso de velo­cidad. En cierto instante (t = O), un automó\,il obviamente excedido de velocidad pasa a 30 mis (67 mph). El policía comienza a perseguirlo en ese instante, manteniendo una aceleración constante de 3.5 m/s2 en la dirección +x. El automóvil mantiene su velocidad durante toda la per­secución. (a) En una gráfica de x contra t, grafique la posi­ción de cada vehículo. (b) ¿Cuánto tarda el policía en atrapar al conductor?

105. ¿Con qué rapidez se debe proyectar verticalmente hacia arriba un objeto para que alcance una altura máxima de 14.0 m sobre su punto de partida)

106. Un tren en una \'ía recta y plana tiene una rapidez ini­cial de 45.0 km/h. Se aplica una aceleración uniforme de 1.50 m/S2 mientras el tren recorre 200 m. (a) ¿Qué rapidez tendrá el tren al término de esa distancia? (b) ¿Cuánto tar­dó el tren en cubrir los 200 m?

107. Gn auto que viaja a 85 km/h en un camino recto se frena uniformemente hasta detenerse en 10 s. ¿Qué distancia recorre en ese tiempo?

108. Un automóvil y una motocicleta parten del reposo al mis­mo tiempo en una pista recta, pero la motocicleta está 25.0 m atrás del automóvil (T Fig. 2.26). El auto acelera con una tasa uniforme de 3.70 m/s2, y la motocicleta, a 4.40 m/s2 (a) ¿Cuánto tardará la motocicleta en alcanzar al automóvil? (b) ¿Qué distancia habrá recorrido cada ve-o

250 rn .-;'­

...... d-- __ ...~.,

~.. .~ .

Reposo.············

.Á. FIGURA 2.26 Carrera empatada Véase el ejercicio lOS. (La figura no está a escala.)

hículo durante ese tiempo? (c) ¿Qué tan adelante del auto estará la motocicleta 2.00 s después? (Ambos vehículos si­guen acelerando.)

109. Una persona lanza una piedra verticalmente hacia arriba con una rapidez inicial de 15 mis en un puente que está 25 m sobre la superficie del agua. Si la piedra salva apenas el puente durante el descenso, (a) ¿qué rapidez tendrá jus­to antes de chocar con el agua? (b) Calcule el tiempo total que la piedra está en el aire.

110. Un objeto en reposo experimenta una aceleración de 1.5 m/s2 durante 6.0 s y luego viaja a velocidad constante durante otros 8.0 s. Calcule la \'elocidad media del obje­to durante el intervalo de 14 s.

111. El Para calcular la profundidad a la que está la superficie del agua en un pozo, una persona deja caer una piedra des­de el borde del pozo y arranca simultáneamente un cro­nómetro, el cual para cuando escucha el chapuzón de la piedra. La lectura es de 3.65 s. La rapidez del sonido es de 340 mis. (a) ¿Por qué estamos considerando la rapidez del sonido) (b) Calcule la profundidad de la superficie del agua bajo el borde del pozo. Suponga que el tiempo de reac­ción de la persona para detener el cronómeh'o es de 0.250 s.

112. Un cohete de prueba que contiene una sonda para deter­minar la composición de la atmósfera superior se dispara verticalmente hacia arriba desde una posición inicial a nivel del suelo. Durante el tiempo t que dura el combustible, el cohete asciende con aceleración constante hacia arriba de magnitud 2g. Suponga que la altura que alcanza el co­hete no es tan grande como para que la fuerza gravitacio­nal de la Tierra no deba considerarse constante. (a). ¿Qué altura y rapidez tiene el cohete cuando se agota el com­bustible? (b) ¿Qué altura máxima alcanza el cohete? (c) Si t = 30.0 s, calcule la altura máxima del cohete.

113. Un automóvil que parte del reposo y viaja en una sola dirección tiene velocidades de 5.0 mis, 10 mis, 15 mis, 20 mis y 25 mis después de 1.0 s, 2.0 s, 3.0 s, 4.0 s Y5.0 s, respectivamente. (a) Calcule la magnitud de la acelera­ción del vehículo. (b) Calcule su velocidad media en el intervalo de:; s. (c) Haga una gráfica de v contra t. (d) Ha­ga una gráfica de x contra t.

114. En un paseo por el campo, un matrimonio carnina 1.S0 km al este por un camino recto en 20.0 min Yluego 2.40 km di­rectamente al norte en 35.0 mino (a) Calcule la velocidad media en cada segmento del paseo y para el paseo total. (b) Calcule la rapidez media para el paseo total. (c) Si el matrimonio sigue un camino en línea recta que lo lleva de vuelta al punto original de partida en 25.0 min, ¿qué rapi­dez v velocidad medias habrá mantenido durante todo el paseo?

115. en estudiante calcula que si esquía a 10 km/h llegará a su cabaña en el bosque a la 1:00 P..\1. Si esquía a 15 km/h, llegará a las 11:00 A.'\1. ¿Con qué rapidez debe esquiar pa­ra llegar a la cabaña al mediodía?

Ejercicios 57

Page 78: Fisica_I

Apéndices ..

APÉNDICE I Relaciones APENDICE IV Lista alfabética de matemáticas elementos químicos

Relaciones algebraicas APÉNDICE V Propiedades de isótopos Relaciones de geometría selectos

y trigonometría

Notación de potencias Respuestas a Ejercicios de refuerzo de 10 (científica) Respuestas a Ejercicios impares

APÉNDICE II Teoría cinética de los gases

APÉNDICE 111 Datos planetarios

Apéndice I Relaciones matemáticas

Relaciones algebraicas Relaciones de geometría y trigonometría 2(a + b)2 = a + 2ab + b2

(a - b)2 = a2 - 2ab + b2 Áreas y volúmenes de algunas figuras comunes (a 2 - b2) = (a + b)(a - b) 2

7 1TdCírculo: 4 = 1Tl- = - (área)" 4Fórmula cuadrática

-b ± Vb2 - 4ac e = 21T r = 1T d (circunferencia)

Si ax2 + bx + e = O, entonces x = ----- ­la

Triángulo: A = iab

Potencias y exponentes Esfera: A = 41T1·2

.lO = 1

V = 51Tr3

Cilindro: A = r,?- (extremo)

x2 = x· X A = 21T1'h (cuerpo)

A = 2(1Tr2) + 2r,rh (total)

3x = .l· x· X V = 1T1.2h

etc. etc. etc. Definiciones de funciones trigonométricas

.la . .lb = x(a~b)

xa/xb = .l(a-b) y x sen e ysen e =- cos e = - tan e = -- =­

xab(Xa)b = r r cos e x

Logaritmos eo (rod) sen e cose ton e Si x = a", entonces n = laga x. 0° (O) O 1 O logaritmos comunes: base 10 30° (1T/6) 0.500 0.866 0.577 (se supone cuando se usa la abreviatura "lag", a menos que 45° (Ti/4) 0.707 0.707 1.00 se especifique otra base) 60° (1T/3) 0.866 0.500 1.73

lag 10' = x 90° (1T/2) 1 O ~::>O

lag xy = log x + log y log x/y = lag x - lag y Para ángulos muy pequeños,

lag x'l = y lag x cos e :::::: 1 sen e ::::: e(radianes) logaritmos naturales: base e = 2.71828 ... (se abreyia "ln") sen e

tan e = -- ::::: e (radianes)ln eX = x cos e lag x = 0.43429 ln x El signo de una función trigonométrica depende del cuadrante, ln x = 2.3026 lag x o sea, de los signos de x y y; por ejemplo, en el segundo

Apéndice I A·l

Page 79: Fisica_I

cuadrante (-x, y), -x/r = cos 8 y y/r = sen 8. El signo también puede asignarse empleando las fórmulas de reducción.

Fórmulas de reducción (e en el segun- (e en el tercer (e en el cuarto do cuadrante) cuadrante) cuadrante)

sen 8 = cos(8 - 90°) = -sen(e - 180°) = -cos(8 - 270°) cose = -sen(e - 90°) = -cos(e - 180°) = sen(e - 270°)

Identidades fundamentales

sen2 e + cos2 8 = 1 sen 28 = 2 sen ecos e cos 28 = cos2 e - sen2 e = 2 cos2 8 - 1 = 1 - 2 sen2 8 sen2 8 = ~(1 - cos 2e) cos2 8 = ~ (1 + cos 2e)

Para las identidades de medio ángulo (8/2), sustituimos 8 por 8/2; por ejemplo,

sen2 e/2 = ~ (1 - cos e) cos2 e/2 = ~ (1 + cos e)

sen (a ± f3) = sen a cos f3 ± cos a sen f3 cos (a ± f3) = cos a cos f3 'f sen a sen f3

tan a =tan f3tan(a'f f3) = ------'--- ­

1 ::c tan a tan f3

Ley de los cosenos Para un triángulo con ángulos A, B YC cuyos lados opuestos son a, by c, respectivamente:

a2 = b2 + c2 - 2bc cos A

(con resultados similares para b2 = ... y para c2 = ...). Si A = 90°, esta ecuación se reduce al teorema de Pitágoras: a2 = b2 + c2 (de la forma 1'2 = x" + y2)

Ley de los senos Para un triángulo con ángulos A, B YC cuyos lados opuestos son a, b y c, respectivamente:

a b c sen A sen B sen C

Notación de potencias de la (científica) En física, muchos números son muy grandes o muy pequeños, para expresarlos, suele utilizarse la notación de potencias de 10 (científica). Cuando elenmos al cuadrado o al cubo el nú­mero 10, obtenemos

102 = 10 X 10 = 100 103 = 10 X 10 X 10 = 1000

Es evidente que el número de ceros es igual a la potencia de' 10. Por ejemplo, 1023 es un 1 seguido de 23 ceros.

También podemos usar potencias negati,'as de 10. Por ejemplo,

10-2 = _1_ = _1_ = O01 102 100 .

Por tanto, si una potencia de 10 tiene un exponente negativo, desplazamos el punto decimal a la izquierda una ,-ez por cada potencia de 10. Por ejemplo, 1 centimetro (cm) es 11100 m, o sea, 10-2 m, que es 0.01 m.

A-2 APÉNDICES

Hay muchas formas de representar un número en notación de potencias de 10, y todas son correctas. Por ejemplo, la distan­cia de la Tierra al Sol es 93 millones de millas. Este valor puede representarse como 93 000 000 millas, 93 X 106 millas, 9.3 X 107

millas o 0.93 X 108 millas. Todas estas representaciones son co­rrectas, aunque se prefiere 9.3 X 107 (En la notación de potencias de 10, se acostumbra tener sólo un dígito a la izquierda del pun­to decimal, a menos que intervengan cifras significativas.)

Así pues, vemos que el exponente, o potencia de 10, cambia cuando se desplaza el punto decimal del número que se escribe como prefijo. Las reglas generales para esta notación son:

Reglas para usar la notación de potencias de 10 1. El exponente, o potencia de 10, se incrementa en 1 por cada

posición que el punto decimal se desplaza a la izquierda.

2, El exponente, o potencia de 10, se decrementa en 1 por cada posición que el punto decimal se desplaza a la derecha.

Esto no es más que una forma de decir que, si el coeficiente (número prefijo) se hace más pequeño, el exponente tiene un aumento correspondiente, y ,-ice,-ersa. En total, el número es el mismo.

Ejemplo 1.• Expresar números en notación de potencias de 10

Exprese los números siguientes en notación de potencias de 10.

(a) 360000 (b) 246.7 (e) 0.0694 (d) 0.000011

Solución. Aplicamos las reglas anteriores para obtener:

(a) 3~Q = 3.6 x 103 (desplazar a la izquierda, regla 1) 5" 32'

(b) 2~.7 = 2467 x lO" (desplazar a la izquierda, regla 1) 21

(e) 0~.9.J = 6.94 x 10-2 (desplazar a la derecha, regla 2) 21

(d) 0.,9QQ261 = 11 x 10-5 (desplazar a la derecha, regla 2) ~ 2345

La notación de potencias de 10 es útil para expresar los re­sultados de operaciones matemáticas con el número correcto de cifras significati,-as. Por ejemplo, consideremos la operación

325 X 45 = 14625

Si expresamos el resultado con dos cifras significativas, obtenemos

325 x 45 = 1.5 x 10'

Ejemplo 2.• Uso de notación de potencias de 10 para expresar resultados de cálculos ­

Realice la operación siguiente con una calculadora y ex­prese el resultado correctamente, empleando cifras signi­ficati,-as y notación científica:

0.0024 = 7

8.05 .

Solución. Si hacemos esta operación en una calculado­ra, obtenemos

0.0024 = 0.000298136 8.05

Page 80: Fisica_I

(Nota: El número de dígitos en el resultado podría variar dependiendo de la calculadora.)

El número 0.0024 tiene dos cifras significativas, así que

0.0024 . -- = 3.0 X 10-<

8.05

Los procedimientos arihnéticos para multiplicar y dividir, así como para sumar y restar, se pueden efectuar en notación de potencias de 10.

Multiplicación de potencias de 10 En la multiplicación, se suman los exponentes.

Ejemplo 3. _

y

División de potencias de 10 Al dividir, el exponente del denominador se resta al exponente del numerador.

Ejemplo 4. _

8 4.8 X 10 = 2.0 X 106 2.4 X 102

y 8

3.4 X 10- = 2.0 X 10-6 1.7 X 10-2

Otro método para dividir consiste en transferir todas las po­tencias de 10 del denominador al numerador cambiando el signo del exponente. Luego se pueden sumar los exponentes de las potencias de 10, porque ahora se están multiplicando. Las partes decimales no se transfieren; se dividen de la forma acostumbrada. E(¡te método requiere un paso adicional, pero. muchos estudiantes sienten que los lleva de manera más con­sistente a la respuesta correcta. Así,

4.8 X 108 4.8 X 108 X 10-2 ------ = 2.0 X 106

2.4 X 102 2.4

Apéndice 11 Teoría cinética de los gases

Los supuestos básicos son:

1. Todas las moléculas de un gas puro tienen la misma masa (m) y están en movimiento continuo y totalmente aleatorio. (La masa de cada molécula es tan pequeña que el efecto de la gravedad sobre ella es insignificante.)

Cuadrado de potencias de 10 Al elevar al cuadrado números exponenciales, multiplicamos el exponente por 2. La parte decimal se multiplica por sí misma.

Ejemplo 5. _

(3 X 10'¡)2 = 9 X 108

(4 X 10-7)2 = 16 X 1O-;'¡

Raíz cuadrada de potencias de 10 Para obtener la raíz cuadrada de un número exponencial, seguimos la regla '\fiO" = 10m. Luego, podemos obtener 10m fácilmente con una calculadora (empleando la función 10'). Habrá ocasiones en que querremos tener una idea del orden de magnitud efectuando la raíz cuadrada mentalmente. En un caso así, expresamos '\fiO" de modo que la potencia de 10 sea un número para que se pueda dividir entre 2, y 10m será la raíz cuadrada. Se puede estimar la raíz cuadrada de un prefijo de una potencia de 10. He aquí un par de ejemplos en los que son e,'identes las raíces cuadradas de los prefijos.

Ejemplo 6. _

'\./9X108 = 3 X 10'; ,------:-::

2.5 X 10-1' = \/25 X 10-18 = 5 X 10-9

Suma y resta de potencias de 10 Al sumar o restar, los exponentes de 10 deben tener el mismo valor.

Ejemplo 7. _

4,6 X 10-8

+ 1.2 X 10-8

5.8 X 10-8

y

4.8 X 107

- 2.5 X lO'

2.3 X 10'

2. Las moléculas de gas están separadas por grandes distan­cias y ocupan un ,'olumen insignificante en comparación con esas distancias.

3. Las moléculas no ejercen fuerzas unas sobre otras, excep­to cuando chocan.

Apéndice 11 A·3

Page 81: Fisica_I

4. Los choques de las moléculas entre sí y con las paredes del nos de esta media, podemos escribir la ecuación 3 así: recipiente son perfectamente elásticas.

~Tm"2

La magnitud de la fuerza ejercida sobre la pared del reci­ p=~ (4) piente por una molécula de gas que choca con ella es F = fl.p / tit. Suponiendo que la dirección de la velocidad (vJ es normal a la Sin embargo, los movimientos de las moléc~s se~an con igual pared, la magnitud de la fuerza media es frecuencia a lo largo de los tres ejes, así que v; = v~ = v; Y

F = !l(mv) = mv~ - (-mv x ) 2mv, (1)

f'.t t.t ':'t

Después de chocar con una pared del recipiente, que por comodidad supondremos es un cubo con lados de longitud L, la molécula rebota en línea recta. Supongamos que la mo­lécula llega a la pared opuesta sin chocar con ninguna otra molécula en el camino. Entonces, la molécula recorre la distan­cia L en un tiempo igual a L/v,. Después del choque con esa pared, suponiendo otra vez que no hay choques en el camino de regreso, el viaje redondo tardará M = 2L/v,. Por tanto, el número de choques por unidad de tiempo, de una molécula con una pared dada es vj2L, y la fuerza media sobre la pared por choques sucesivos es

2mvx 2mv, mv~F=--=--=- (2)

t.t 2L/v, L

Los movimientos aleatorios de las muchas moléculas pro­ducen una fuerza relativamente constante sobre las paredes; la presión (p) es la fuerza total sobre una pared dividida entre el área de la pared:

(3)

Los subíndices se refieren a moléculas individuales. La media de los cuadrados de las velocidades está dada por

donde N es el número de moléculas en el recipiente. En térmi-

Apéndice 111 Datos planetarios

Masa (en Gravedad su-Radio comparación Densidad perficia: (e.1 com­

ecuatoriol con lo de media poración con ia Nombre (km) lo Tierra)' (x 103 kg(m 3) de /0 Tierra)

Mercurio 2439 0.0553 5.43 0.378 Venus 6052 0.8150 5.24 0.894 Tierra 6378.140 1 5.513 1 Marte 3397.2 0.1074 3.93 0.379 Júpiter 71398 317.89 1.36 2.54

2v = V; -'- ~ -;- ~ = 3V; . Entonces

W = V'lieaz

donde V,ñeaz es la rapidez eficaz o cuadrática media. Si sustitui­mos este resultado en la ecuación 4 y reemplazamos L3 por V (dado que L3 es el volumen del recipiente cúbico), obtenemos

: ,pV = 3' mV;lieaz (5)

Este resultado es correcto aunque se hizo caso omiso de los choques entre moléculas. Estadísticamente, estos choques se cancelan en promedio, de modo que el número de choques con cada pared es el descrito. Este resultado también es indepen­diente de la forma del recipiente. El uso de un cubo meramente simplifica la deducción.

Ahora combinamos este resultado con la ley empírica de los gases perfectos:

pV = NksT = ~:\Tmv~<;,az

La energía cinética media por molécula de gas es entonces pro­porcional a la temperatura absoluta del gas:

(6)

El tiempo de choque es insignificante en comparación con el tiempo entre choques. Algo de energía cinética se convertirá momentáneamente en energía potencial durante un choque; sin embargo, podemos hacer caso omiso de esta energía po­tencial, porque cada molécula pasa un tiempo insignificante chocando. Por tanto, con esta aproximación, la energía cinética total es la energía interna del gas, y la energía interna de un gas perfecto es directamente proporcional a su temperatura ab­soluta.

Inclinación respecto

Eíe semlmavo, ceriodo crbltol 010 x 10= km AU' Años Díos Excentricidad eclíptico

57.9 0.3871 0.24084 87.96 0.2056 7°00'26" 108.2 0.7233 0.61313 224.68 0.0068 3°23'40" 149.6 1 1.00004 365.25 0.0167 0°00'14" 227.9 1.5237 1.8808 686.95 0.0934 P51'09" 778.3 5.2028 11.862 4337 0.0483 1°18'29"

Saturno 60000 95.17 0.71 1.07 1427.0 9.5388 29.456 10760 0.0560 2°29'17" Urano 26145 14.56 1.30 0.8 2871.0 19.1914 84.07 3C700 0.0461 0°48'26"

Teptuno 24300 17.24 1.8 1.2 4497.1 30.0611 164.81 60200 0.0100 P46'27" Plutón 1500-1800 0.02 0.5-0.8 -0.03 5913.5 39.5294 248.53 90780 0.2484 1J009'03"

'Masa del planeta/masa de la Tierra, donde A1T = 6.0 X 102~ kg. tUnidad astronómica: 1 CA = 1.3 x 105 km, la distancia media entre la Tierra y el Sol.

A-4 APÉNDICES

Page 82: Fisica_I

Apéndice IV Lista alfabética de elementos químicos (la tabla periódica se da al final del libro)

Número Número Número atómico atómico atómico

(núm. de pro- Masa (núm. de pro- Mesa (núm. de pro- Mase Elemento Simbolo tones) atómica Eiemento Símboio tones) atómica Eiemento Símbolo tones) atómica

Actinio Ac 89 227.0278 Francio Fr 87 (223) Plata Ag 47 107.8682 Aluminio Al 13 26.98154 Gadolinio Gd 64 137.25 Platino Pt 78 195.08 Americio Am 95 (243) Galio Ga 31 69.72 Plomo Pb 82 207.2 Antimonio Sb 51 121.757 Germanio Ge 32 72561 Plutonio Pu 94 (244) Argón Ar 18 39.948 Hafnio Hf 72 178.49 Polonio Po 84 (209) Arsénico As 33 74.9216 Halmio Ha 105 (262) Potasio K 19 39.0983 Astato At 85 (210) Hassio Hs 108 (265) Praseodimio Pr 159 140.9077 Azufre S 16 32.066 Helio He 2 4.00260 Prometio Pm 61 (145) Bario Ba 56 137.33 Hidrógeno H 1 1.00794 Protactinio Pa 91 231.0359 Berkelio Bk 97 (247) Hierro Fe 26 35.847 Radio Ra 88 226.0254 Berilio Be 4 9.01218 Holrnio Ho 67 164.9304 Radón Rn 86 (222) Bismuto Bi 83 208.9804 Indio In 49 114.82 Renio Re 75 186.207 Bohrio Bh 107 (264) Iridio Ir 77 192.22 Rodio Rh 45 102.9055 Boro B 5 10.81 !terbio Yb 70 173.04 Rubidio Rb 37 85.4678 Bromo Br 35 79.904 Itrio Y 39 88.9059 Rutenio Ru 44 101.07 Cadmio Cd 48 112.41 Kriptón Kr 36 83.80 Rutherfordio Rf 104 (261) Calcio Ca 20 40.078 Lantano La 57 138.9055 Samario Sm 62 150.36 Californio Cf 98 (251) Lawrencio Lr 103 (260) Seaborgio Sg 106 (263) Carbono C 6 12.011 Litio Li 3 6.941 Selenio Se 34 78.96 Cerio Ce 58 140.12 Lutecio Lu 71 174.967 Silicio Si 14 28.0855 Cesio Cs 55 132.9054 Magnesio :vlg 12 24.305 Sodio ~a 11 22.98977 Cloro CI 17 35.453 YIanganeso i\tln 25 34.9380 Tantalio Ta 73 180.9479 Cromo Cr 24 51.996 Meitnerio Mt 109 (268) Tecnecio Tc 43 (98) Cobalto Co 27 58.9332 :v1endelevio Md 101 (258) Telurio Te 52 127.60 Cobre Cu 29 63.546 YIercurio Hg 80 200.59 Terbio lb 65 158.9254 Curio Cm 96 (247) Molibdeno Mo 42 95.94 Talio Tl 81 204.383 Dubnio Db 105 (262) :'\eodimio l\d 60 144.24 Torio Th 90 232.0381 Disprosio Dy 66 162.50 ="Jeón i\e 10 20.1797 Tulio Tm 69 168.9342 Einstenio Es 99 (252) \Jeptunio I'\p 93 237.048 Titanio Ti 22 47.88 Erbio Er 68 167.26 ~íquel l\i 28 58.69 Tungsteno W 74 183.85 Escandio Sc 21 44.9559 Niobio :\'0 41 92.9064 Uranio U 92 238.0289 Estaño Sn 50 118.710 Nitrógeno 1 7 14.0067 Vanadio V 23 50.9415 Estroncio Sr 38 87.62 Nobelio No 102 (259) Xenón Xe 54 131.29 Europio Eu 63 151.96 Oro Au 79 196.9665 Yodo I 53 126.9045 Fermio Fm 100 (257) Osmio Os 76 190.2 Zinc Zn 30 65.39 Flúor F 9 18.998403 Oxígeno O 8 15.9994 Zirconio Zr 40 91.22 Fósforo P 15 30.97376 Paladio Pd 46 106.42

Apéndice V Propiedades de isótopos selectos

Número Número Abur.dancia (%) o modo atómico de masa !vIase de aeslntegración~ Semivida

(l) Elemento Símbolo (A) atómica" {s: es radiactivo) (si es radiactivo)

O (Neutrón) n 1 1.008665 ¡r 10.6 min 1 Hidrógeno H 1 1.007825 99.983

Deuterio D 2 2.014102 0.015 Tritio T 3 3.016049 ¡r 12.33 a

2 Helio He 3 3.016029 0.00014 4 4.002603 :::::100

3 Litio Li 6 6.015123 7.5 7 7.016005 92.5

4 Berilio Be 7 7.016930 CE, 'Y 53.3 d

Apéndice V A-S

Page 83: Fisica_I

Número Número Abundancia (%) o modo atómico de maso Maso de desintegración' Semivida (si

(Z) Elemento Símbolo (A) atómico' (si es radiactivo) es radiactivo)

8 8.005305 20' 6.7 X 10-17 S

9 9.012183 100 5 Boro B 10 10.012938 19.8

11 11.009305 80.2 12 12.014353 ¡r 20.4 ms

6 Carbono C 11 11.011433 ¡r, CE 20.4ms 12 12.000000 98.89 13 13003355 1.11 14 14.003242 ¡r 5730 a

7 itrógeno N 13 13.005739 ¡r 9.96 mm 14 14.003074 99.63 15 15.000109 0.37

8 Oxígeno O 15 15.003065 ir, CE 122 s 16 15.994915 99.76 18 17.999159 0.204

9 Flúor F 19 18.998403 100 10 ="león Ne 20 19.992439 90.51

22 21.991384 9.22 11 Sodio Na 22 21.994435 ir, CE, "Y 2.602 a

23 22.989770 100 24 23.990964 ¡r, "Y 15.0h

12 Magnesio Mg 24 23.985045 78.99 13 Aluminio Al 27 26.981541 100 14 Silicio Si 28 27.976928 92.23

31 30.975364 ¡r, "Y 2.62h 15 Fósforo P 31 30.973763 100

32 31.973908 ¡r 14.28 d 16 Azufre S 32 31.972072 95.0

35 34.969033 ¡r 87.4 d 17 Cloro Cl 35 34.968853 75.77

37 36.965903 24.23 18 Argón Ar 40 39.962383 99.60 19 Potasio K 39 38.963708 93.26

40 39.964000 ¡r, CE,"y, ¡r 1.28 X 109 a 20 Calcio Ca 30 39.962591 96.94 24 Cromo Cr 52 51.940510 83.79 25 Manganeso Mn 55 54.938046 100 26 Hierro Fe 56 55.934939 91.8 27 Cobalto Co 59 58.933198 100

60 59.933820 ¡r, "Y 5.271 a 28 Níquel Ni 58 57.935347 68.3

60 59.930789 26.1 64 63.927968 0.91

29 Cobre Cu 63 62.929599 69.2 64 63.929766 ¡r,¡r 12.7h 65 64.927792 30.8

30 Zinc Zn 64 63.929145 48.6 66 65.926035 27.9

33 Arsénico As 75 74.921596 100 35 Bromo Br 79 78.918336 50.69 36 Kripton Kr 84 83.911506 57.0

89 88.917563 ¡r 3.2 mm 38 Estroncio Sr 86 85.909273 9.8

88 87.905625 82.6 90 89.907746 ¡r 28.8 a

39 Itrio Y 89 89.905856 100 43 Teenecio Te 98 97.907210 ¡r, "Y 4.2 X 106 a 47 Plata Ag 107 106.905095 51.83

A-6 APÉNDICES

Page 84: Fisica_I

Número Número Abundancia (%) o modo atómico de masa Masa de desintegraciónt Semivida (si

(l) Elemento Símbolo (A) atómica * (si es radiactivo) es radiactivo)

109 108.904754 48.17 48 Cadmio Cd 114 113.903361 28.7 49 Indio In 115 114.90388 95.7; rr 5.1 X a1014

50 Estaño Sn 120 119.902199 32.4 53 Yodo 1 127 126.904477 100

131 130.906118 rr,y 8.04 d 54 Xenón Xe 132 131.90415 26.9

136 135.90722 8.9 55 Cesio Cs 133 132.90543 100 56 Bario Ba 137 136.90582 11.2

138 137.90524 71.7 144 143.92273 rr 11.9 s

61 Prometio Pm 145 144.91275 CE, a,y 17.7 a 74 Tungsteno (wolframio) W 184 183.95095 30.7 76 Osmio Os 191 190.96094 rr,y 15.4 d

192 191.96149 41.0 78 Platino Pt 195 194.96479 33.8 79 Oro Au 197 196.96656 100 80 Mercurio Hg 202 201.97063 29.8 81 Talio TI 205 204.97441 70.5

210 209.990069 rr 1.3 min 101782 Plomo Pb 204 203.973044 rr,1.48 1.4 X a

206 205.97446 24.1 207 206.97589 22.1 208 207.97664 52.3 210 209.98418 a,rr,y 22.3 a 211 210.98874 rr,y 36.1 min 212 211.99188 rr,y 10.64 h 214 213.99980 rT,y 26.8 min

83 Bismuto Bi 209 208.98039 100 211 210.98726 a, {3-, y 2.15 min

84 Polonia Po 210 209.98286 a,y 138.38 d 214 213.99519 a,y 164 ¡.LS

86 Radón Rn 222 222.017574 a,{3 3.8235 d 87 Francia Pr 223 223.019734 a,{3-,y 21.8 min

88 Radio Ra 226 226.025406 a,y 1.60 X 103 a 228 228.031069 {3- 5.76 a

89 Actinio Ac 227 227.027751 a,{3-,y 21.773 a 90 Torio Th 228 228.02873 a,y 1.9131 a

232 232.038054 100; a, y 1.41 X 1010 a 92 Uranio U 232 232.03714 a,y na

233 233.039629 a,y 1.592 X 105 a 235 235.043925 0.72; a, y 7.038 X 108 a 236 236.045563 a, 'Y 2.342 X 107 a 238 238.050786 99.275; a, y 4.468 X 109 a 239 239.054291 {3-,y 23.5 min

93 Neptunio Np 239 239.052932 {3-,y 2.35 d 94 Plutonio Pu 239 239.052158 a,y 2.41 X 104 a 95 Americio Am 243 243.061374 a,y 7.37 X 103 a 96 Curio Cm 245 245.065487 a,y 8.5 X 103 a 97 Berkelio Bk 247 247.07003 a,y 1.4 X 103 a 98 California Cf 249 249.074849 a,y 351 a 99 Einstenio Es 254 254.08802 a,y,{3- 276 d

100 Permio Pm 253 253.08518 CE,a,y 3.0 d

*Las masas dadas en toda esta tabla son las del átomo neutro; incluyen los Z electrones. t"CE" significa captura de electrones.

Apéndice V A-7

Page 85: Fisica_I

Respuestas a Ejercicios de refuerzo

Capítulo 2

2.1 (a) No. Si se usan metros por minutos, la respuesta estará en minutos. (b) 0.200 mlmin (o 0.00333 mis). 2.2 (a) 51 = 2.00 mis; 52 = 1.52 mis; 53 = 1.72 mis "* O, aunque la velocidad es cero. 2.3 No. Si la velocidad también es en la dirección negativa, el objeto se acelerará. 2.4 9.0 mis en la dirección del movimiento original. 2.5 Sí, 96 m. (Mucho más rápido, ¿no?) 2.6 No, cambia la posición .lo, pero la distancia de separación es la misma.

2.7 VB = 2vA' 2.8 x = v2/2a, .lB = 48.6 m y Xc = 39.6 m; el Blazer no deberá acercarse a menos de 9.0 m. 2.9 1.16 s más. 2.10 Tiempo para que el billete caiga su longitud = 0.179 s. Es­to es menor que el tiempo de reacción medio (0.192 s) calcu­lado en el Ejemplo, así que la mayoría de la gente no atrapa el billete. 2.11 Yu = Yd = 5.12 m, medido desde la referencia y = Oen el punto donde se soltó. 2.12 Ecuación 2.8', t = 4.6 s; ecuación 2.10', t = 4.6 5.

A8 Respuestas a Ejercicios de refuerzo

Page 86: Fisica_I

Respuestas a Ejercicios impares

Capítulo 2 1. (a) escalar, (b) vectorial, (e) escalar,

(d) vectorial. 3. (a).

5. Sí, en un viaje redondo. No; la distancia siempre es mayor que la magnitud del despla­zamiento, o igual.

7. La rapidez es la magnitud de la velocidad. 9. La distancia recorrida es 300 m o mayor.

11. 1.65 m hacia abajo. 13. 30 km, no. 15. (a) 0.50 mis (b) 8.3 mino 17. (a) entre 40 y 60 m, pues ningún lado de

un triángulo puede ser mayor que la suma de los otros dos lados. La magnitud del des­plazamiento es la hipotenusa del triángulo rectángulo, así que no puede ser menor que el más largo de los lados mutuamente perpendi­culares. (b) 45 m 27° al oeste del norte.

19. (a) 2.7 cmls (b) 1.9 cm/s. 21. (a) 7.35 s (b) no. 23. (a) 50-2.05 = 1.0 mis; 52.05-3.05 = O;

530s-45s = 1.3 mis; 5 4.3 5-6.3 s = 2.8 mis; 565s-75s = O; 57.55-9.05 = 1.0 mis (b) VO-20s = 1.0 mis; V20s-3.0s = O; V30s-45s = 1.3 mis; V4.5s-63s = -2.8 mis; V65s-75s = O; V7.3s-9.0s = 1.0 mis (e) V¡Os = 50-2.05 = 1.0 mis; V2.5s = 52.05-3.05 = O; V-i.5s = O; V6.0s = 54.55-6.55 = -2.8 mis (d) V45s-9.0s = -0.89 mis. 25. 8.7 x 102 km. 27. 59.9 mi/h; no. 29. imposible, porque la rapidez media a ca­

sa tendría que ser infinita. 31. Sí, aunque la rapidez del automóvil es

constante, su velocidad no lo es debido al cambio de dirección. 33. Vo.

35. Sí. Por ejemplo, un objeto que se mueve en el eje +x (velocidad positi\'a) se frena (ace­leración negativa o aceleración en el eje -x).

37. 1.85 m/s2.

39. 3.7 s. 41. 36m.

43. aO_4 = 2.0 m/s2; a4-10 = O;

aJO-lB = -1.0m/s2. .

45. 150 S.

47. (d).

49. (a).

51. no, 9.9 m/s2 .

53. (a) 3.5 m/s2 (b) 4.5 S.

55. (a) 81.4 kmlh (b) 0.794 S.

57. 3.09 s Y13.7 S. Después de 3.09 s, está a 175 m de donde se aplicó el empuje en reversa. Sin embargo, si ese empuje se aplica continuamente (posible, aunque poco probable), invertirá su di­rección y regresará a 175 m del punto en el que se aplicó, tardando 13.7 s en llegar ahí.

59. (a) viaja en la dirección +x y luego invier­te su dirección. El objeto tiene una \'elocidad inicial en la dirección +x y se requiere tiempo para que el objeto desacelere y se detenga, y luego invierta su dirección. (b) 23 s (e) 40 mis en-x.

61, 1.43 x 10-4 S.

63. (a) 30 mN (b) 75 mis. 65. no, x = 13.3 m. 67. (a) -12 mis; -4.0 mis (b) -18 m

(e) 50 m. 69. (a) xJ-posición inicial, x-posición finol,

va·velocidad inicial, u·velocidad final, a-acele­ración, t-intervalo de tiempo (b) La condición en la que estas ecuaciones se cumplen es que el mo\'irniento tenga aceleración constante.

71. (d).

73. cero y 9.8 miS2 hacia abajo. 75, La pelota se mueve con \'elocidad

constante, 77. 78.4 m. 79. (a) Cuatro veces, pues el desplazamiento

es proporcional al cuadrado del tiempo (b) 15.9 m y 4.0 m.

81. no es un buen negocio. 83. 67 m. 85. poco menos de 8.0 mis. 87. (a) 48 m (b) 38 mis hacia abajo. 89. (a) el mismo, 1.14 s (b) 1l.2m/s,-1l.2m/s

(iguales pero opuestas).

91. (a) menos del 930,0, pues la altura depen­de del cuadrado de la \'elocidad (b) 3.61 m.

93. cae 14 cm adelante de la profe. 95. (a) V6, pues el inter"alo de tiempo de­

pende de la raíz cuadrada de la aceleración (b) Tierra: 16.5 m, 3.67 s; Luna: 99.2 m, 22.0 S.

97. (a) -20.9 mis (b) 2.87 S.

99. (a) 21 mis (b) 19 m.

101. 45 mi/h. 103. (a) mayor que R pero menor que 2R. En todo triángulo rectángulo, la hipotenusa es mayor que cualquiera de los dos lados mutua­mente perpendiculares (R) \' menor que la su­ma de los dos lados perpendiculares (R + R = 2R). (b) 71 m. 105. 16.6 mis. 107. 1.2 x 102 m. 109. (a) 27 mis (b) 4.3 s

111. (a) El sonido tarda en viajar del fondo del pozo a la persona. (b) 51.7 m. 113. (a) 5.0 m/s 2 (b) 13 mIs. (e) e'

25 mis

5 s

(d) y

62.5 m

Ss

115. 12 km/h.

Respuestas a Ejercicios impares A9

Page 87: Fisica_I

La tabla periódica de los elementos

..--5--'· GRUPO GRUPO

d - PGRUPOIGRUPOIGRUPOIGRUPOIGRUPO GRUPO 1 11 Elementos de transición HI IV V VI VII VIII

1 H 2 He 1.01 4.00

151 152

3 Li 4 Be 5 B 6 C 7 N 8 O 9 F 10 Ne 2 6.94 9.01 10.81 12.01 14.0 I 16.00 19.00 20.18

251 2s2 217 1 2172 2173 2174 2175 2716

11 Na 12 Mg 13 Al 14 Si 15 P 16 S 17 CI 18 Ar 3 22.99 24.31 26.98 28.09 30.97 32.07 35.45 39.95

(J) 351 352 317 1 3172 3173 3174 317 5 3176 O o 19 K 20 Ca 21 Se 22 Ti 23 V 24 Cr 25 Mn 26 Fe 27 Co 28 Ni 29 Cu 30 Zu 31 Ga 32 Ge 33 As 34 Se 35 Br 36 Kr O 4 39.10 40.08 44.96 47.88 50.94 52.00 54.94 55.85 58.93 58.69 63.55 65.39 69.72 72.61 74.92 78.96 79.90 83.80 a: w Q..

4s1

37 Rb 452

38 Sr 3d1452 3d24s2

39 'y 40 Zr 3d345 2

41 Nb 3d545 1

42 Mo 3d545 2

43 Te 3d545 2

44 Ru 3d7452

45 Rh 3d84s2

46 Pd 3d1045 1

47 Ag 3d10452 417 1

48 Cd 49 In 4172

50 Su 4173

51 Sb 4p4

52 Te 4175

53 I 4176

54 Xe ,5 85.47 87.62 88.96 91.22 92.91. 95.94 (98) 10 l.07 102.91 106.42 107.87 112.41 114.82 118.71 121.76 J27.60 126.90 131.29

551 552 4d15s2 4d255 2 4d455 1 4d55s 1 4d5 5s2 4d75s1 4d8 5s 1 4d105s6 4d105s 1 4d105s2 517 1 5172 5173 5174 5175 5176

i6 55 Cs 132.91

56 Ba 57 La 72 Hf 73 Ta 137.33 138.91 * 178.49 180.95

74 W 183.85

75 Re 186.21

76 Os 190.2

77 Ir 192.22

78 Pt 195.08

79 Au 196.97

80 Hg 200.59

81 TI 204.36

82 Pb 207.2

83 Bi 208.98

84 Po (209)

85 At (210)

86 Rn (222)

65 1 6s2 5d16s 2 5d2652 5d3 6s 2 5d465 2 5d56s2 5d6 6s2 5d765 2 5d96s1 5d106s' 5d106s2 617 1 617 2 6173 6174 6p5 6176

,7

87 Fr (223)

88 Ra 226.03

89 Ae 104 Rf 105 Db 106 Sg 107 Bh 108 Hs 109 Mt 110 227.03 t (261) (262) (263) (264) (265) (268) (269)

111 (272)

112 (277)

751 752

/

6d17s2 6d27s2

-­6d37s 2 6d47s2 6d575 2 6d67s2 6d77s2

58.85 -1- Masa atómica

NLJmero~ . atómico 2?" ..~e 51mbolo

/ /

Configuración -~3d6452

/ / electrónica exterior

/

1 -/

/7 f --.---­ =­ _ -1 /

-58 Ce 59 Pr 60 Nd 61 Pm 62 Sm 63 Eu 64 Gd 65 Tb 66 Dy 67 Ho 68 Er 69 Tm 70 Yb 71 Lu

* 140.12 J4ü.91 144.24 ( 145) 150.36 151.96 157.25 ISR.93 IG2.50 164.93 167.26 lN!.93 173.04 174.97 (Lantánidos) 5d 14f'6s2 4j36s2 4j46s2 4(5652 4f66s2 4j76s2 5d14P652 5d14f86s 4j106¡;'2 4/ 11 6s2 4f126s2 4f13 652 4f14 6s 2 5¡jl4j1 46s2

90 Th 91 Pa 92 U 93 Np 94 Pu 95 Am 96 Cm 97 Bk 98 Cf 99 Es 100 Fm 101 Md 102 No 103 Lr t 232.04 231.04 238.03 237.05 (244) (243) (247) (247) (251) (252) (257) (25(i) (259) (260) (Actinidos)

6d27s2 5J26d 17s 2 5J36d17s 2 5f46d,'7s2 5f66do7s2 5J76do7s2 5J76d17s 2 5f8 6d 17s2 5f06d07s2 5S11 6d07s2 5j126d07s2 5f136d0752 5f146d075 2 5J146d17s2

Page 88: Fisica_I

UNIVERSIDAD AUTÓNOMA DE NUEVO LEÓN

SECRETARÍA ACADÉMICA

DIRECCIÓN DE ESTUDIOS DEL NIVEL MEDIO SUPERIOR

Física Primera edición

Manual de experime

(Galileo Galilei)

Comité de Física

M.E.e. Ernesto Araujo Castillo' M.e. Ludovico González Simián

M.E.e. Luciano Escamilla Reyes M.e. Carlos Mata Martínez

M.e. César González Simián M.E.e. Moisés Navarro Guzmán

®

México' Argentina' Brasil' Colombia' Costa Rica' Chile' Ecuador España' Guatemala' Pana11Ú • Perú' Puerto Rico' Uruguay' Venezuela

Page 89: Fisica_I

/ Datos de catalogación bibliográfica

Araujo, Escamilla, González, González, Mata y Navarro

Física 1. Manual de experimentos. Primera edición

PEARSON EDUCACIÓN, México, 2008

ISBN: 978-970-26-1172-1 Área: Bachillerato

Formato: 21 x 27 cm Páginas: 56

Todos los derechos reservados 2008 Universidad Autónoma de uevo León Esta edición en español es la única autorizada

Edición en español Editor: Hugo Rivera üliver

e-mail: [email protected] Editor de desarrollo: Bernardino Gutiérrez Hernández Supervisor de producción: Enrique Trejo Hernández

PRIMERA EDICIÓN, 2008

D.R. © 2008 por Pearson Educación de México, S.A. de cv. Atlacomulco 500-50 Piso Col. Industrial Atoto CP. 53519, Naucalpan de Juárez, Edo. de México

Cámara Nacional de la Industria Editorial Mexicana. Reg. Núm. 1031.

Prentice Hall es 'una marca registrada de Pearson Educación de México, S.A. de Cv.

Reservados todos los derechos. Ni la totalidad ni parte de esta publicación pueden reproducirse, registrarse o transmitirse, por un sistema de recuperación de información, en ninguna forma ni por ningún medio, sea electrónico, mecánico, fotoquímico, magnético o electroóptico, por fotocopia, grabación o cualquier otro, sin permiso previo por escrito del editor.

El préstamo, alquiler o cualquier otra forma de cesión de uso de este ejemplar requerirá también la autori­zación del editor o de sus representantes.

ISBN 10: 970-26-1172-5 ISBN 13: 978-970-26-1172-1

Impreso en México. Printed in Mexico. 1 2345 6 789 O - 10 09 08

PEARSON

Educación ®

Page 90: Fisica_I

CONTENIDO

PRÓLOGO v

MANUAL DE EXPERIMENTOS

EXPERIMENTO 1. Investigación de un fenómeno 1

EXPERIMENTO 2. Uso del Vernier 7

EXPERIMENTO 3. Laboratorio de gráficas l 11

EXPERIMENTO 4. Laboratorio de gráficas TI 15

EXPERIMENTO 5. Movimiento rectilíneo uniforme 19

EXPERIMENTO 6. Movimiento rectilíneo uniforme acelerado 23

EXPERIMENTO 7. Caída libre de los cuerpos 33

GUÍA DEL ESTUDIANTE

Autoevaluación 39

Físíall/Manual de experimentos ili

Page 91: Fisica_I

PRÓLOGO

Las nuevas orientaciones que rigen en los últimos años la enseñanza de la física en los países técnicamente más desarrollados se centran en tres puntos primordiales:

1) Un aumento sustancial del tiempo dedicado al trabajo experimental. 2) Que los experimentos no sean solamente observados en demostraciones de

cátedra, sino que sean realizados directamente por los alumnos, trabajando en equipos o brigadas.

3) Que estos experimentos sean cuantitativos.

De acuerdo con el espíritu de estas orientaciones y cumpliendo uno de los objetivos fundamentales de nuestra Universidad -donde la superación académica es cada vez más dinámica-, se ha recopilado el presente trabajo con el fin de ayudar a los estudiantes del bachillerato a obtener un conocimiento integral de la física.

El desarrollo de cada práctica de laboratorio sigue el proceso que se muestra a continuación:

Primero: Los experimentos se basan en todo momento en la temática tratada por el libro de Física 1.

Segundo: El profesor da una explicación general de la práctica y si es necesario, brinda la asesoría correspondiente para fundamentar, motivar o acla­rar las dudas que surjan.

Tercero: Las prácticas están diseñadas para plantear al alumno situaciones y formularle una serie de interrogantes que lo obliguen a analizar los resultados y, por tanto, se le pide la elaboración de tabulaciones y gráficas para entender y visualizar más claramente la relación que existe entre las variables involucradas en el experimento, facilitándole, de este manera, la interpretación de los resultados.

Cuarto: Terminado el trabajo experimental, los estudiantes, bajo la dirección del profesor, analizan y discuten los resultados obtenidos y logran llegar a la conclusión correspondiente sobre el trabajo realizado.

Estas prácticas no se exceden en teoría ni tampoco intentan crear un manual de pro­cedimientos, únicamente nos limitamos a enunciar los objetivos, el material necesa­rio y el contenido informativo básico que el alumno debe tener en el laboratorio.

Es importante mencionar que existen muchos trabajos de esta índole, y tal vez muy buenos, pero no ha habido una verdadera difusión de los mismos y menos aún

Física l/Manual de experimentos v

Page 92: Fisica_I

una estadística de su efectividad. Por otro lado, sabemos de las insuficiencias que subyacen en el presente trabajo, las cuales nos proponemos ir emiqueciendo de ma­nera sistemática en las próximas ediciones. Por lo que, esperamos que tanto alum­nos como maestros consideren en este sentido la presente edición.

Comité de Física

M.E.e. Emeto Arauja Castillo

M.E.e. Luciano Escamilla Reyes

M.e. César González Simián

M.e. Ludovico González Simián

M.e. Carlos Mata Martinez

M.E.e. Moisés Navarro Guzmán

Monterrey, N.L., agosto de 2007.

vi Física l/Manual de experimentos

Page 93: Fisica_I

Nombre: _ Grupo: __ Fecha: _

Experimento 1

Investigación de un fenómeno. El movimiento l/Si buscas resultados distintos, no hagas siempre lo mismo!!.

Albert Einstein

Objetivo Establecer una hipótesis y desarrollar la metodología necesaria para la comproba­ción, o verificación de ésta.

Análisis teórico ¿Quién es un científico? Un científico es un ser inquisitivo, desea saber qué clase de mundo es éste, qué clase de cosas hay en él, cómo se comportan y si existen reglas básicas en su comportamiento. Busca entender los fenómenos naturales y las leyes que los explican, utilizando el método científico y en particular, el método científico experimental. Un experimento es una pregunta que se hace a la naturaleza e impli­ca siempre una hipótesis. Las hipótesis deben ser siempre susceptibles de compro­bación a través del experimento. No deben, en general, diferir con los hechos o principios establecidos. Puede haber ocasiones en que una hipótesis, por ser revolu­cionaria, pueda cambiar los principios establecidos en todo un campo de la ciencia. Entonces, será preciso cambiar los principios previamente adoptados, los que no coinciden con la nueva teoría.

Material 1 Cinta métrica. 1 Bloque de madera. 1 Carrito. 2 Bolas metálicas. 1 Base soporte. 1 Varilla soporte. 1 Juego de pesas. 1 Mesa.

Procedimiento El científico empieza siempre haciéndose una pregunta. En nuestro experimento la pregunta es: ¿cómo se mueven los objetos? Si fueras un científico experimentado, con esta pregunta te sería suficiente para empezar una investigación, pero como no lo eres, habrá que sugerirte algunos de los posibles pasos de la investigación.

Existen métodos para enfocar un problema científico y no hay dos científicos que lo hagan exactamente de la misma forma. Puedes empezar el estudio del movimien­to reuniendo todos los hechos que consigas con relación a los cuerpos en movimiento

Física 1/:vIanual de experimentos 1

Page 94: Fisica_I

y después tratar de organizarlos para que tengan sentido. En un inicio es posible hacer una conjetura con respecto a la pregunta de ¿cómo se mueven los objetos? A esta conjetura se le llama hipótesis y podría plantearse de la siguiente manera:

Hipótesis: Los cuerpos se mueven cuando se les empuja o se tira de ellos.

Tal vez sea cierta, o tal vez no, para saberlo con exactitud se deberá demostrar.

Paso 1. Coloca un bloque liso de madera encima de la mesa y dale un rápido empu­jón con la mano, sin mantenerla en contacto con el bloque. ¿Se mueve el blo­que cuando lo tocas con tu mano o continúa moviéndose después de haberla quitado? ¿Qué le sucede al bloque al dejar de estar en contacto con la mano?

Como resultado de esta investigación probablemente es necesario revisar la hipóte­sis y sustituirla con la siguiente:

Primera hipótesis revisada: Los cuerpos se mueven cuando se les empuja o se tira de ellos y disminuye su rapidez cuando ya no actúa la fuerza.

Paso 2. Coloca el carrito encima de la mesa, dale un empujón y mide la distancia que recorre. Luego, ubícalo de tal forma que quede con las ruedas al aire; empú­jalo de nuevo con lo que parezca ser la misma fuerza y mide la distancia que recorrió. ¿Cómo son las distancias, iguales o diferentes? _

Esta parte de la investigación no es muy exacta, ya que no puedes estar muy se­guro de que cada vez empujaste el carrito con la misma fuerza, por lo tanto, repite el experimento como se indica en el paso 3.

Paso 3. Suspende una bola metálica de un soporte por medio de una cuerda como se muestra en la figura 1. Cuando la bola cuelgue sin mo"erse, coloca el ca­rrito de modo que apenas toque la bola; después, desplaza la bola hacia atrás hasta una altura de 15 centímetros, con respecto a la mesa; manteniendo ti­rante la cuerda, luego suelta la bola y mide la distancia recorrida por el ca­rrito. Realiza otras dos mediciones y determina el promedio de las tres medidas. Anota los datos en la tabla 1. Coloca el carrito en la posición iniciat sólo que con las ruedas hacia arriba, y repite el experimento soltando la bo­la desde la misma altura. Realiza otras dos mediciones y determina el pro­medio de las tres medidas. Anota los datos en la tabla 1.

Física l/Manual de experimentos 2

Page 95: Fisica_I

Nombre: _ Grupo: _ Fecha:

Figura 1

Como verás posteriormente en el curso de física, al iniciar la oscilación de la bola desde la misma altura, el carrito recibe cada vez el mismo empuje. Como resulta­do del empuje el carrito se mueve, y en todos los casos su velocidad disminuye cuando se suprime la fuerza de éste. La hipótesis establecida parece ser efectiva.

Sin embargo, la distancia que recorre el carrito con las ruedas hacia abajo no es la misma que recorre con las ruedas hacia arriba, a pesar de que en ambos casos se usó el mismo objeto, el carrito, y que, además recibió el mismo empuje. La diferen­cia en la disminución de la rapidez debe, en consecuencia, tener algo que ver con el contacto entre las áreas del carrito y la mesa, es decir, ésta debe presentar resisten­cia al movimiento del carrito y la resistencia (o fuerza de rozamiento) debe ser me­nor cuando se emplean las ruedas.

Por experiencias anteriores, ya debes estar familiarizado con esta clase de resis­tencia por rozamiento, por ejemplo al intentar mover un mueble pesado: cuando el mueble toca el piso, se siente una fuerza que se opone al movimiento, pero si las pa­tas tienen ruedas, la fuerza que se opone es menor.

La hipótesis anterior necesita otra revisión, para tomar en cuenta la fuerza de ro­zamiento.

Segunda hipótesis revisada: Los cuerpos se mueven cuando se les empuja o se tira de ellos, y su movimiento disminuye cuando se retira la fuerza y llegan más pronto al reposo cuando es mayor el rozamiento que cuando éste es menor.

Física 1/Manual de experimentos 3

Page 96: Fisica_I

Nota: Con el procedimiento del paso 3¡ la hipótesis anterior puede estar demostra­da. Como un ejercicio extra puedes probar la hipótesis teniendo otro cuerpo (otra bola) que pese lo mismo que el carrito y que deba llegar más lejos¡ cuan­do reciba el mismo empuje. Con una bola más pesada que el carrito repite el paso 3¡ usando esta vez la bola como un cuerpo y el carrito como otro. (Pre­viamente pesa la bola de acero y agrega pesas al carrito hasta que pese lo mis­mo que la bola.) Anota las medidas en la tabla 2.

Tabla 1 Distancia recorrida por el carrito Distancia recorrida por el carrito con las ruedas hacia arriba con las ruedas hacia abajo

Ensayo 1 Ensayo 1

Ensayo 2 Ensayo 2

Ensayo 3 Ensayo 3

Suma Suma

Promedio Promedio

Tabla 2 Distancia recorrida por el carrito Distancia recorrida por la bola con las ruedas hacia arriba

Ensayo 1 Ensayo 1

Ensayo 2 Ensayo 2

Ensayo 3 Ensayo 3 Suma Suma Promedio Promedio

Reporte de trabajo

Nombre: Grupo: _

Turno: Fecha: Revisado:

Maestro: _

Física l/Manual de experimentos 4

Page 97: Fisica_I

Nombre: _ Grupo: __ Fecha:

Toma en cuenta los datos que obtuviste cuando respondas a las siguientes preguntas.

1. ¿Había fuerza de rozamiento cuando el carrito se movía sobre sus ruedas?

2. ¿Cómo lo sabes?

3. ¿Es posible que la fuerza de rozamiento actúe sobre la bola rodando? ¿Por qué?

4. A medida que la fuerza de rozamiento disminuya, ¿qué sucede con la distancia recorrida por un cuerpo en movimiento?

5. Si la fuerza de rozamiento se hubiera reducido a cero, y si ninguna otra fuerza exterior actuara sobre un cuerpo en movimiento, ¿qué pasaría con el movimien­to del cuerpo?

A las preguntas como ésta se les llama"experimentos imaginarios". En realidad, no es posible alcanzar una situación donde el rozamiento sea igual a cero, pero se pueden investigar situaciones determinadas en las que el rozamiento vaya disminuyendo. Por tanto, en el experimento imaginario se ha ido más allá de las situaciones reales y lo que es posible deducir que sucede es, probablemente, lo que pasaría en realidad si no hubiera rozamiento.

6. Discute los métodos utilizados en este experimento, contestando las siguientes preguntas:

a) ¿Por qué es útil tener una hipótesis?

b) ¿Por qué es mejor el método seguido en el paso 3 que en el paso 2?

Física l/Manual de experimentos 5

Page 98: Fisica_I

c) ¿Por qué es mejor hacer tres mediciones y tomar el promedio en lugar de ha­cer una sola medición?

d) ¿Cómo se prueba una hipótesis?

Física l/Manual de experimentos 6

Page 99: Fisica_I

Nombre: _ Grupo: _ Fecha:

Experimento 2

Uso del Vernier "El problema del hombre no está en la bomba atómica¡ sino en su corazón".

Albert Einstein

Objetivo Que el alumno utilice el Vernier para medidas exteriores¡ interiores¡ de profundidad y de espesor en diferentes cuerpos geométricos.

Análisis teórico Todo instrumento de medición tiene un grado de precisión determinado¡ dependien­do de la mínima medición que pueda hacerse con él. Por ejemplo¡ una regla de un metro de largo graduada en centímetros¡ será menos precisa que otra graduada en milímetros y ésta a su vez será menos precisa que un Vernier graduado en décimas de milímetro.

La selección de un instrumento dependerá de lo que se vaya a medir y del gra­do de precisión que se requiera.

El Vernier consta¡ de acuerdo con la figura 1¡ de dos partes: una fija en la cual está la graduación en centímetros y milímetros y otra móvil con un botón liberador y con una graduación en décimas de milímetro. Este instrumento¡ permite realizar tres tipos fundamentales de mediciones:

a) Medidas de grosores¡ diámetros o dimensiones exteriores. b) Medidas de diámetros o dimensiones interiores. c) Medidas de profundidad o alojamientos interiores.

En este experimento vamos a usar el Vernier para medidas exteriores¡ interiores y de profundidad de varias piezas. Convendrá que todos los alumnos tomen piezas igua­les para comparar los valores obtenidos en cada lectura.

Material 1 Vernier. 1 Regla. 1 Valero. 1 Cilindro hueco.

Procedimiento Se abre el calibre del Vernier utilizando el botón y desplazando la parte móvil lo su­ficiente para que la pieza cuya dimensión se desea conocer pueda ser abarcada por éste. Una vez colocada la pieza se cierra el calibre hasta que pueda ser sujetada con cierta presión.

Física l/Manual de experimentos 7

Page 100: Fisica_I

En la escala fija se leen los centímetros que hay desde el cero de esta escala has­ta el cero o primera raya de la escala móvil. Se mide tan pronto como la división de la escala móvil se aproxima más a una división de la escala fija y este número de la escala móvil son los milímetros que hay que añadir, a los centímetros ya leídos, pa­ra obtener una medición más exacta.

Para entender mejor lo anterior, se ejemplifica la medición en la figura 1.

mediciones interiores

cursor

3 4 5 6 7 'S 9 10 11 12 13 14

tescala graduada mediciones de profundidad

patilla fija

Figura 1

En la figura se observa que la pieza medida tiene más de 2.8 centímetros ya que el cero de la escala móvil no coincide exactamente con e12.8 de la escala fija. Para en­contrar con exactitud la magnitud medida, buscamos la división de la escala móvil que coincida con una de la escala fija y vemos que coincide el .05, por tanto, la lon­gitud o medida indicada es de 2.805 cm.

Nota: Para medir el diámetro exterior de una pieza se usa la sección marcada como patillas inferiores.

Para hacer la medición de un diámetro interior se utiliza la sección marca­da como patillas superiores.

Para determinar la profundidad de una pieza se emplea la sección mar­cada como sonda.

Reporte de trabajo

"ombre: Grupo: _

Turno: Fecha: Revisado:

Maestro: _

Física l/Manual de experimentos 8

Page 101: Fisica_I

Nombre: _ Grupo: __ Fecha:

Tabla de datos

Cilindro Medida Medida Medida de externa interna profundidad

Vernier

Regla Valero

Vernier

Regla

1. Toma medidas exteriores, interiores y de profundidad de las figuras que te faci­

lite el maestro. Utiliza el Vernier y la regla para todas las mediciones y anota los

resultados en la tabla.

2. Si existe alguna diferencia entre la medición de un aparato y otro, explica el por­qué de la misma.

3. ¿Qué aparato consideras que es más preciso? Explica tu respuesta.

4. ¿De qué depende el grado de precisión de un instrumento de medición?

5. ¿De qué dependerá la selección de un instrumento de medición?

Física l/Manual de experimentos 9

Page 102: Fisica_I

Nombre: _ Grupo: _ Fecha:

Experimento 3

Laboratorio de gráficas I "Un hombre sabio querrá estar siempre con quien sea mejor que él".

Platón

Objetivo Que el alumno practique el uso de instrumentos de medición como la regla y el cro­nómetro, en la medición de valores de dos cantidades físicas de un mismo fenóme­no y trazará el gráfico con los datos recolectados.

Análisis teórico La interpretación de gráficas es de gran importancia para la comprensión de la rela­ción que existe entre dos variables que describen un fenómeno físico.

En el caso de tener una recta como gráfica, se interpreta el hecho de que la rela­ción entre las variables gratificadas es "directamente proporcional" y podemos de­cir que la pendiente de dicha recta es la constante física necesaria para obtener la relación matemática del fenómeno en estudio.

Material 1 Riel de flotación. 1 Bomba de aire. 1 Cinta de medir. 1 Banda de hule (liga). 1 Cronómetro. 1 Hoja de papel milimétrico. 1 Regla de 30 cm.

Procedimiento 1. Instalar y nivelar el riel de flotación. 2. Probar el lanzamiento del deslizador con ayuda de una banda de hule (liga). 3. Marcar el punto inicio en el lugar donde la fuerza de la liga ya no actúe. 4. Medir y marcar, a partir del inicio, cada 20 cm hasta llegar a 100 cm. 5. Medir con el cronómetro el tiempo que tarda el deslizador en recorrer 20 cm. 6. Repetir la medición anterior esta vez para 40,60,80 Y100 cm, respectivamente. 7. Anotar los datos obtenidos en la tabla.

Recomendación. Repite cada toma del tiempo (para cada distancia) por lo menos 3 veces y luego calcula el tiempo promedio para usarlo en la tabla de datos.

Física l/Manual de experimentos 11

Page 103: Fisica_I

Tabla de datos

Tiempo (s) Distancia (cm)

Con los datos de la tabla, traza una gráfica en la hoja de papel milimétrico.

I I I

I ; I

, I ¡ ! I I ~ I I I

[ I I ¡ ,, I [

, , ; I

I ! , ,

¡ I

; j

! , ,

I ! I j ,

,

; I i

I I ¡

I

I [

,

, ,, I

I I I I I I I

I

!

i I

, I I I I I I !

I I I I I I ~ I I I I I I I

I I I I I I I

I I I I I I I I I I I I I I I I I

I I I I I I I I I I

I I I I

I I I I I I

I

12 Física l/Manual de experimentos

Page 104: Fisica_I

Nombre: _ Grupo: _ Fecha:

Reporte de trabajo

ombre:

Turno:

Maestro:

Fecha:

_

Grupo:

Revisado:

_

1. ¿Cuál es la forma de la gráfica que obtuviste?

2. De acuerdo con el análisis teórico, ¿cuál es el tipo de relación que existe entre las dos variables que mediste en este experimento?

3. Calcula el valor de la constante de proporcionalidad entre las dos variables uti­lizando la gráfica, ¿qué resultado obtienes?

4. Escribe la forma matemática de este fenómeno con las dos variables y el valor de la constante.

5. Recuerda que debes utilizar el software Excel para trazar la gráfica y realizar su análisis.

Física l/Manual de experimentos 13

Page 105: Fisica_I

Nombre: _ Grupo: _ Fecha:

Experimento 4

Laboratorio de gráficas 11 "Es de gran alivio conocer las propias limitaciones".

Albert Einstein

Objetivo Que el alumno practique el uso de instrumentos de medición como la regla, el cro­nómetro y el riel de flotación, para la medición valores de dos cantidades físicas de un mismo fenómeno. Trazará el gráfico con los datos recolectados.

Análisis teórico La interpretación de gráficas es de gran importancia para comprender la relación que existe entre dos variables que describen un fenómeno físico.

En el caso de tener una curva (parábola) como gráfica, se desprende el hecho de que la relación entre las variables graficadas es "proporcional al cuadrado" con lo cual es posible obtener que, en la relación matemática se tiene la variable indepen­diente al cuadrado.

Materiales 1 Riel de flotación. 1 Bomba de aire. 1 Cinta de medir. 1 Cronómetro. 1 Hoja de papel milimétrico. 1 Regla de 30 cm.

Procedimiento 1. Instalar y colocar con cierta (mínima) inclinación el riel de flotación. 2. Probar el lanzamiento del deslizador. 3. Marcar el punto inicio. 4. Medir y marcar, a partir del inicio, cada 20 cm hasta llegar a 100 cm. 5. Medir con el cronómetro el tiempo que tarda el deslizador en recorrer 20 cm. 6. Repetir la medida anterior para 40,60,80 Y100 cm, respectivamente. 7. Anota los datos obtenidos en la tabla.

Recomendación. Repite cada toma del tiempo (para cada distancia) por lo menos 3 veces y luego calcula el tiempo promedio y úsalo en la tabla de datos.

Física l/Manual de experimentos 15

Page 106: Fisica_I

Tabla de datos

Tiempo (s) Distancia (cm)

Con los datos de la tabla, traza la gráfica en la hoja de papel milimétrico.

I I

I I ; ,

i I I I

I I I ; ; , I

I i I , ¡

I I I I

I I I I I

I I ¡ i I

! , ! , I i !

I I I I I I I ! I J i ,

, , : I I I , I I

I I

I I I

I I

i ! , , :

1 ! I

I !

I I I I I

, , i I

!

. :

I I

I I

I

I I

i I

I I I I , ; i

,

; , I I I I ,

, ! ! ! I I I !

, ; I I I I I I I ;

! ! ¡ I I . , , : I : i ¡ : ¡ I I , I

I !

; I

I I

I

I , :

I

I

I ¡ ,

I

: . I I I

16 Física l/Manual de experimentos

Page 107: Fisica_I

Nombre: _ Grupo: _ Fecha:

Reporte de trabajo

Nombre:

Turno:

Maestro:

Fecha:

_

Grupo:

Revisado:

_

1. ¿Cuál es la forma de la gráfica que obtuviste?

2. De acuerdo con el análisis teórico, ¿cuál es el tipo de relación que existe entre las dos variables que mediste en este experimento?

3. Calcula el valor de la constante de proporcionalidad entre las dos variables uti­lizando la gráfica, ¿cuál es éste?

4. Escribe la forma matemática de este fenómeno con las dos variables y el valor de la constante.

5. Utiliza el software de Excel para trazar la gráfica y realizar su análisis.

Física l/Manual de experimentos 17

Page 108: Fisica_I

--------------Nombre: Grupo: __ Fecha:

Experimento 5

Movimiento rectilíneo uniforme "En la naturaleza no hay nada más antiguo que el movimiento y son muchos y amplios los libros que los filósofos le han dedicado, sin embargo yo he descubierto que existen muchas cosas de interés acerca de éste, que hasta ahora han pasado inadvertidas".

Galileo Galilei

Objetivo Que el alumno observe un movimiento rectilíneo uniforme e identifique las caracte­rísticas fundamentales que lo definen a partir de datos experimentales de posición, en función del tiempo; para ello utilizará instrumentos de medición de mayor pre­cisión así como el programa Excel para trazar el gráfico con el cual se apoyará para su análisis.

Análisis teórico Determinado cuerpo efectúa un movimiento rectilíneo uniforme, si éste se mueve en línea recta recorriendo iguales desplazamientos, en inten'alos sucesivos de tiem­po iguales. Para describir gráficamente este movimiento, es necesario construir una tabla de datos teniendo como variables la posición y el tiempo. Será necesario tomar un punto de referencia para determinar la posición, siendo ésta la distancia que exis­te entre el punto de referencia y el objeto en cualquier instante. Con los datos de la tabla se debe construir una gráfica de posición contra tiempo, tomando la posición como variable dependiente y el tiempo como la variable independiente para así de­terminar la relación correspondiente entre las mismas.

Material 1 Sistema de flotación lineal. 1 Bomba de aire. 1 Generador de chispas. 1 Deslizador con electrodo de chispas. 1 Regla de chispeo. 1 Banda de hule (liga). 1 Papel de registro. 1 Trozo de hilo. 1 Regla graduada. 1 Computadora con Excel.

Procedimiento 1. Instala el equipo de flotación y verifica su nivel. 2. Coloca la tira de papel de registro en la regla de chispeo y conecta el generador

de chispas al sistema de flotación.

Física l/Manual de experimentos 19

Page 109: Fisica_I

3. Coloca sobre la guía del sistema de flotación un deslizador (de masa conocida) con el electrodo de chispeo, ajustando con las manos una de las puntas del mis­mo a una distancia de 0.5 cm (aproximadamente) de la tira de papel de registro, y la otra al alambre que se encuentra a lo largo del riel.

4. Con un lápiz sujeta el deslizador del pasador metálico. 5. Enciende la bomba de aire y el generador de chispas. Selecciona la frecuencia de

chispeo adecuada. 6. Prepara el deslizador y oprime el botón del generador de chispas, para tomar el

segundo punto como referencia del movimiento. 7. Lanza el deslizadar quitando el lápiz y liberando la liga que se utiliza como sis­

tema de lanzamiento y efectúa el registro. Finaliza el registro antes de que el des­lizador llegue al otro extremo del sistema de flotación.

8. :v1ide la distancia que hay entre el punto de referencia del registro y cada uno de los puntos seleccionados. Con los valores obtenidos construye tu tabla de datos.

Tabla de datos Núm. de intervalo Tiempo (s) Posición (cm) Velocidad (cm/s)

1

2

3

4 I

5

6

7

8

9

20 Física l/Manual de experimentos

Page 110: Fisica_I

Nombre: _ Grupo: __ Fecha:

! I

I ,

I I

I II I

I I

I I I I

! I

I ¡ i ,

Reporte de trabajo

Nombre: Grupo: _

Turno: Fecha: Revisado:

Maestro: _

1. Traza la grafica de posición contra tiempo (5 us. t) en la hoja con cuadrícula.

2. ¿Qué tipo de gráfica obtuviste?

3. ¿Cuál es el valor de la pendiente de la recta?

4. Calcula la velocidad para cada intervalo con la ecuación u = s / t Y escribe los resultados en la tabla de datos.

5. ¿Qué tienen en común los valores calculados para cada intervalo?

Física l/Manual de experimentos 21

Page 111: Fisica_I

6. ¿Qué tienen en común los valores calculados de la velocidad y la pendiente de la recta?

7. De acuerdo con las definiciones de los tipos de movimiento, ¿qué tipo de movi­miento es éste?

8. ¿Qué puedes concluir de la gráfica obtenida con respecto al movimiento rectilí­neo uniforme?

22 Física l/Manual de experimentos

Page 112: Fisica_I

Nombre: _ Grupo: __ Fecha:

Experimento 6

Movimiento uniforme acelerado "La ciencia llegó del cielo a la tierra por el plano inclinado de Galileo".

Anónimo

Objetivos 1. Que el alumno obtenga la velocidad y la aceleración de un cuerpo con movi­

miento rectilíneo, a partir de procesar datos experimentales acerca de la posición en función del tiempo, e identificar las características de estas magnitudes en gráficos de posición contra tiempo, así como de velocidad contra tiempo y de aceleración contra tiempo.

2. Que el alumno encuentre la relación matemática entre el desplazamiento de un cuerpo y el tiempo de duración.

Análisis teórico El movimiento rectilíneo uniformemente acelerado se efectúa a lo largo de una tra­yectoria recta con aceleración constante.

La aceleración se define como el cambio de la velocidad con respecto al tiempo. Si la velocidad aumenta, la aceleración es positiva, y si la velocidad disminuye, la aceleración es negativa.

La aceleración es constante, cuando su magnitud, dirección y sentido son cons­tantes. es decir, que no varían con el tiempo.

Una de las formas más fáciles de visualizar las características esenciales de un fenómeno, estudiado experimentalmente, consiste en representar a través de gráfi­cas los resultados numéricos correspondientes a las mediciones efectuadas, ya que, además de su análisis, se puede obtener más información con cálculos básicos para aprender y aplicar convenientemente las diversas técnicas para granear. En esta prác­tica se aplicará una de estas técnicas llamada cambio de variable.

Muchas de las leyes físicas con las cuales trabaja un estudiante, se expresan me­diante funciones del siguiente tipo:

(1)

donde x y y son la variable independiente y dependiente, respectivamente y siendo a y n constante real positiva o negativa.

Esta expresión significa que y y x son proporcionales. A la constante a se le de­nomina como constante de proporcionalidad. .

'- ­A continuación se muestran ejemplos de leyes físicas que pueden expresarse co­

mo funciones del tipo (1):

1) F = ma

Proporcionalidad directa (n = 1). Constante de proporcionalidad m:

2) h = tgt2 Proporcionalidad directa al cuadrado (n = 2). Constante de proporcionalidad 1/2 g.

Física l/Manual de experimentos 23

Page 113: Fisica_I

3) F = Gm 1n /? Proporcionalidad inversa al cuadrado (n = -2).l 2 Constante de proporcionalidad G.

A partir de los casos de leyes físicas, se aprecia que, un gran número de ellas se ex­presan por funciones del tipo (1), pero siendo un poco más rigurosos en la observa­ción de dichos casos, se destaca que los valores más frecuentes de 11 son: 1, -1,2, - 2 y' /:. Sin negar que n puede tomar otros valores muy distintos a éstos, a continua­ción se analizarán las gráficas representativas para n = 1Y11 = 2.

1) Para n = 1, Y = ax

Se trata de una recta que pasa por el origen y cuya pendiente es a:

y

3a

2a

a

x

Figura 1. La función y=xn representa una recta cuando n=1.

2) Para 11 = 2, entonces y = ax2

Se trata de una parábola que pasa por el origen, pero no tiene pendiente constante o única, entonces el valor de la misma deberá interpretarse de la manera que se expone a continuación.

y

14a

12a 10a /

8a

6a

4a 2a

o 2 3 4 x

Figura 2. La función y= axn representa una parábola cuando n= 2.

24 Física l/Manual de experimentos

Page 114: Fisica_I

Como tú ya sabes¡ en la realización de experimentos se pueden obtener tabula­ciones y éstas¡ al granearse¡ muestran la relación entre las variables medidas en el experimento. Dicha relación puede ser de cualquier tipo¡ pero se destacaron dos de las más comunes que obedecen a la función (1).

Uno de los propósitos de los experimentos consiste en hacer mediciones que per­mitan establecer la relación matemática que satisfacen las variables propias del fe­nómeno en investigación. En ese caso¡ por simple inspección de la gráfica¡ puede concluirse si ésta pertenece a alguno de los tipos aquí citados¡ y de ser ast determi­nar si el valor del exponente n es positivo¡ negativo o fraccionario. Sin embargo¡ la función no estará bien definida mientras no se conozcan los valores de a y n. Si la gráfica es una rectal n =1 Ysu pendiente es a: entonces¡ ¿cómo encontrar dichos va­lores en los demás casos?

Para lograrlo se emplean diversas técnicas y aquí utilizaremos la técnica del cam­bio de variable¡ la cual ilustraremos con el siguiente ejemplo.

Supongamos que al medir el desplazamiento y el tiempo de un cuerpo en un experimento¡ se obtuvo la siguiente tabulación:

Tabla 1

t (s) h (m)

0.20 0.2 0.28 0.4 0.35 0.6 0.40 0.8

h(m)

0,8 +------------------T-------1

0.6 +-------------~---------1

0.4 +---------------:;¡¡~----------1

0,2 +-------__""--------------1

O>--.......o!~=__ ____.J

0,1 0.2 0.3 0.4 0,5

Figura 3. Esta gráfica representa las mediciones mostradas en la tabla 1.

Física 1/:vIanual de experimentos 25

Page 115: Fisica_I

La gráfica de estos valores se muestra en la figura 3, la cual, al ser comparada con la figura 2 muestra una parábola. El problema es encontrar la relación entre n y t. Por la observación de la curva se deduce que h es proporcional a t elevado al cuadrado. Si esto último fuera cierto la gráfica de h en función de r daría una recta pasando por el origen, entonces:

(2)

donde m sería la pendiente de esta ecuación. Elevando al cuadrado el tiempo, tenemos la siguiente tabla:

Tabla 2 t2 (S2) h (m)

0.04 0.2

0.078 0.4

0.123 0.6

0.160 0.8

0,8

e 06'0 '(3 '¿;j O

O- 0.4

0.2

Tiempo al cuadrado

Figura 4. Esta gráfica representa la función que relaciona a las variables mostradas en la tabla 2.

La gráfica de la tabla 3 se muestra en la figura 4, la cual representa una recta. Podemos calcular su pendiente tomando dos puntos y utilizando la siguiente

relación.

m = (h2 - h1) / (t2 - ti), sustituyendo los valores de dos puntos cualesquiera te­nemos que:

m = (0.4 - 0.2)/(0.078 - 0.040) = 5.1 m/s2

En consecuencia, la relación buscada es:

h = 5.1 t2

26 Física l/Manual de experimentos

Page 116: Fisica_I

Nombre: _ Grupo: _ Fecha:

Si relacionamos la ecuación anterior con la ecuación de movimiento uniforme­mente acelerado (s =1/2 at2), podemos concluir que la pendiente (m) equivale a 1/2 dea.

Material 1 Sistema de flotación. 1 Bomba de aire. 1 Deslizador. 1 Portapolea y polea. 1 Juego de pesas. 1 Generador de chispas. 1 Cinta de registro. 1 Trozo de hilo. 1 Juego de marcadores de color.

Procedimiento 1. Instala el equipo de flotación con un pequeño ángulo de inclinación.

2. Coloca la cinta de registro sobre la regla de chispeo.

3. Coloca el deslizador en el extremo más alto del riel.

4. Enciende la bomba de aire.

5. Enciende el generador de chispas y deja que dé unos cuantos chispazos para mar­car el punto de inicio.

6. Al soltar el deslizador debes mantener oprimido el generador de chispas hasta el momento antes de que el deslizador llegue al otro extremo del sistema de flota­ción.

7. Marca estos resultados en la cinta, utilizando para ello un marcador de cierto color.

8. Efectúa el experimento dos veces más y marca con distintos colores los resulta­dos obtenidos.

9. Determina los valores promedio de los datos obtenidos.

Tabula los valores promedio de la posición, para cada tiempo determinado en el ex­perimento.

Física l/Manual de experimentos 27

Page 117: Fisica_I

Intervalo

1 2

3

4

5

6 7

t (8) S (cm) t2(82) a (cm/52) V (cm/s)

I I

i .

!

I I

l

I I I I I

I '

, ,

I I

I ,

! I

1 •

¡ ¡

i

I I

28 Física l/Manual de experimentos

Page 118: Fisica_I

Nombre: _ Grupo: __ Fecha:

;

I I I I I I , ,

I I I I , I I I I i

I I I I I I I I I i I I

I ¡ I

I I

I I I I I , i

I I I ¡ I

I I I ! !

¡ ,

i

I ¡

, I

I I

!

I ¡ ¡ I I I I

I I I

I I I

I . I ; I I í I I

I I

I I I : I i ; I

I I I I I I

I I I I I

I I

I I I I I ,

! I , , ! , ! : ,

I I I

,, [ 1

, I

j

I I , I , , I I I I I i I I I

I I I ! I i

I i ,

I

í I ¡ I I

I ,

Física l/Manual de experimentos 29

Page 119: Fisica_I

, I I

: , I I I i

I I I I I ; : , ; iI

, I ' I

I I I ! I . I I ; II I

I : I !

I I I I , I I I I I I I I

II I I I I i I I I I

I I I I I ' I

I I I I I I I

¡ I , I , , i I ¡i , ¡ r I !

I r I ; i . ! I , I , I I i I I

i I I I , I I

I I I , I I I : ,

I I I I ! I I I I

I I I I I I

I , I

Reporte de trabajo

ombre: Grupo: _

Tumo: Fecha: Revisado:

Maestro: _

1. En la hoja cuadriculada realiza la gráfica de posición contra tiempo (s vs. t).

2. ¿Qué tipo de gráfica es?

3. ¿Cuál es el tipo de relación entre las variables?

4. En otra hoja con cuadrícuJa realiza la gráfica de posición contra tiempo al cua­drado (5 vs. t2

).

5. ¿Qué tipo de gráfica es?

6. Calcula la pendiente de la gráfica anterior.

7. Con el valor de la pendiente, calcula el valor de la aceleración y añade este va­lor en la tabla de datos.

30 Física l/Manual de experimentos

Page 120: Fisica_I

ombre: _ Grupo: _ Fecha:

8. Con la fórmula s = t (V + Vo)/2 calcula la velocidad final, considera que la ve­locidad inicial es igual a cero. Escribe estos valores en la tabla de datos.

9. Realiza la gráfica de velocidad contra tiempo en otra hoja de cuadrícula.

10. ¿Cómo es la pendiente de la gráfica anterior?

11. Calcula la pendiente de la gráfica V vs. t y menciona qué representa.

12. Compara el valor de esta pendiente con el valor de la aceleración calculado an­teriormente y si existe alguna diferencia entre éstas explica él porqué de ello.

13. Con el valor de la pendiente (que es la constante de proporcionalidad), escribe la relación matemática entre el desplazamiento y el tiempo.

Física l/Manual de experimentos 31

Page 121: Fisica_I

Nombre: _ Grupo: _ Fecha:

Experimento 7

Caída libre de los cuerpos "El verdadero sabio sólo es riguroso consigo mismo; con los demás es amable".

Plutarco

Objetivos 1. Que el alumno obtenga el valor de la aceleración de la gravedad para un cuer­

po en "caída libre", a distintas alturas, a partir de datos experimentales de la al­tura en función del tiempo, utilizando métodos analíticos.

2. Que el alumno obtenga el valor de la aceleración de la gravedad para objetos en "caída libre", con distintas masas y desde la misma altura, a partir de datos ex­perimentales de la altura en función del tiempo, utilizando métodos analíticos y encuentre si existe alguna dependencia entre la masa y la gravedad.

Análisis teórico Un cuerpo está en caída libre si desciende hacia la superficie de la Tierra, sin que tenga algún tipo de resistencia. Si un cuerpo tiene una razón pequeña entre su área y su volumen, dicho cuerpo tendrá una muy baja resistencia cuando se deje caer, por tanto se puede considerar que está en caída libre.

La aceleración de la gravedad es una cantidad vectorial cuya dirección está orien­tada hacia el centro de la Tierra, además, su valor varía según el lugar, para fines prácticos se considera como una constante, dada por g = 9.8 m/ S2 o bien g = 32 pies/s2.

Cuando dejamos caer un objeto desde cierta altura, observamos que éste aumen­ta su velocidad, debido a que es un movimiento uniformemente acelerado, con di­rección hacia abajo.

Material 1 Pedestal. 1 Cronómetro digital. 1 Imán de sujeción. 3 Esferas de acero de diferente peso.

Procedimiento 1. Toma en cuenta el montaje de la figura 1 para realizar el experimento.

Física 1/Manual de experimentos 33

Page 122: Fisica_I

Imán de sujeción

h

Figura 1

2. Coloca el imán de sujeción a la altura H indicada por tu maestro.

Nota: Recuerda que para la altura h entre las tapas interiores deberás tomar en cuen­ta el diámetro de la esfera (figura 2).

Imán de sujeción c:::==:j ~.. A---"C""""")-----!.-----'----d------,.----­

1 h =H·d H

.. B .. c=:::=:::JL- --'

Interruptor

Figura 2

3. Enciende el cronómetro y selecciona la escala que corresponde a milésimas de segundo.

4. Energiza el imán de sujeción oprimiendo la tecla INICIAR el cronómetro. Sin dejar de oprimir, coloca la esfera en el imán de sujeción y retira la mano de la es­fera la cual deberá quedar sujeta al imán.

34 Física l/Manual de experimentos

Page 123: Fisica_I

Nombre: _ Grupo: _ Fecha:

Nota: La acción de oprimir la tecla INICIAR¡ deberá ser lo más bre\'e posible¡ con el objeto de evitar que se magnetice la esfera y retarde la caída.

5. Acciona la fuente del imán para dejar caer la esfera y lee el tiempo marcado en el cronómetro¡ repite el experimento dos veces más y calcula el tiempo prome­dio. Anota los resultados en la tabla 1.

6. Coloca el imán de sujeción a dos diferentes alturas (tres en total) y anota el tiem­po de caída¡ haz esto tres veces para cada posición. Tabula estos tiempos para cada altura y calcula el tiempo promedio para cada una de ellas en la tabla 1.

7. Efectúa la misma operación pero con esferas de diferentes masas y a una misma altura¡ después registra los datos en la tabla 3.

Reporte de trabajo

Nombre: Grupo: _

Turno: Fecha: Revisado:

Maestro: _

1. Para cada altura encuentra el tiempo promedio y anota los datos en la siguien­te tabla:

Tabla 1

H tI t2 t, t (prom) a J Ó

2. Por medio de la ecuación g =2 h/t2 encuentra el valor de la aceleración de la gra­vedad para cada altura y anota los datos en la tabla 2.

Tabla 2

h tprom g

I

Física l/Manual de experimentos 35

Page 124: Fisica_I

3. ¿Qué diferencia existe entre el valor calculado de g y el valor teórico para una masa que cae libremente desde diferentes alturas?

4. Calcula ahora el valor de g para diferentes masas y anota los datos en la tabla 3.

Tabla 3

m h (cte) tI t2 t3 tprom o ó

I

I

5. Compara entre sí los valores de g que obtuyiste para diferentes masas, ¿cuál es tu conclusión?

36 Física l/Manual de experimentos

Page 125: Fisica_I

UNIVERSIDAD AUTÓNOMA DE NUEVO LEÓN

ING. JOSÉ ANTONIO GONZÁLEZ TREVIÑO Rector

DR. JESÚS ANCER RODRÍGUEZ Secretario General

DR. UBALDO ORTIZ MÉNDEZ Secretario Académico

ING. JERÓNIMO ESCAMILLA TOVAR Director de Estudios de Nivel Medio Superior

San Nicolás de los Garza, Nuevo León, agosto de 2007.

Guía del estudiante

Page 126: Fisica_I

Nombre: _ Grupo: __ Fecha:

Autoevaluación 1. En el movimiento rectilíneo uniforme, dos cantidades físicas que poseen siem­

pre la misma magnitud (diferente de cero) son:

a) La distancia y el tiempo.

b) La rapidez y la aceleración.

e) La distancia y el desplazamiento.

d) La aceleración y la distancia.

e) La distancia y la velocidad.

2. En el movimiento rectilíneo uniforme, dos cantidades físicas que poseen siem­pre la misma magnitud (diferente de cero) son:

a) La distancia y el tiempo.

b) La velocidad y la aceleración.

e) La rapidez y la velocidad.

d) La distancia y la velocidad.

3. Si un móvil con movimiento rectilíneo uniforme recorre 60 m en 12 s, 25 m los recorrió en:

a) 6 s b) 5 s c) 4 s d) 4.5 s e) 5.5 s

4. Si un móvil con movimiento rectilíneo uniforme recorre en 8 s 40 m, en 4.5 s recorrió:

a) 17.5 m b) 27.5 m e) 20.5 m d) 21 m e) 22.5 m

5. De los siguientes gráficos, el que nos informa del movimiento rectilíneo uni­formemente acelerado que posee un móvil es:

A) s B) v e) a

E) a ...

Física l/Guía del estudiante 39

Page 127: Fisica_I

6. De los siguientes gráficos¡ el que nos informa del movimiento rectilíneo uniforme que posee un móvil es:

A) s... B) v e) a'"

O) a

E) 'l:::. I t

7. La pendiente en un gráfico que muestra la posición de un móvil en función del tiempo representa:

a) La distancia recorrida por el móvil.

b) El tiempo transcurrido.

el La aceleración del móvil.

d) La velocidad del móvil.

e) Ninguna de las anteriores.

8. La pendiente en el gráfico v X t (velocidad-tiempo)¡ que muestra la velocidad de un móvil en función del tiempo¡ representa:

a) El desplazamiento realizado por el móvil.

b) El tiempo transcurrido.

el La aceleración del móvil.

d) La velocidad del móvil.

e) Ninguna de las anteriores.

9. Si la aceleración de un objeto es cero¡ esto significa que el objeto:

a) No se mueve.

b) No varía su velocidad.

e) Su velocidad cambia uniformemente.

d) Su velocidad cambia irregularmente.

e) a y b son correctas.

10. Si un móvil se desacelera significa que:

a) Su velocidad es negativa.

b) Su desplazamiento es negativo.

40 Física l/Guía del estudiante

Page 128: Fisica_I

Nombre: _ Grupo: __ Fecha:

e) La distancia que recorre en cada segundo es cada vez menor.

d) La distancia que recorre en cada segundo es cada vez mayor.

e) La distancia que recorre cada segundo es la misma.

11. Un móvil se acelera a razón de 4 miS2, esto significa que el móvil:

a) Recorre 4 metros cada segundo.

b) Tarda 4 segundos en recorrer 1 metro.

e) Su velocidad cambia 4 mi s cada segundo.

d) En cada S2 recorre 4 metros.

12. Si un móvil se acelera a razón de - 2 miS2/ esto significa que el móvil:

a) Recorre - 2 metros cada segundo.

b) Tarda 2 segundos en recorrer -1 metro.

c) Su velocidad cambia 2 mis cada segundo.

d) En cada S2 recorre 2 metros.

13. La ecuación s = vt se aplica cuando:

a) La rapidez es constante.

b) La aceleración es constante.

e) La distancia es constante.

d) La rapidez no es constante.

e) Nada de lo anterior.

14. Un cuerpo que se desplaza con aceleración constante, debe experimentar cam­bios en:

a) La velocidad.

b) La masa.

e) La aceleración.

d) El peso.

e) Ninguna de las anteriores.

15. La rapidez de un cuerpo que se mueve en línea recta con aceleración positiva constante y aumenta proporcionalmente,con respecto a:

a) La distancia recorrida.

b) El tiempo transcurrido.

e) El desplazamiento realizado.

d) El cuadrado de la distancia recorrida.

e) Nada de lo anterior.

Física 1I Guía del estudiante 41

Page 129: Fisica_I

16. Un ratón corre a lo largo de un túnel recto y angosto. Si su gráfica de velocidad V5. tiempo es una recta paralela al eje del tiempo, la aceleración es:

a) Constante y diferente de cero.

b) Cero.

c) Variable.

d) Cuadrática.

e) Inversa.

17. Si [L] representa la dimensión de la longitud y [T] del tiempo, entonces las dimensiones de la aceleración son:

b) [L / T] e) [U / T] d) [L / T2] e) [T / L]

18. Tomando como positiva la dirección del movimiento inicial, la pendiente de la gráfica de velocidad U5. tiempo para un cuerpo que se mueve en línea recta es negativa cuando:

a) La aceleración tiene sentido contrario a la del mO\'imiento inicial.

b) La aceleración hace aumentar la rapidez.

c) La aceleración hace aumentar la velocidad.

d) La aceleración es constante y tiene el mismo sentido que el mm'imiento inicial.

19. Si el desplazamiento de un cuerpo es una nmción cuadrática del tiempo, el cuer­po se mueve con:

a) Aceleración no constante.

b) Aceleración constante diferente de cero.

c) Rapidez uniforme.

d) Velocidad uniforme.

e) Aceleración cero.

20. Si se desprecia la fuerza de fricción y dos esferas del mismo tamaño (una de plástico y otra de plomo) son dejadas caer desde la misma altura de la super­ficie terrestre, entonces:

a) La de plomo tarda menos tiempo en caer.

b) La "v " de la de plomo será mayor. o

e) Llegarán al suelo con la misma velocidad ambas esferas.

d) La de plástico se acelera menos.

e) Todas son correctas.

42 Física 1/Guía del estudiante

Page 130: Fisica_I

Nombre: _ Grupo: _ Fecha:

21. Si se desprecia la fuerza de fricción y dos esferas del mismo tamaño (una de hule y otra de acero) son dejadas caer desde la misma altura de la superficie terrestre, entonces:

a) Tardarán el mismo tiempo en llegar al suelo.

b) Llegarán al suelo con la misma velocidad.

e) Tendrán la misma aceleración.

d) Sus velocidades iniciales tendrán el mismo valor.

e) Todas son correctas.

22. En el vacío el valor de g en la superficie terrestre es de aproximadamente 9.8m/s2, esto significa que un objeto que cayera en estas condiciones cerca de esta superficie:

a) Recorrería 9.8 m cada segundo.

b) Se desplazaría con una velocidad de 9.8 mis.

e) Disminuiría su velocidad 9.8 mis cada segundo.

d) Aumentaría su velocidad 9.8 mis cada segundo.

e) Tardaría 9.8 s en caer.

23. En el vacío el valor de g en la superficie lunar es de aproximadamente 1.6 miS2,

esto significa que un objeto que cayera en estas condiciones cerca de esta su­perficie:

a) Recorrería 1.6 m cada segundo.

b) Tardaría 1.6 s en caer.

e) Se desplazaría con una velocidad de 1.6 mis.

d) Disminuiría su velocidad 1.6 mis cada segundo.

e) Aumentaría su velocidad 1.6 mis cada segundo.

24. Si se desprecian las fuerzas de fricción con la atmósfera y dos objetos son deja­dos caer desde la misma altura, pero uno en la Luna y otro en la Tíerra, entonces:

a) El objeto en la Tierra llegará al suelo en mayor tiempo.

b) El objeto en la Luna llegará al suelo en mayor tiempo.

e) El objeto en la Tierra llegará con menor velocidad al suelo.

d) El objeto en la Luna llegará con mayor velocidad al suelo.

e) Tardaría 1.6 s en caer.

25. Si se desprecian las fricciones en la atmósfera y se dejan caer dos objetos desde la misma altura, pero uno en la Luna y otro en la Tierra (la g lunar es aproxi­madamente 1/6 del valor de lag terrestre), entonces:

a) El objeto en la Tierra llegará en mayor tiempo al suelo.

Física 1/Guía del estudiante 43

Page 131: Fisica_I

b) El objeto en la Luna llegará en menor tiempo al suelo.

c) El objeto en la Luna llegará con mayor velocidad al suelo.

d) El objeto en la Tierra llegará con mayor velocidad al suelo.

26. Si se desprecia la fricción con el aire y el objeto B es dejado caer desde el doble de altura que el objeto A con respecto a la superficie terrestre, entonces:

a) El objeto B tardará el doble de tiempo en caer que el objeto A.

b) El objeto B llegará al suelo con el doble de velocidad que el objeto A.

c) El objeto B sufre la misma aceleración que el objeto A.

d) El objeto B sufre el doble de aceleración que el objeto A.

e) El objeto A sufre el doble de aceleración que el objeto B.

27. Si se desprecia la fricción con el aire y el objeto A es dejado caer desde la mitad de la altura que el objeto B con respecto a la superficie terrestre, entonces:

a) El objeto A tardará la mitad de tiempo en caer que el objeto B.

b) El objeto A llegará al suelo con la mitad de velocidad que el objeto B.

e) El objeto A sufre la mitad de la aceleración que el objeto B.

d) El objeto A sufre la misma aceleración que el objeto B.

e) El objeto B sufre la mitad de la aceleración que el objeto A.

28. Un saco de arena dejado caer desde un globo de aire caliente llega al suelo con cierta rapidez. El globo se eleva lentamente y se detiene. Si entonces se deja caer un segundo saco idéntico al primero y llega al suelo con una rapidez doble en comparación con la del primero, ¿qué altura tenía el globo al soltar el segundo saco, en comparación con la que tenía al soltar el primero? (Se desprecia la resistencia del aire.)

a) t de la altura.

b) 2 veces la altura.

e) 8 veces la altura.

d) 4 veces la altura.

e) Ninguna de las anteriores.

29. Dos esferas, El y Ez de radio 0.1 m y de pesos PI y Pz ' se dejan caer desde

una altura de 3 m en el mismo lugar y al mismo tiempo. Se puede afirmar (si se desprecia la resistencia del aire) que:

a) El y Ez llegarán juntas al suelo solamente si PI YPz son iguales.

b) Si PI fuera mayor que Pz' El llegará primero al suelo.

e) El y Ez llegarán juntas al suelo, a pesar de que sus pesos fueran diferentes.

44 Física l/Guía del estudiante

Page 132: Fisica_I

Nombre: _ Grupo: __ Fecha:

d) La esfera que tuviera mayor densidad llegará primero al suelo.

e) Si PI fuera menor que P2, E2 llegará primero al suelo.

30. Dos cuerpos parten en caída libre en el mismo instante. El cuerpo A sale con una velocidad inicial va hacia abajo, mientras que B parte del reposo. Si A pesa más que B, de los gráficos siguientes, el que muestra correctamente el cambio de velocidad de los dos cuerpos con respecto al tiempo es:

31. Se arroja una pelota verticalmente hacia arriba en un lugar donde la aceleración de la gravedad es 9.8 m/s2 y la fricción con el aire es despreciable. En el punto más alto de su trayectoria la velocidad es igual a cero. En ese punto, la ace­leración de la pelota es:

a) También es cero.

b) Vertical hacia arriba y vale 9.8 m/ S2.

c) Vertical hacia abajo y mayor que 9.8 m/s2.

d) Vertical hacia abajo y vale 9.8 m/52.

e) Vertical hacia abajo y menor que 9.8 m/s2.

Física l/Guía del estudiante 45

Page 133: Fisica_I

32.� En un experimento se verificó que la velocidad inicial necesaria para que un cuerpo alcance una altura H cuando es lanzado verticalmente hacia arriba¡ era igual a vo' Si el mismo cuerpo fuera lanzado con una velocidad inicial igual a 2v

la nueva altura alcanzada (despreciando la resistencia del aire) sería:

a) 2H�

b) H/2�

c) 3H�

d) H/4�

e) 4H�

33.� Desde lo alto de una torre se deja caer un cuerpo A 2.0 segundos después se deja caer otro cuerpo B. Despreciando la fricción del aire se puede afirmar que la distancia entre los dos cuerpos:

a) Permanecerá constante durante la caída de ambos.�

b) Disminuirá¡ si B pesara más que A.�

c) Disminuirá¡ aunque B y A pesen lo mismo.�

d) Aumentará continuamente¡ sin importar los pesos de A y B.�

e) Sólo aumentará si A pesara más que B.�

34.� El diagrama mostrado en la siguiente figura representa la velocidad aproxima­da de un pequeño cohete lanzado verticalmente hacia arriba. De las siguientes afirmaciones¡ la incorrecta es:

a)� Durante los 10 segundos iniciales de movimiento¡ su aceleración es constante e igual a 50 m/s2.

b)� La altura máxima que alcanza es de 1.5 x 104 metros.

c)� El cohete comienza a descender después de 10 segundos.

d)� Durante todo el tiempo que el cohete permanece en el aire¡ los movimientos que describe son uniformemente variados.

e)� La aceleración del cohete 10 segundos después del inicio del movimiento se mantiene constante y con magnitud igual a 10 m/s2 .

v (mis)

500

t(s)

o

·500

46 Física l/Guía del estudiante

Page 134: Fisica_I

Datos físicos*

Cantidad Símbolo Valor aproximado

667 X 10-11 N'm2/kg 2Constante de la gravitación universal G Aceleración debida a la gravedad (valor g 9.80 m/s2 = 980 cm/s2 = 32.2 ft/s 2

generalmente aceptado en la superficie terrestre) Velocidad de la luz e 3.00 X lOs mis = 3.00 X 1010 cmls = 1.86 X 103 mi s

Constante de Boltzmann kB 1,38 X 10-23 J/K

Número de Avogadro ~r" 6.02 X 1023 mol- I

Constante de los gases R = N"kB 8.31 Ji mol'K = 1.99 calj mol'K

Constante de la ley de Coulomb k = l/417Ec' 9.00 X 109 N'm2 'C 2

1.60 X 10- 19 CCarga del electrón e 8.85 X 10-12 C 2 ' N'm2

Constante dieléctrica del espacio libre Ea

4017 X 10- 7 T'm 'A = 1.26 X 10-6 T'm/APermeabilidad del espacio libre ¡.to 1.66 X 10-27 kg -931 MeVUnidad de masa atómica U 6.63 X 1O-3~ hConstante de Planck J¡ 1.05 X 1O-~ hfí = h/h 9.11 X 10-31 kg = 5.-!9 X 1O-~ u -0.511 MeVMasa del electrón me 1.672 62 X 10-27 kg = 1.007276 u -938.27 MeVMasa del protón mp

Masa del neutrón 1110 1.67493 X 1G-:~ kg X 1.008665 u ~939.57 MeV

Radio de Bohr del átomo de hidrógeno n 0.053nm

'Valores de NI5T Reference on Constants, Units and uncertainty

Radio ecuatorial de la Tierra 6.378 X 103 km = 3963 mi Radio polar de la Tierra 6.357 X 103 km = 3950 mi

Promedio: 6.4 X 103 km (para cálculos generales) Masa de la Tierra 5.98 X 102~ kg Diámetro de la Luna 3500 km = 2160 mi Masa de la Luna 7.4 X 1022 kg = s'fmasa de la Tierra Distancia media de la Tierra a la Luna 3.8 X 10° km = 2.4 X 10° mi Diámetro del Sol U X 106 km = 864000 mi Masa del Sol 2.0 X 1030 kg Distancia media de la Tierra al Sol 1.5 X lOs km = 93 X 106 mi

'El apéndice III contiene más datos planetarios.

Símbolos matemáticos - - -­ --

es igual a Alfa A el :\Tu ~ lJ

* no es igual a Beta B f3 Xi - i; es aproximadamente igual a Gamma r Omicron O o aproximadamente Delta -l 5 Pi rr 17

ex es proporcional a Épsilon E 1'; Rho P p > es mayor que Zeta Z [ Sigma 2: (J'

2:: es mayor que o igual a Ha H TI Tau T T

» es mucho mayor que Theta 8 (J Úpsilon y v

< es menor que Iota 1 Phi <P dJ ~ es menor que o igual a Kappa K K chi X X « es mucho menor que Lambda A ;\ Psi 1lJ ¡J; + más o menos Mu '\f ¡.t Omega n úJ

menos o más x valor medio de x ~x cambio en x Ixl valor absoluto de x 2', suma de x infinito

Page 135: Fisica_I